Superpage
Uterus


Copyright: 2002-2024, PathologyOutlines.com, Inc.

Gyn related: Jobs, Fellowships, Conferences, Cases, CME, Board Review, What's New

Related chapters: Cervix, Placenta

Editorial Board oversight: Stephanie L. Skala, M.D. (last revised November 2023)
Page views in 2024 to date: 31

Abnormal uterine bleeding
Definition / general
  • In women of reproductive age: bleeding or spotting between periods, bleeding after sex, abnormally heavy periods or irregularity in timing of periods
  • In postmenopausal women: any bleeding is considered abnormal uterine bleeding (AUB) (ACOG: Abnormal Uterine Bleeding [Accessed 25 August 2023])
Essential features
  • Abnormal uterine bleeding is a clinical, not pathological, diagnosis and leads to large numbers of endometrial biopsies, accounting for most endometrial biopsies encountered in practice
  • After ruling out significant endometrial pathology that can lead to abnormal uterine bleeding (e.g., endometrial polyp, hyperplasia, atypical hyperplasia, carcinoma, leiomyoma [submucosal], endometrial stromal tumors, endometritis, exogenous hormone effects, pregnancy related bleeding), a simplified descriptive sign out of these cases is preferable to a detailed histological description (suggested diagnostic terminology provided in Table 1)
  • In women of reproductive age, the diagnostic sign out reflects the dynamic nature of the endometrium with cyclical changes (unlike small biopsies from most body sites where benign or unremarkable suffice if significant pathology has been excluded)
  • In postmenopausal women, the normal state of the endometrium is atrophic / inactive; anything else is a significant histopathological finding
Terminology
  • Dysfunctional uterine bleeding (DUB)
  • Postmenopausal bleeding (PMB)
ICD coding
  • ICD-10: N93.8 - other specified abnormal uterine and vaginal bleeding
  • ICD-11: GA2Y - abnormal uterine or vaginal bleeding
Epidemiology
  • Women of any age but especially at menarche, perimenopause or after menopause
Sites
  • Endometrium
Pathophysiology
  • Significant pathology that can lead to abnormal uterine bleeding (e.g., endometrial polyp, hyperplasia, atypical hyperplasia, carcinoma, leiomyoma [submucosal], endometritis, exogenous hormone effects) must first be excluded (Medicine (Baltimore) 2018;97:e11457, Hum Reprod Update 2023;29:457)
  • In the absence of a specific pathological diagnosis, as noted above, the pathophysiology of abnormal uterine bleeding is variable depending on age
  • In premenopausal or perimenopausal women, it is often related to irregularity in the pituitary - gonadal axis, such that the endometrium fails to undergo uniform cycles of growth / secretory change / apoptosis
  • In postmenopausal women, the most common cause is atrophy with bleeding from fragile vessels in the thinned endometrium
  • Other causes of abnormal uterine bleeding that will not appear in an endometrial biopsy include adenomyosis or endometriosis
Etiology
Clinical features
Diagnosis
  • Low power examination of the biopsy will establish adequacy of the specimen and rule out most of the important diagnoses that must be excluded
  • Having excluded endometrial polyp, hyperplasia, atypical hyperplasia, carcinoma, leiomyoma, endometritis and exogenous hormone effects, a final diagnosis can be concisely reported from a limited number of descriptive possibilities (see Table 1) (Int J Gynecol Pathol 2019;38:119)

    Table 1
    Diagnosis (choose 1 or more)
    Nondiagnostic sample (no endometrial tissue present)
    Proliferative endometrium
    Secretory endometrium
    Menstrual type breakdown
    Inactive endometrium
    Atrophic endometrium
Treatment
  • Treatment of significant underlying pathology and if there is none, then observation or (hormonal) therapy to control bleeding, based on severity of symptoms
  • If hormonal therapy is unsuccessful, surgical management (endometrial ablation, hysteroscopic endometrial resection or hysterectomy) is required (PLoS One 2019;14:e0219294)
Microscopic (histologic) description
  • Nondiagnostic sample (no endometrial tissue present)
  • Proliferative endometrium (see Microscopic images at Uterus - Anatomy & histology)
    • Orderly and evenly spaced simple tubular glands, with mitotic activity readily identified in both glands and stroma
    • If few mitotic figures, the prefix weakly can be used
    • Diagnosis of disordered proliferative is made if there is some architectural irregularity but not glandular predominance, i.e., it is not hyperplastic; the distinction between disordered proliferative and proliferative is subject to interobserver variability and does not change management
  • Secretory endometrium (see Microscopic images at Uterus - Anatomy & histology)
    • Variation through luteal phase but absence of mitotic activity in glands and evidence of secretory activity (cytoplasmic vacuoles that are subnuclear or supranuclear or intraluminal glandular secretions)
    • Note that some vacuolization can be seen in proliferative endometrium
    • If there is irregularity in the development of secretory changes or glandular architecture, a designation of irregular secretory endometrium can be used
    • Hysteroscopy can show a polypoid appearance of the endometrium (so called secretory pseudopolyp); this is a normal finding of no clinical consequence
  • Menstrual type breakdown (see Microscopic images at Uterus - Anatomy & histology)
    • Apoptosis of glands and stroma
    • Stromal cells may aggregate into tightly packed balls with condensed chromatin (i.e., blue balls)
    • There have been recommendations to separate endometrial breakdown into that which occurs after ovulation and that which is not postovulatory but this is not of clinical relevance and is often impossible
  • Inactive endometrium
    • Endometrial glands lined by cuboidal or columnar epithelium without evidence of mitotic figures or hormonal effects
  • Atrophic endometrium (see Microscopic images at Uterus - Anatomy & histology)
    • Scant strips of cuboidal epithelium with little or no underlying stroma; this a normal finding after menopause
    • There may be cystic dilation of the atrophic glands (cystic atrophy) but this is usually not appreciated in a biopsy specimen unless it is present within an endometrial polyp
Microscopic (histologic) images

Contributed by C. Blake Gilks, M.D. and @MirunaPopescu13 on Twitter
Nondiagnostic sample Nondiagnostic sample Nondiagnostic sample

Nondiagnostic sample

Proliferative endometrium Proliferative endometrium Proliferative endometrium

Proliferative endometrium


Proliferative endometrium

Proliferative endometrium

Secretory endometrium Secretory endometrium

Secretory endometrium

Secretory endometrium Secretory endometrium

Secretory endometrium

Menstrual endometrium

Menstrual endometrium


Menstrual endometrium

Menstrual endometrium

Inactive endometrium Inactive endometrium

Inactive endometrium

Atrophic endometrium Atrophic endometrium Atrophic endometrium

Atrophic endometrium


Abnormal uterine bleeding Abnormal uterine bleeding Abnormal uterine bleeding Abnormal uterine bleeding

Abnormal uterine bleeding

Sample pathology report
  • Endometrium, biopsy:
    • Atrophic endometrium (see comment)
    • Comment: Negative for hyperplasia or carcinoma (please note that a comment is not needed for most endometrial biopsy specimens).
Differential diagnosis
  • Endometrial polyp:
    • Tissue fragments with abnormal glandular architecture and altered stroma (hypercellular or hypocellular and fibrous appearing), thick walled blood vessels
  • Nonatypical hyperplasia:
    • Glandular crowding, with a predominance of glands over stroma, lacking cytological atypia
  • Atypical hyperplasia (EIN):
    • Both glandular crowding and distinct cytological differences in the epithelial cells lining the crowded glands, compared to the adjacent endometrial glandular epithelium (atypia)
  • Carcinoma:
    • Greater degree of glandular crowding and cytological atypia than is seen in atypical hyperplasia
  • Leiomyoma:
    • May not be sampled in a biopsy but if present, the cellularity of the smooth muscle is usually different (more or less cellular) than normal myometrium
  • Stromal tumor:
  • Endometritis:
    • Histological findings vary depending on the infectious etiology
    • In chronic endometritis, there are increased numbers of lymphoid aggregates, altered stroma, with more spindled morphology and plasma cells
  • Exogenous hormone effects:
    • Progestins result in hypodeveloped glandular epithelium with widely spaced simple glands, stromal predominance and decidual-like changes of the stromal cells
Additional references
Board review style question #1
Which statement is true regarding endometrial biopsy for assessment of abnormal uterine bleeding (AUB)?

  1. Can be signed out with a final diagnosis of benign in most cases
  2. Can show a wide range of appearances that reflect the normal cyclical changes of the endometrium
  3. Is rarely indicated
  4. Must contain at least 15 fragments of endometrial tissue to be considered inadequate for diagnosis
  5. Will result in a histopathological diagnosis of abnormal uterine bleeding in most cases
Board review style answer #1
B. Can show a wide range of appearances that reflect the normal cyclical changes of the endometrium. The findings in endometrial biopsies taken for abnormal uterine bleeding can show a wide range of appearances that reflect the cyclical changes in the endometrium in women during their reproductive years; accordingly, the histopathological diagnosis provides a description of the features observed microscopically (e.g., proliferative endometrium) and serves to exclude underlying pathology such as carcinoma. Answer A is incorrect because a simple diagnosis of benign, while true, does not convey sufficient information to guide treatment decisions. Answer C is incorrect because endometrial biopsy can be done safely as an office procedure for investigation of abnormal uterine bleeding and is a common surgical pathology specimen. Answer D is incorrect because an endometrial biopsy should only be considered inadequate for diagnosis if there is no endometrial tissue present. Answer E is incorrect because abnormal uterine bleeding is a clinical and not a histopathological diagnosis.

Comment Here

Reference: Abnormal uterine bleeding
Board review style question #2

What is the most common finding when endometrial biopsy is performed for assessment of abnormal uterine bleeding (AUB) in a postmenopausal woman?

  1. Atrophic endometrium
  2. Atypical hyperplasia
  3. Chronic endometritis
  4. Disordered proliferative endometrium
  5. Endometrial carcinoma
Board review style answer #2
A. Atrophic endometrium. The most common finding in a postmenopausal woman (defined as more than 1 year since last period) undergoing biopsy for assessment of abnormal uterine bleeding is atrophic endometrium. The chief clinical concern is that there is underlying carcinoma or premalignant change that is causing the abnormal uterine bleeding but that is not the case in most patients. Note that endometrial biopsy has a sensitivity of ~90% for the diagnosis of endometrial carcinoma but because most endometrial biopsies are from patients who do not have carcinoma, the negative predictive value is high (> 99%) (Int J Gynecol Pathol 2020;39:19). Answers B, C, D and E are incorrect because they are each encountered much less frequently in practice than atrophic endometrium.

Comment Here

Reference: Abnormal uterine bleeding

Adenomatoid tumor
Definition / general
  • Benign tumor composed of mesothelium and smooth muscle
Essential features
  • Uncommon benign tumor most frequently seen in the genital tract of both sexes
  • In the uterus, they are commonly asymptomatic
    • Typically present as subserosal or intramural small, solitary nodule
    • Usually an incidental finding in resection specimen for unrelated disease (leiomyoma, adenomyosis, endometrial cancer)
    • Could raise diagnostic difficulties, as morphology may overlap with various differential diagnoses (see Differential diagnosis) (Adv Anat Pathol 2020;27:394)
ICD coding
  • ICD-O: 9054/0 - adenomatoid tumor, NOS
Epidemiology
  • Reproductive aged women
    • Mean age is 45 years; range of 24 - 72 years
    • 1.2% of hysterectomy specimens (5% of uterine surgical specimens but true incidence may be higher due to misdiagnosis and missed diagnosis) (Biomed Rep 2013;1:352)
    • Multifocal and diffuse in immunosuppressed patients (Int J Gynecol Cancer 2009;19:242)
Sites
Pathophysiology
  • Neoplastic rather than reactive (based on molecular, clinical and pathological features) (Hum Pathol 2016;48:88)
Etiology
Clinical features
Diagnosis
  • By histopathology
Laboratory
  • Routine hematological and biochemical parameters are within normal limits
  • Occasional anemia and low hemoglobin level because of uterine bleeding
Radiology description
  • Magnetic resonance: small solid uterine masses with homogeneous hypointensity on T2 weighted images and lower enhancement or cystic lesions with inner irregular solid nodules (J Comput Assist Tomogr 2015;39:499)
Prognostic factors
Case reports
Treatment
  • Surgical removal (hysterectomy or rarely, simple excision)
  • Excision is adequate treatment
Gross description
  • Solitary, small (range of 0.2 - 3.5 cm; average is 2.1 cm), solid nodular, well circumscribed but nonencapsulated, ill defined margins, gray-white-yellow rubbery cut surface
  • Occasionally grossly inconspicuous
  • Rarely diffuse, multifocal, large or predominantly cystic (J Cancer Res Ther 2015;11:967)
  • Large multicystic tumors have a honeycomb, spongy pattern / cysts filled with serous fluid
  • Exophytic serosal component may rarely be seen
Gross images

Contributed by Ayse Ayhan, M.D., Ph.D.

Adenomatoid tumor and leiomyoma

Microscopic (histologic) description
  • Organized in vascular-like, gland-like, complex slit-like and cystic branching spaces; also tubules or combination of above
  • Less commonly cribriform, solid, anastomosing cords, single cells, focal papillary architecture
  • Tubules may contain basophilic secretions
  • Thread-like bridging strands (attenuated cytoplasm that traverse the pseudovascular spaces) (Ann Diagn Pathol 2003;7:273)
  • No mitoses, no necrosis, no atypia
  • Background: hyperplastic smooth muscle
  • Stroma: edematous, inflammatory infiltrate; sometimes lymphoid follicles; lymphocytes or foamy macrophages can be found in the lumina of the tubules
Microscopic (histologic) images

Contributed by Jian-Jun Wei, M.D. and Ayse Ayhan, M.D., Ph.D.

Vascular-like, cystic branching spaces in a background of benign smooth muscle


AE1 / AE3 positive in AT cells

Alcian blue staining secretions

Calretinin and D2-40

Cytology description
  • Flattened, cuboidal epithelioid cells: scant, pale, eosinophilic cytoplasm (may contain vacuoles) with hairy basophilic apical surface
  • Bland, round nuclei, small nucleoli
  • Occasional signet ring-like cells, which are negative for PAS and mucicarmine and may be positive for acidic Alcian blue and colloidal iron
  • No mitotic activity / no cytological atypia
  • Occasional nests of squamoid cells
Negative stains
Electron microscopy description
  • Abundant long, slender microvilli; desmosomes; complex intercellular canalicular system; single layer basement membrane (Cancer 1970;25:171)
Molecular / cytogenetics description
Sample pathology report
  • Uterus, hysterectomy:
    • Adenomatoid tumor (see comment)
    • Comment: This benign entity is composed of mesothelial cells arranged in vascular-like and cystic branching spaces among a background of smooth muscle. The mesothelial cells are positive for calretinin, which supports the diagnosis above.
Differential diagnosis
  • Adenocarcinoma (including signet ring cell carcinoma):
    • Malignant glandular neoplasm, sometimes with signet ring cells (diagnostic pitfall: mesothelial cells may appear signet ring-like)
    • Cytology may be more pleomorphic in adenocarcinoma but not always
    • Cytokeratins are positive (in adenocarcinoma and adenomatoid tumor), mesothelial markers are negative
  • Hemangioma / lymphangioma / peritoneal inclusion cysts:
    • Presence of vascular / lymphatic elements (which are not present in adenomatoid tumor)
    • CD31 and CD34 positive
  • Leiomyoma:
    • Border with adjacent myometrium may be more defined / bulging compared with adenomatoid tumors
    • Absence of mesothelial lining
    • Cytokeratins are negative, smooth muscle markers (SMA, h-caldesmon) are positive
  • Lipoleiomyoma:
  • Well differentiated liposarcoma:
    • Common sites include deep thigh of lower extremity, retroperitoneum, head and neck area, spermatic cord
    • At least focal cytologic atypia typically present
    • Cytokeratins are negative, MDM2 is positive, CDK4 is positive
  • Mesothelioma (benign versus malignant):
    • M > F
    • Strong association with various industrial / environmental / domestic exposures
    • No single marker significantly sensitive or sufficient for mesothelioma
Additional references
Board review style question #1

A 45 year old woman underwent a hysterectomy for symptomatic fibroids. Multiple nodules are identified grossly. A photomicrograph of one of the nodules is included above. Which IHC stain would be the most appropriate to order and would stain positive in the cells of interest?

  1. Calretinin
  2. GMS
  3. MDM2
  4. p16
  5. SOX10
Board review style answer #1
A. Calretinin. Calretinin is the correct answer as it highlights the mesothelial cells in the adenomatoid tumor. All the other answer choices do not highlight the cells of interest.

Comment Here

Reference: Adenomatoid tumor
Board review style question #2
An adenomatoid tumor is identified in the myomectomy specimen of a 45 year old woman. What is the next best step in management?

  1. 3 cycles of carbo / taxol
  2. External beam radiation
  3. Infertility workup
  4. Nothing, excision is sufficient
  5. Uterine artery embolization
Board review style answer #2
D. Nothing, excision is sufficient. Adenomatoid tumor is a benign, frequently incidentally found tumor with excellent prognosis. There is no evidence that it impairs fertility. Excision is curative and no additional treatment is needed. For these same reasons, all the other choices are incorrect.

Comment Here

Reference: Adenomatoid tumor

Adenomyosis / adenomyoma
Definition / general
  • A nonneoplastic lesion of myometrial tissue characterized by the presence of endometrial glands and stroma within myometrium (Best Pract Res Clin Obstet Gynaecol 2006;20:511)
  • Synonyms: myometrial endometriosis, superficial adenomyosis (1 - 2.5 mm in myometrium), stromal adenomyosis, incomplete adenomyosis, adenomyosis with sparse glands
  • Usually an incidental finding in hysterectomy specimens (Int J Gynecol Pathol 1996;15:217)
  • May be diffuse or focal
  • May be involved by hyperplasia and carcinoma
Terminology
  • Adenomyoma: A circumscribed nodular aggregate of benign endometrial glands surrounded by endometrial stroma, with leiomyomatous smooth muscle bordering the endometrial stromal component
ICD coding
  • ICD-10: N80.0 - endometriosis of uterus
Epidemiology
Sites
  • Frequently in posterior, less commonly in anterior uterine wall
  • Rarely in cornua or by cervical os
Pathophysiology
Etiology
  • Adenomyosis and endometriosis are usually regarded as closely related, but
    • Microscopic appearance, and probably their pathogenesis, are somewhat different
    • They may occur independently of each other
    • Adenomyosis mostly is made up of nonfunctional (basal) endometrium and is frequently connected with the mucosa (vs. endometriosis, composed of functional layers)
    • Adenomyosis may represents a unique form of endometrial diverticulosis
  • Hypothetical mechanisms include (Crum: Diagnostic Gynecologic and Obstetric Pathology, 2nd Edition, 2011)
    • Instillation of endometrium within the myometrium
    • In situ metaplasia of pluripotent stem cells retained in myometrium or
    • Improper partitioning of the endometrium from the myometrium
  • Of note, del(7) (q21.2q31.2), a deletion found in typical leiomyoma, has been found in three cases of adenomyosis, suggesting some pathobiologic overlap between leiomyomata and adenomyosis (Cancer Genet Cytogenet 1995;80:118)
  • Definitive distinction between these explanations requires further study
Clinical features
Diagnosis
  • By histopathologic examination of well oriented hysterectomies
  • Essentially should not be diagnosed in curettings or hysteroscopic material
Radiology description
Prognostic factors
  • Benign; excellent prognosis even if not removed
Case reports
Treatment
Gross description
Gross images

Contributed by Ayse Ayhan, M.D., Ph.D.
Pre and post-fixed uterus wall

Pre and post-fixed uterus wall

Cut surface of uterus

Cut surface of uterus



Images hosted on other servers:
Missing Image

Thickened and spongy myometrium

Microscopic (histologic) description
Microscopic (histologic) images

Contributed by Ayse Ayhan, M.D., Ph.D.
Depth of penetration

Depth of penetration

Endomyometrial junction is irregular

Endomyometrial junction is irregular

Glandular tissue usually inactive

Glandular tissue usually inactive

Stroma poor and gland poor

Stroma poor and gland poor

Residual nonneoplastic endometrial glands

Residual nonneoplastic endometrial glands


Adenosarcoma arising in endometriosis

Adenosarcoma arising in endometriosis

Missing Image

Various images

AE1 / AE3 and CD10

AE1 / AE3 and CD10



Images hosted on other servers:
Missing Image

Endometrial glands and stroma

Positive stains
Differential diagnosis

Anatomy & histology
Definition / general
  • Pear shaped organ which serves as physiologic site of implantation for fertilized egg and undergoes changes to support subsequent placental attachment and embryonic / fetal development
Essential features
  • Mucosal component (endometrium) is hormonally responsive and undergoes physiologic and morphologic changes throughout the menstrual cycle
Terminology
  • Anatomic divisions
    • Uterine corpus: main portion of the uterus comprising the upper two - thirds, which houses the endometrial lined cavity
    • Uterine cervix: lower one - third of uterus, which attaches to vaginal canal; see Histology
    • Fundus: domed superior portion of uterus located superior to points of fallopian tube insertion
    • Cornua: lateral portions of uterine corpus; sites of fallopian tube insertion
    • Isthmus (lower uterine segment): inferior portion of corpus, which connects to the cervix
  • Histologic divisions
    • Endometrium: mucosal layer lining the uterine cavity composed of endometrial glands and specialized stroma; blood supplied by spiral arteries
      • Stratum basalis: deep layer of endometrium, which is minimally hormone responsive and serves to replenish the stratum functionalis following menses
      • Stratum functionalis: hormone responsive superficial layer of endometrium; undergoes functional and morphologic changes throughout menstrual cycle; shed during menses
    • Myometrium: structural wall of uterus composed primarily of smooth muscle
    • Serosa: thin, outermost layer of uterus consisting of loose connective tissue and mesothelium
Physiology
  • In infants and children, the endometrium is functionally inactive
  • Menarche:
  • In gestational aged women, the endometrium undergoes hormonally driven changes throughout the menstrual cycle (Endotext: The Normal Menstrual Cycle and the Control of Ovulation [Accessed 11 June 2020])
    • Menstrual phase (days 0 - 5):
      • Estrogen and progestin levels fall in the absence of implantation of a fertilized egg, resulting in breakdown of endometrial stroma
      • Stratum functionalis is shed; spiral arteries constrict to minimize blood loss
    • Proliferative phase (days 6 - 14):
      • Stratum functionalis is regenerated by cells from stratum basalis
      • Primarily driven by increasing estrogen levels
      • Corresponds to follicular phase of cycle in ovary
      • Ends at approximately day 14 when ovulation occurs
    • Secretory phase (days 15 - 28):
      • Stratum functionalis undergoes changes to support implantation in the event of fertilization
      • Glands become convoluted and endometrial cells increase glycogen stores
      • Primarily driven by progestin
      • Corresponds to luteal phase of cycle in ovary
  • Following menopause, the endometrium becomes inactive and may eventually undergo atrophy
  • Pregnancy changes
    • Endometrium retains secretory phenotype and stroma becomes decidualized in response to progestins (detailed in Microscopic description section)
    • Myometrium undergoes mechanoadaptation to allow distension and accommodation of developing fetus (J Cell Mol Med 2008;12:1360)
Diagrams / tables

Contributed by Kyle Devins, M.D.
Average 28 day cycle

Average 28 day cycle

Gross description
  • Pear shaped, smooth external surface
  • Anatomic position within the pelvic cavity between the urinary bladder (anterior) and rectum (posterior)
  • Anterior portion can be identified by a higher peritoneal reflection, due to location of the bladder in vivo (J Clin Pathol 1993;46:388)
  • Hysterectomy specimens are often opened in the coronal plane by inserting a probe into the external cervical os and cutting along the probe to reveal the endometrial cavity
  • Endometrial cavity is shaped like an inverted triangle when viewed in the coronal plane
Gross images

Contributed by Kyle Devins, M.D.
Uterus

Uterus

Bivalved uterus

Bivalved uterus

Microscopic (histologic) description
  • Endometrium (Mod Pathol 2000;13:285)
    • Proliferative phase
      • Cellular blue appearance at low power
      • Round to tubular glands
      • Even, regular spacing between glands
      • Pseudostratified columnar cells in glands
      • Numerous mitotic figures in glands and stroma
    • Interval phase (Day 16)
      • Partially developed subnuclear vacuoles
      • Mitoses present, but not as numerous as in proliferative phase
      • Less than 50% of cells in a gland with continuous and well developed subnuclear vacuoles
    • Secretory phase
      • Relatively pink appearance at low power
      • Convoluted, irregularly shaped glands
      • Single layer of columnar or cuboidal cells in glands
      • Early secretory phase (Day 17 - 19)
        • Day 17: Continuous and well developed subnuclear vacuoles in > 50% of a gland, rare mitoses
        • Day 18: Sub- and supranuclear vacuoles (piano keys) with nuclei in the center of cell
        • Day 19: Nuclei at base of cell, supranuclear vacuoles, start of luminal secretions
      • Mid secretory phase (Day 20 - 22)
        • Day 20: Maximal intraluminal secretions, stromal cells with hyperchromatic nuclei and high N:C ratio
        • Day 21: Increased stromal edema
        • Day 22: Peak stromal edema
      • Late secretory phase (Day 23 - 27)
        • Day 23: Predecidua surrounds spiral arterioles
        • Day 24: Predecidua bridges multiple vessels
        • Day 25: Thin band of predecidua beneath endometrial surface
        • Day 26: Thick band of predecidua beneath surface
        • Day 27: Abundant predecidua expanding downward from endometrial surface, increased number of stromal granulocytes
    • Menstrual phase
      • Endometrial stromal breakdown: dense round aggregates of stromal cells admixed with inflammatory cells and blood
      • Papillary syncytial metaplasia is common, thought to be a reparative response
    • Gestational changes
      • Decidual change: stroma gains abundant eosinophilic cytoplasm, appears polygonal with distinct cell borders
      • Arias-Stella reaction in glandular cells (Am J Case Rep 2012;13:271)
        • Nuclear enlargement and hyperchromasia
        • Abundant eosinophilic vacuolated cytoplasm
        • Hobnail appearance with cells protruding into glandular lumen
    • Atrophy
      • Common in postmenopausal women due to estrogen withdrawal
      • Glands composed of inactive low columnar to cuboidal cells
      • Glands often detached from stroma, forming hairpin structures
      • May have cystic change
      • Cystic atrophy is associated with postmenopausal tamoxifen for breast cancer (Am J Obstet Gynecol 1998;178:1145)
  • Myometrium
    • Short fascicles of smooth muscle
    • Large, thick walled blood vessels
Microscopic (histologic) images

Contributed by Kyle Devins, M.D.
Endometrium Overview

Endometrium Overview

Proliferative endometrium

Proliferative endometrium

Proliferative phase, low power

Proliferative phase, low power

Secretory endometrium

Secretory endometrium

Subnuclear vacuoles

Subnuclear vacuoles

Stromal edema

Stromal edema


Intraluminal secretion

Intraluminal secretion

Late secretory

Late secretory

Spiral arteriole

Spiral arteriole

Secretory endometrium, day 26

Secretory endometrium, day 26

Endometrial stromal breakdown

Endometrial stromal breakdown

Stromal aggregates

Stromal aggregates


Arias-Stella reaction

Arias-Stella reaction

Atrophy

Atrophy

Atrophic gland

Atrophic gland

Cytology description
Cytology images
Positive stains
Videos

Histology of benign cycling endometrium

Board review style question #1

    What phase endometrium is pictured in the image?

  1. Atrophy
  2. Menstrual phase
  3. Proliferative phase
  4. Secretory phase
Board review style answer #1
D. Secretory phase. The endometrial glands are lined by a single layer of low columnar cells with pink cytoplasm and prominent glandular secretions. There is predecidual change in the stroma.

Comment Here

Reference: Anatomy & histology
Board review style question #2

    The pictured endometrial sample was most likely obtained from which of the following patients?

  1. Gestational age woman during menses
  2. Postmenopausal woman
  3. Pregnant woman
  4. Prepubertal female
Board review style answer #2
C. Pregnant woman. The image shows an example of gestational endometrium. There is decidual change in the stroma with abundant eosinophilic cytoplasm and polygonal cells with prominent cell borders. The glandular cells show Arias-Stella reaction.

Comment Here

Reference: Anatomy & histology

Arias-Stella reaction
Definition / general
  • Reactive and benign phenomenon of the endometrium induced by hormonal stimulation and characterized by cytomegaly and nuclear enlargement of endometrial glands; typically associated with intrauterine or extrauterine pregnancies or with gestational trophoblastic disease
Essential features
  • Pseudoneoplastic glandular response of the female genital tract, first described by Javier Arias-Stella in 1954 as atypical endometrial changes associated with the presence of chorionic tissue (AMA Arch Pathol 1954;58:112)
  • Typically found in the endometria of postabortion curettings of intrauterine or extrauterine pregnancies or with gestational trophoblastic disease; however, it can occur anywhere in the epithelium of female genital tract during pregnancy and under hormonal intake
  • Endometrial glands are lined by enlarged or hobnail cells, characterized by abundant eosinophilic, cleared to vacuolated cytoplasm and by hyperchromatic, pleomorphic and smudged nuclei with hobnail appearance; however N:C ratio is maintained (Am J Surg Pathol 2004;28:608)
  • Usually easy to recognize because it predominantly affects young and pregnant patients; however, it may occasionally raise concern for carcinoma if detected in nonpregnant or older patients (Int J Gynecol Pathol 1994;13:241)
  • Rarely can be seen in nonpregnant patients on hormonal therapy (oral contraceptives, ovulation inducing drugs and hormone replacement therapies and phytoestrogens) and more rarely in patients without hormonal treatment (Diagn Cytopathol 2005;32:94)
Terminology
  • Arias-Stella phenomenon, Arias-Stella change, Arias-Stella effect
ICD coding
  • ICD-11: GA1Y - other specified noninflammatory disorders of female genital tract
Epidemiology
  • Relatively common phenomenon
  • Most commonly occurs in reproductive aged women and in the background of pregnancy; however, it may be seen in women of all ages and in any type of hormonal alteration (exogenous and endogenous)
Sites
Pathophysiology
  • Represents a cellular transdifferentiation process: conversion of a differentiated cell type into another cell type (Adv Anat Pathol 2002;9:12)
  • Hormone related atypical endometrial reaction, related to the effects of estrogens and progesterone acting simultaneously (Adv Anat Pathol 2002;9:12)
  • Very rarely, Arias-Stella reaction (ASR) can occur in the absence of pregnancy or hormonal intake, also in foci of endometriosis (J Clin Diagn Res 2016;10:ED03)
Etiology
Clinical features
  • Incidental finding
Diagnosis
Laboratory
  • Increased serum levels of beta human chorionic gonadotropin (βhCG) in gestational cases
Radiology description
  • Ultrasonography can reveal a pregnancy (intrauterine or extrauterine)
Prognostic factors
  • Benign condition that typically regresses postpartum
Case reports
Treatment
  • Benign condition that does not need treatment
Gross description
  • Does not typically form mass lesion
  • May extensively involve pre-existing polyp
Frozen section description
  • Since Arias-Stella can be found anywhere in the female genital tract and outside it in ectopic endometrial glands (e.g., in endometriotic lesions), in view of the presence of giant cells with coarse chromatin and macronucleoli, a false positive diagnosis of malignancy can be made during intraoperative examination if the clinician does not provide information regarding current or recent pregnancy or hormonal treatment (J Clin Diagn Res 2016;10:ED03, AMA Arch Pathol 1954;58:112)
  • ASR can occasionally be mistaken for clear cell carcinoma; differentiation from it is made by the lack of stromal invasion, as well as by the absence of the classic tubular and papillary areas typical of clear cell carcinoma and the lack of mitotic activity (Am J Surg Pathol 2019;43:325)
  • In pregnant patients, diagnosis of malignancy should be made with caution when evaluating frozen sections
Microscopic (histologic) description
  • Endometrial glands lined by large polyhedral cells with abundant eosinophilic cytoplasm with large hyperchromatic, pleomorphic and smudged nuclei
  • Focal or diffuse: can involve portion of a gland, a single gland or a different number of glands
  • Affected glands may have micropapillary or cribriform growth and intraglandular papillary epithelial tufts; occasionally they can be confluent cells protruding into the lumen, with nuclei placed at the bulbous shaped apical portion of the cytoplasm, imparting hobnail appearance
  • Absence of stromal invasion
  • Dr. Arias-Stella classified the histologic variants into 5 types (Adv Anat Pathol 2002;9:12):
    • Minimal atypia: characterized by mild nuclear enlargement
    • Early secretory pattern: marked nuclear enlargement, subnuclear and supranuclear vacuoles
    • Secretory or hypersecretory pattern: enlarged nuclei and intense and diffuse cytoplasmic vacuolization
    • Regenerative, proliferative or nonsecretory pattern: nuclei with a vesicular configuration with glands showing no / minimal secretory activity
    • Monstrous cell pattern: nuclei show giant and bizarre forms with homogenous chromatin and frequent pseudoinclusions
  • Mitotic figures in the ASR are rare and thus, frequent or atypical mitoses should raise concern for adenocarcinoma
Microscopic (histologic) images

Contributed by Ayse Ayhan, M.D., Ph.D.

Enlarged nuclei, sub / supranuclear vacuoles

Hobnail growth pattern

Napsin A positivity

Cytology description
  • Cellular enlargement, to double or many times the normal size
  • Abundant eosinophilic, clear to vacuolated cytoplasm
  • Hypertrophied and hyperchromatic nuclei of ovoid or round shape with irregular nuclear contours and variable chromatin patterns (smudgy, open, even)
  • Normal N:C ratio
  • Mitotic figures in 10 - 15% of cases; rarely can be atypical or numerous (Arch Pathol Lab Med 1981;105:116)
Positive stains
Negative stains
Electron microscopy description
Molecular / cytogenetics description
Sample pathology report
  • Endometrium, biopsy:
    • Gestational endometrium with Arias-Stella reaction
Differential diagnosis
Board review style question #1

The finding in the figure above is found in gestational endometrium. What is the diagnosis?

  1. Arias-Stella reaction
  2. Clear cell carcinoma
  3. Endometrioid carcinoma
  4. Endometritis
Board review style answer #1
A. Arias-Stella reaction

Comment Here

Reference: Arias-Stella reaction
Board review style question #2
Where can the Arias-Stella phenomenon occur?

  1. Anywhere in the female genital tract
  2. In the endometrium only
  3. In the uterine cervix
  4. Only in gestational endometrium
Board review style answer #2
A. Anywhere in the female genital tract

Comment Here

Reference: Arias-Stella reaction

Atypical polypoid adenomyoma
Definition / general
  • Polypoid lesion composed of atypical, architecturally complex endometrial glands within benign myomatous / fibromyomatous stroma (Am J Surg Pathol 2015;39:1148)
Essential features
  • Biphasic neoplasm composed of crowded atypical endometrial glands (with or without squamous morules) in myomatous / fibromyomatous stroma
  • Stroma typically positive for SATB2, in contrast to most examples of myoinvasive carcinoma
  • Risk of recurrence or concurrent or subsequent endometrial endometrioid carcinoma
Terminology
  • Atypical polypoid adenomyofibroma
ICD coding
  • ICD-11: XH7ZB1 - atypical polypoid adenomyoma
Epidemiology
  • Usually diagnosed in women of reproductive age; range of 25 - 73 years (Am J Surg Pathol 1996;20:1)
  • Associated with obesity, infertility, nulliparity
Sites
  • Lower uterine segment > uterine corpus
Pathophysiology
Etiology
  • Unknown
Clinical features
Diagnosis
  • Based on histologic features
Radiology description
  • T2 weighted MRI images: slightly hyperintense polypoid mass mixed with markedly hyperintense foci corresponding to endometrial glands
  • T1 weighted MRI images: isointense with myometrium, occasional hyperintense cystic foci
  • Solid portions, other than cystic foci, show contrast enhancement in the arterial phase and washout or plateau pattern in the late phase
  • Reference: J Comput Assist Tomogr 2015;39:32
Prognostic factors
Case reports
  • 29 year old woman with low grade endometrioid adenocarcinoma arising from atypical polypoid adenomyoma; diagnosed in early pregnancy and managed conservatively (Gynecol Oncol Rep 2019;28:101)
  • 35 year old woman with atypical polypoid adenomyoma treated by hysteroscopy with photodynamic diagnosis using 5-aminolevulinic acid (Photodiagnosis Photodyn Ther 2019;27:295)
  • 36 year old woman with endometrioid adenocarcinoma developing 8 years after conservative management for atypical polypoid adenomyoma (Gynecol Oncol Case Rep 2014;8:21)
  • 45 year old woman with atypical polypoid adenomyoma treated with local resection (Pathologica 2020;112:214)
Treatment
Gross description
Gross images

Images hosted on other servers:
Missing Image

Polypoid mass

Microscopic (histologic) description
  • Biphasic tumor:
    • Endometrioid glands, usually with complex architecture and sometimes with cytologic atypia
    • Glandular component often shows lobulated architecture
    • Benign fibromyomatous stroma, rarely with myxoid change
  • Squamous morular metaplasia is frequently seen and may show central necrosis
  • Well circumscribed or slightly irregular margin
  • Reference: Am J Surg Pathol 2015;39:1148
Microscopic (histologic) images

Contributed by Stephanie L. Skala, M.D.

Lobulated architecture

Crowded glands and muscular stroma

Smooth muscle underlying surface endometrium

Crowded glands and muscle bundles

Squamous morule with central necrosis


Haphazard gland arrangement

Myxoid spindled stroma

Stromal SATB2 positivity

Positive stains
Negative stains
Molecular / cytogenetics description
Sample pathology report
  • Uterus, total hysterectomy:
    • Atypical polypoid adenomyoma
Differential diagnosis
Board review style question #1

The tumor above is seen in endometrial curettings from a 36 year old patient. Which of the following statements is true?

  1. Atypical polypoid adenomyoma can progress to endometrioid carcinoma
  2. Atypical polypoid adenomyoma is associated with TP53 mutations
  3. Comedonecrosis of squamous morules implies poor prognosis
  4. Hysterectomy is always the first line treatment for atypical polypoid adenomyoma
  5. Progestin typically leads to regression of atypical polypoid adenomyoma
Board review style answer #1
A. Atypical polypoid adenomyoma can progress to endometrioid carcinoma

Comment here

Reference: Atypical polypoid adenomyoma
Board review style question #2
Patients with atypical polypoid adenomyoma that are diagnosed on endometrial biopsy are at an increased risk for which of the following?

  1. Carcinosarcoma
  2. Clear cell carcinoma
  3. Endometrioid carcinoma
  4. Endometriosis
  5. Serous carcinoma
Board review style answer #2
C. Endometrioid carcinoma

Comment here

Reference: Atypical polypoid adenomyoma

Carcinosarcoma (MMMT)
Definition / general
  • Biphasic, malignant tumor with high grade epithelial and sarcomatous components
  • Sarcomatous component is derived from the carcinomatous component as a result of metaplasia / transdifferentiation (epithelial to mesenchymal transition)
Essential features
  • Rare, aggressive neoplasm
  • Occurs in postmenopausal women, most frequently in the uterine corpus
  • Biphasic tumor with malignant epithelial (more frequently high grade carcinoma) and sarcomatous (can be homologous and heterologous) components
  • Staged like endometrial carcinomas according to the International Federation of Gynecology and Obstetrics and the American Joint Committee on Cancer staging classifications
Terminology
  • Malignant mixed Müllerian tumor, malignant mesodermal mixed tumor, metaplastic carcinoma
ICD coding
  • ICD-O: 8980/3 - Carcinosarcoma, NOS
Epidemiology
Sites
  • Most frequent in the uterine corpus
  • Can also arise in the cervix, ovaries, fallopian tubes, vagina, peritoneum and extragenital sites
Pathophysiology
  • 4 theories have been proposed (J Clin Pathol 2002;55:321):
    • Collision theory: the sarcoma and carcinoma are two independent neoplasms
    • Combination theory: both components are derived from a single stem cell that undergoes divergent differentiation early in the evolution of the tumor
    • Conversion theory: the sarcomatous element derives from the carcinomatous element during the evolution of the tumor
    • Composition theory: the spindle cell component is a pseudosarcomatous stromal reaction to the carcinoma
  • The Cancer Genome Atlas (TCGA) data supports the conversion and combination theories (Nat Commun 2019;10:4965, Cancer Cell 2017;31:411)
  • Carcinomatous cells convert themselves to sarcomatous cells via epithelial to mesenchymal transition; this is supported by high epithelial to mesenchymal transition gene signature scores and is likely due to epigenetic alterations at microRNA promoters and histone gene mutations and amplifications (Cancer Cell 2017;31:411, Proc Natl Acad Sci U S A 2016;113:12238)
Etiology
  • Almost all are sporadic
  • Tamoxifen use and pelvic radiation therapy have been associated with an increased incidence
  • Other predisposing factors include chronic estrogen exposure, nulliparity, diabetes and obesity
  • References: Gynecol Oncol 2017;144:329, Cancer 1980;45:1625
Clinical features
  • Vaginal bleeding, abdominal mass and pelvic pain are the usual presenting symptoms
  • 10% of patients have distant metastasis at presentation
  • Extrauterine spread in up to 45% of patients at presentation (Ann Oncol 2016;27:1257)
Diagnosis
  • Frequently only rendered after surgical resection
Prognostic factors
  • 5 year survival rate is approximately 30% (stage I - II: 59%; stage III: 25%; stage IV: 9%) (Gynecol Oncol 2010;116:419)
  • This is a much more aggressive tumor than high grade endometrial carcinoma where the 5 year survival rate for stage I disease is over 80% (Am J Surg Pathol 2007;31:979)
  • Stage is the most consistent independent predictor of outcome
  • Presence of heterologous elements is a poor prognostic factor in early stage disease but its significance remains to be determined in advanced stage disease (Am J Surg Pathol 2007;31:1653)
  • Metastatic component frequently of carcinomatous type but can be of sarcomatous component or of combinations of both; carcinoma components tended to spread lymphatically, while sarcoma components tended to spread locoregionally (Ann Oncol 2016;27:1257)
  • Compared with other high grade endometrial carcinomas, lung metastasis are more frequent (Gynecol Oncol 2005;98:274)
Case reports
Treatment
  • Total abdominal hysterectomy and bilateral salpingo-oophorectomy with pelvic lymphadenectomy
  • Radiotherapy or chemotherapy
Gross description
  • Polypoid lesions
  • Fleshy, bulky and friable
  • Hemorrhage and necrosis common
  • May fill uterine cavity and protrude through cervical os
  • Usually extensive myoinvasion and often extends beyond the uterus
  • Reference: Semin Diagn Pathol 2010;27:274
Gross images

Contributed by Joana Ferreira, M.D. and Ana Félix, M.D., Ph.D.

Macroscopic appearance

Polypoid growth



Images hosted on other servers:

Polypoid fleshy mass at fundus of uterus

Infiltrating partially necrotic tumor

Serosal and vaginal invasion

Polypoid tumor with superficial myometrial invasion

Microscopic (histologic) description
  • Biphasic tumor with carcinomatous and sarcomatous elements, both high grade (Int J Gynecol Pathol 1990;9:1)
  • Carcinomatous and sarcomatous components are juxtaposed
  • Angiolymphatic invasion common, more commonly of the carcinomatous component
Microscopic (histologic) images

Contributed by Joana Ferreira, M.D., Ana Félix, M.D., Ph.D. and @ahmsab_MD on Twitter

Large polypoid tumor with minimal myometrial invasion

Atypical cells and chondroid matrix

Necrosis, squamous differentiation

Serous carcinoma

Chondrosarcomatous area


Pleomorphic sarcomatous area

Sarcomatous area

Osteosarcomatous area

Rhabdomyosarcomatous
areas

Carcinosarcoma (MMMT)

Carcinosarcoma (MMMT)

Cytology description
  • Cytologic findings have been described on cervicovaginal smears, endometrial and peritoneal aspirates (Diagn Cytopathol 2019;47:547)
  • Fine needle aspiration can be especially useful documenting recurrent or metastatic disease; demonstration of both sarcomatous and carcinomatous components can be difficult
Positive stains
Molecular / cytogenetics description
  • TCGA endometrial carcinoma molecular subgroups can be applied to carcinosarcomas (Nat Commun 2019;10:4965):
    • Proportions of POLE and MSI subtypes are similar to other types of endometrial carcinoma
    • Copy number high (CNH) are much more common in carcinosarcomas
  • Most share common features with high grade serous ovarian and serous endometrial carcinomas, although a minority has features consistent with an endometrioid subtype
  • TP53 is frequently mutated
  • Mutations in the phosphatidylinositol3-kinase pathway genes (PIK3CA, PTEN, PIK3R1) are also common
  • Other significantly mutated genes include FBXW7, PPP2R1, CDH4, KRAS, ARID1A, ARHGAP35, SPOP, RB1, U2AF1, ZBTB7B (Cancer Cell 2017;31:411)
Sample pathology report
  • Uterus, total hysterectomy and bilateral salpingo-oophorectomy:
    • Uterine carcinosarcoma with a heterologous component (see synoptic report and comment)
    • Comment: There is a malignant biphasic cell proliferation composed of carcinomatous elements: (serous carcinoma) intimately admixed with sarcomatous elements (chondrosarcoma and osteosarcoma). The constellation of morphological features supports the diagnosis of carcinosarcoma (malignant mixed Müllerian tumor). These are considered malignant epithelial tumors and behaves like a high grade carcinoma.
Differential diagnosis
Board review style question #1


A 75 year old woman presents with vaginal bleeding. A hysterectomy is performed (see images above). What is the diagnosis?

  1. Carcinosarcoma with heterologous differentiation
  2. Carcinosarcoma with homologous differentiation
  3. Serous endometrial carcinoma
  4. Rhabdomyosarcoma
Board review style answer #1
A. Carcinosarcoma with heterologous differentiation

Comment Here

Reference: Carcinosarcoma (MMMT)

Clear cell carcinoma
Definition / general
  • Tumor of postmenopausal patients that histologically resembles ovarian clear cell carcinoma with clear, oxyphil or hobnail cells
Essential features
  • Diagnosed based on characteristic morphology
  • Solid architecture more common than papillary
  • HNF-1B, napsin A and AMACR positive
  • Can be any of the 4 molecular subtypes of endometrial carcinoma but most common is no specific molecular profile (p53, mismatch repair and POLE wild type, estrogen receptor negative)
Terminology
  • Also called clear cell adenocarcinoma
ICD coding
  • ICD-O: 8310/3 - clear cell adenocarcinoma, NOS
  • ICD-10: C54.1 - malignant neoplasm of endometrium
Epidemiology
Sites
  • Uterus
Pathophysiology
  • Heterogeneity of molecular pathogenesis (Histopathology 2015;66:664)
  • All 4 molecular subtypes can be seen (p53 abnormal, mismatch repair deficient, POLE mutant and no specific molecular profile) (Histopathology 2015;66:664)
  • p53 abnormal (serous-like) are aggressive, similar to other p53 abnormal endometrial carcinomas (Histopathology 2015;66:664)
  • Mismatch repair deficient often shows mixed morphology, with clear cell and endometrioid components (J Natl Cancer Inst 2016;108:djv427)
  • Only no specific molecular profile (p53, POLE and mismatch repair wild type) fit the classic clinical profile of clear cell carcinoma
Etiology
  • Unknown
Clinical features
Diagnosis
  • Diagnosis is based on characteristic findings on an endometrial biopsy or curettage, which is typically performed for postmenopausal bleeding
  • Abdominal and pelvic imaging can be performed for the purpose of clinical staging
Prognostic factors
Case reports
Treatment
  • Total hysterectomy and bilateral salpingo-oophorectomy
  • Radiation therapy may be considered
Clinical images

Images hosted on other servers:

Endometrial mass

Gross images

Contributed by @BellassaiJb on Twitter
Clear cell carcinoma Clear cell carcinoma

Clear cell carcinoma

Microscopic (histologic) description
  • Diagnosis should be based on the presence of prototypical morphologic (both architectural and cytological) features (Int J Gynecol Pathol 2019;38:S40)
  • Cytological features (Am J Cancer Res 2013;3:70, Int J Gynecol Pathol 2019;38:S40)
    • Polygonal cells with moderate to abundant clear or eosinophilic cytoplasm
    • Hobnail cells and flat cells
    • Occasional enlarged irregular nucleoli
    • Variable cytological atypia
    • Relatively low mitotic index
    • High mitotic index or pleomorphic nuclei does not rule out clear cell carcinoma in an otherwise typical tumor
    • Occasionally targetoid bodies, eosinophilic globules or psammoma bodies
  • Architectural features (Am J Cancer Res 2013;3:70, Int J Gynecol Pathol 2019;38:S40)
    • Solid, glandular or papillary architecture or a combination of these
    • Stromal hyalinization (uncommon)
    • No diffuse nuclear stratification in the papillary areas or diffuse columnar cell changes
Microscopic (histologic) images

Contributed by Jutta Huvila, M.D.

Solid architecture

Glandular architecture

Glandular pattern

Papillary and glandular architecture

Papillary architecture


Hyalinized stroma

Clear cell cytology

Clear and hobnail cells

Oxyphilic cells

Clear and hobnail cells


Targetoid bodies

Psammoma bodies

ER

Napsin A

MSH6

Molecular / cytogenetics description
  • ~50% are related to p53 abnormal, mismatch repair deficient or POLE mutant
  • Remaining cases show occasional KRAS or PIK3CA mutations, without PTEN or TP53 abnormalities (Hum Pathol 2019;92:10)
Sample pathology report
  • Endometrium, biopsy:
    • Clear cell carcinoma of the endometrium (MMR intact, p53 wild type) (see comment)
    • Comment: There is intact expression of mismatch repair proteins (PMS2 and MSH6) and wild type expression of p53, i.e. staining of variable intensity in < 80% of tumor cell nuclei.
Differential diagnosis
Board review style question #1



Which of the following is true about clear cell carcinoma of the endometrium (shown in the image)?

  1. It is associated with a favorable prognosis
  2. It is associated with high levels of estrogen exposure (endogenous or exogenous)
  3. There is molecular heterogeneity
  4. There is no association with Lynch syndrome
Board review style answer #1
C. There is molecular heterogeneity. Clear cell carcinomas of the endometrium can be associated with mutations in POLE (in which case they have a very favorable prognosis), mismatch repair deficiency (which may be a result of Lynch syndrome), mutations in TP53 (poor prognosis) or none of the above.

Comment Here

Reference: Clear cell carcinoma
Board review style question #2
The immunoprofile of most endometrial clear cell carcinomas includes

  1. Immunonegativity for AMACR
  2. Immunopositivity for progesterone receptor
  3. Immunoreactivity for Napsin A
  4. Mutant pattern staining for p53
Board review style answer #2
C. Immunoreactivity for napsin A. The immunophenotype of a majority of endometrial clear cell carcinomas is positivity for napsin A and AMACR, negativity for progesterone receptor and wild type staining pattern for p53.

Comment Here

Reference: Clear cell carcinoma

Disordered proliferative
Definition / general
Essential features
  • Continuum of the spectrum of changes seen with persistent, unopposed estrogen stimulation, which can lead to hyperplasia without atypia
  • Presence of irregularly shaped or cystic dilated glands with relatively normal gland to stroma ratio
Epidemiology
Sites
  • Endometrium
Pathophysiology
  • Unopposed estrogen → disordered proliferative endometrium (early phase) → hyperplasia without atypia (later phase) (Mod Pathol 2000;13:309)
Etiology
Clinical features
  • Asymptomatic or abnormal uterine bleeding
Diagnosis
  • Endometrial biopsy or curettage
Radiology description
  • Ultrasound may show irregularly thickened endometrium
Treatment
Microscopic (histologic) description
  • Cystically dilated glands (> 2x normal size) randomly interspersed among proliferative endometrial glands
  • Dilated glands usually with irregular shape (branched, convoluted, scalloped outer contours)
  • > 10% of overall glands
  • Relatively normal gland to stroma ratio (glands occupy < 50% of the surface area)
  • Metaplastic changes common, including tubal metaplasia, eosinophilic syncytial metaplasia, etc.
  • Stromal hemorrhage and breakdown common
  • Lack of cytologic atypia
Microscopic (histologic) images

Contributed by Hao Chen, M.D., Ph.D.

Irregular dilated glands

Irregular dilated glands with tubal metaplasia

Sample pathology report
  • Endometrium, biopsy:
    • Disordered proliferative endometrium
    • Anovulatory type endometrium
Differential diagnosis
  • Proliferative endometrium:
    • Irregular glands may be present but only focal (< 10%) and small and only mildly dilated
    • Vast majority of glands: round donut or straight tubular shape, lined with tall pseudostratified columnar epithelium; mitotic figures commonly seen
  • Endometrial hyperplasia without atypia:
    • Continuum with disordered proliferative endometrium
    • Irregular dilated glands, more diffusely distributed
    • Gland to stroma ratio > 1 (glands occupy ≥ 50% of the surface area)
  • Endometrial polyp:
    • Often with dilated glands and metaplasia
    • Polypoid
    • Dense fibrotic stroma
    • Thick walled vessels
  • Endometrioid intraepithelial neoplasm (EIN) / atypical hyperplasia (AH):
    • Clonal growth
    • Gland to stroma ratio > 1 (glands occupy ≥ 50% of the surface area)
    • Cytologically distinct from background endometrium
    • Nuclear atypia
    • Often loss of expression of PAX2, PTEN
  • Chronic endometritis:
    • Can result in glandular crowding, abnormal gland shapes and variable degrees of cytologic atypia
    • Presence of stromal plasma cells
    • Presence of stromal spindling and edema
Board review style question #1

Which of the following is true about disordered proliferative endometrium?

  1. Associated with a significantly elevated risk of malignancy
  2. May contain foci of atypia
  3. Most common with women in their 20s
  4. Treatment is with exogenous estrogens
  5. Typically seen in patients with factors leading to unopposed estrogen stimulation (obesity, anovulation)
Board review style answer #1
E. Typically seen in patients with factors leading to unopposed estrogen stimulation (obesity, anovulation)

Comment Here

Reference: Disordered proliferative

Endometrial biopsy reporting (pending)
[Pending]

Endometrial carcinoma-general
Definition / general
  • Malignant epithelial neoplasm originating from the endometrium
Essential features
  • Endometrial cancer is the most common gynecologic malignancy in high income countries
  • General principles in the macroscopic approach, histologic classification, diagnosis and management of endometrial cancer are outlined in the recent recommendations by the International Society of Gynecological Pathologists (Int J Gynecol Pathol 2019;38:S1)
ICD coding
  • ICD-10: C54 - Malignant neoplasm of the corpus uteri
    • C54.0 - Isthmus uteri (lower uterine segment)
    • C54.1 - Endometrium
    • C54.3 - Fundus uteri
    • C54.9 - Corpus uteri, unspecified
Epidemiology
Sites
  • Uterine corpus (fundus, corpus or lower uterine segment)
Pathophysiology
  • From a pathophysiologic perspective, endometrial carcinomas have been traditionally divided into 2 types:
    • Type 1: includes endometrioid and mucinous carcinoma
      • These lesions are associated with long term elevated estrogen levels, which lead to persistent proliferative stimulation of the endometrium
      • Risk factors leading to hyperestrogenism include obesity, exogenous hormonal therapy (e.g. tamoxifen use for breast cancer), ovarian cortical hyperplasia / hyperthecosis, polycystic ovarian syndrome and hormone producing tumors (e.g. granulosa cell tumor of the ovary), among others
      • PTEN, ARID1A, PIK3CA, KRAS gene alterations are common
      • Atypical endometrial hyperplasia / endometrioid intraepithelial neoplasia is regarded as the precursor lesion (J Clin Oncol 2010;28:788, Cancer 2005;103:2304)
    • Type 2: includes serous, clear cell, undifferentiated carcinoma and carcinosarcoma
      • These tumors have a lesser association with unopposed estrogen exposure
      • Serous carcinoma is characterized by early alterations in TP53
      • Precursor lesion for clear cell carcinoma has not been identified
  • Other associations include diabetes, dysfunctional uterine bleeding, hypertension, infertility, Muir-Torre syndrome, Turner syndrome (usually well differentiated adenocarcinoma; 2/3 have squamous differentiation) and tamoxifen use for breast cancer (increased risk for endometrioid, serous carcinoma and carcinosarcoma) (Am J Surg Pathol 2001;25:936, OMIM: Muir-Torre Syndrome [Accessed 1 May 2020])
  • From a biologic and clinical perspective, the classification of endometrial carcinoma is evolving towards a molecular based grouping (see Molecular / cytogenetics description below)
Etiology
Diagrams / tables

Contributed by Carlos Parra-Herran, M.D.

Intraoperative consultation in endometrial cancer

Molecular based classification of endometrial carcinoma

Clinical features
  • Most patients (~90%) present with abnormal uterine bleeding (hypermenorrhea, menometrorrhagia, postmenopausal bleeding)
  • Clear vaginal discharge and constitutional symptoms (weight loss, anemia) can also occur
  • Some patients are diagnosed after an abnormal cervicovaginal cytology result (Pap smear)
    • As Pap smear abnormalities can be the first presenting sign, the presence of endometrial cells in pap smears of women over the age of 45 needs to be reported (Cytojournal 2017;14:22)
    • Likewise, a diagnosis of adenocarcinoma should prompt consideration for endometrial sampling
Diagnosis
  • Ultrasound, pelvic or transvaginal, is often used to identify endometrial thickening or masses occupying the endometrial cavity
  • Hysteroscopy can be performed to better visualize the cavity and perform directed sampling (for example, intact resection of a polypoid lesion)
  • Sampling of the endometrium is the most commonly used test if endometrial cancer is suspected
    • Endometrial biopsy can be done at the gynecologist office by inserting a flexible tube in the canal
    • Endometrial curettage, also performed as an outpatient procedure, is obtained by dilating the cervix to insert a curette to scrape the uterine lining
      • It often results in a more abundant sample compared with endometrial biopsy
Radiology description
  • In principle, any anatomic lesion in premenopausal women (e.g. polyp, nodule, lush irregularity) and any worrisome thickening of the endometrium in postmenopausal women requires sampling
  • Historically, an endometrial thickness of 10 mm or more has been considered the threshold for endometrial cancer in postmenopausal women
Prognostic factors
  • Cancer stage is the strongest predictor of outcome
    • Stage requires pathologic evaluation of resection (hysterectomy) material
    • 2 main staging systems are used: FIGO (2019) and TNM system from the Union for International Cancer Control and the American Joint Committee on Cancer (2018); both are currently harmonized
    • Disease free 5 year survival is > 90% for stage I, ~85% for stage II and ~45% for stage III carcinomas
    • 10 - 30% of patients present at advanced stage (FIGO stage III - IV)
    • Nodal metastases are most common to pelvic and paraaortic nodes; metastases also occur to bone, brain, liver, lung, skin
    • Most recurrences are local (vaginal vault, pelvis)
    • Staging depends on several prognostic histologic variables, some of which are discussed below
    • Locations of tumor spread not addressed in FIGO staging system include pelvic serosa (should be classified as stage IIIA), abdominal serosa (should be stages as FIGO IVB (M1)) and omentum (should be staged as FIGO IVB (M1)) (Int J Gynecol Pathol 2019;38:S93)
  • Myometrial invasion
  • Cervical stromal involvement
  • Serosal involvement
  • Adnexal involvement
  • Lymph node involvement
  • Histologic type
  • Tumor grade
  • Vascular space invasion
  • Molecular group
Case reports
Treatment
  • Total abdominal hysterectomy with bilateral salpingo-oophorectomy (TAHBSO) is standard treatment
  • Regional nodal dissection is performed in patients with endometrioid carcinoma FIGO 2 or 3, high grade histologic type, tumor size > 2 cm or with myometrial invasion on imaging or suspicious lymph nodes
  • Medical treatment with exogenous progestin is a valid option in patients who desire to preserve fertility or are not eligible for surgery, have a FIGO 1 endometrioid carcinoma and no or superficial myometrial invasion on imaging (Obstet Gynecol Int 2010;2010:431950)
  • Radiation therapy is usually indicated in patients with FIGO stage Ib or greater or with local (vaginal, pelvic) recurrence
  • Chemotherapy is indicated for high risk early stage or advanced stage disease, recurrences and distant metastases (Oncologist 2010;15:1026)
Gross description
  • Distinct mass or growth is usually seen upon examination of the cavity
    • Lesion is typically exophytic and soft
    • It can have homogeneous appearance or contain a heterogeneous cut surface with variable hemorrhage and necrosis (more common in high grade tumors)
  • Some cases present as diffuse endometrial thickening; the endometrial tissue is lush, soft and friable
  • Carcinomas with extensive squamous differentiation can have a flaky appearance, whereas those with mucinous differentiation are soft and gelatinous (colloid appearance)
  • Tumor should be sectioned perpendicular to the anterior and posterior uterine walls in order to identify areas of growth into the wall
    • These include myometrial invasion and involvement of adenomyosis, which may be difficult to distinguish grossly
    • Area of deepest growth into the wall should always be sampled as a full thickness section (either a single section or a composite section if the uterine wall is thick)
    • Inclusion of the adjacent uninvolved endometrium in the section is always preferred (if present)
  • If the tumor grossly extends to lower uterine segment, a sagittal section, going from proximal to distal ends to include tumor, lower uterine segment and cervix is recommended
  • References: Lester: Manual of Surgical Pathology, 3rd Edition, 2010, Nucci, Parra-Herran: Gynecologic Pathology, 2nd Edition, 2020
Gross images
Frozen section description
  • Tumor size should be reported routinely and some practices request documentation of tumor size intraoperatively
  • Intraoperative consultation and frozen section evaluation of hysterectomy specimens can also be requested to determine tumor type, grade and depth of myometrial invasion, all of which is taken into consideration to determine the need for lymph node sampling
  • If intraoperative consultation is performed:
    • Specimen should be carefully examined and opened in the coronal plane resulting in anterior and posterior halves (identical to routine processing)
    • If there is no grossly visible lesion, frozen sections are not indicated as sampling is random and identifies malignancy in only 15% of the cases (Am J Clin Pathol 2017;148:345)
    • If a tumor is grossly visible, a representative full thickness section at the point of deepest invasion should be obtained
      • Of note, depth of invasion on frozen section slides has a 36% risk of underestimation and 3% risk of overestimation (Am J Clin Pathol 2017;148:345)
    • Tumor grade has 80% concordance between frozen section and final diagnosis (Am J Clin Pathol 2017;148:345)
    • Proposed algorithm is presented in Diagram 1
Microscopic (histologic) description
  • Pathologic definition of carcinoma of the endometrium:
    • Diagnosis of carcinoma is based on features indicative of invasion into the surrounding mesenchyme (endometrial stroma or myometrium)
      • Stromal invasion is typically seen in the form of glandular confluence and complex architecture: loss of individual glandular contours with gland fusion, lack of intervening stroma and back to back architecture
      • Invasion usually presents in cribriform, microacinar and solid architectural patterns; other helpful features are stromal desmoplasia (stroma has myofibroblasts, edema, inflammatory cells and myxoid change), stromal necrosis (stroma replaced by necrotic and inflammatory debris) or combinations of these findings between adjacent glands
      • So called microcystic, elongated and fragmented (MELF) pattern of growth represents, by definition, invasion
        • This pattern can be deceptive and easily missed at low power magnification
  • Depth of myometrial invasion, cervical stromal involvement, serosal involvement and adnexal involvement are relevant to pT category (see staging)
  • Histologic grading:
    • Endometrioid and mucinous carcinomas are graded with a 3 tier system developed by the International Federation of Gynecology and Obstetrics (FIGO):
      • FIGO 1: predominant glandular growth and < 5% nonsquamous solid component; glandular architecture is identified by the presence of patent lumina within the gland, relatively preserved polarity of the epithelium and absent to mild epithelial stratification
      • FIGO 2: 6 - 50% nonsquamous solid component
      • FIGO 3: > 50% nonsquamous solid component
      • Architectural grading described above is upgraded by 1 if there is severe nuclear atypia (pleomorphism, nuclear enlargement and nucleoli evident at low power magnification)
        • Endometrioid carcinoma FIGO grade 2 purely based on cytologic atypia (that is, with severe atypia but architecturally well differentiated) is extremely rare and must be treated as a diagnosis of exclusion; it is imperative to first exclude serous and clear cell carcinoma
      • In general, a 2 tier system can be also applied, with FIGO1 and FIGO2 being considered low grade and FIGO 3 being considered high grade
    • Other carcinoma types (serous, clear cell, carcinosarcoma, undifferentiated, mixed) are by definition high grade
  • Lymph node involvement:
    • 2018 AJCC staging manual introduces different categories for lymph node metastases in gynecologic malignancies, mostly based on size
  • Lymphovascular space invasion:
    • Lymphovascular space invasion (LVI) is an independent predictor of nodal metastases and local recurrence (Gynecol Oncol 2012;124:31, Arch Gynecol Obstet 2013;288:1391)
    • LVI is defined as tumor cells in a space lined by endothelial cells outside the immediate invasive border
    • Extent of LVI, not just the presence, correlates significantly with regional and distal lymph node involvement, locoregional recurrence and survival (Eur J Cancer 2015;51:1742, Histopathology 2019;75:128)
      • Extent should be reported as follows:
        • Absent: no LVI as defined
        • Focal: a single focus of LVI
        • Substantial: diffuse or multifocal LVSI
    • Under this system, the term focus is understood as a cluster containing up to 5 individual involved vascular spaces
      • It can be inferred that substantial LVI represents either > 1 focus as defined or any focus with > 5 individual involved vascular spaces
    • Artificial tumor intrusion into vascular spaces (vascular pseudoinvasion) can be seen in laparoscopic, robotic assisted hysterectomies (Am J Surg Pathol 2008;32:560, Am J Surg Pathol 2009;33:298)
      • This phenomenon occurs not only in cancer related surgery: displacement of normal endometrial glands and stroma has been reported in 13% of laparoscopic hysterectomies performed for benign conditions (Am J Surg Pathol 2008;32:560)
    • Of note, other studies have shown no association between laparoscopic hysterectomy or the use of a uterine manipulator and the prevalence of LVI (JSLS 2014;18:e2014.00021, Am J Obstet Gynecol 2013;208:71.e1)
    • Real vascular invasion is seen in lymphatics and venous vessels, not in arterial vessels; intravascular foci are round and conform to the shape of the vessel; sometimes they are partially adherent to the vessel wall; cells have more eosinophilic cytoplasm and rounder shape compared with the native tumor; the presence of a perivascular lymphocytic infiltrate also supports real LVI (Mod Pathol 2010;23:1073)
    • Artificial intrusion should be considered if the intravascular tumor retains a gland shape or stromal elements within it, has a large ("chunky") size and involves arterial vessels or other elements are identified within vascular spaces (benign endometrium, surface necrotic material or exudates) (Mod Pathol 2010;23:1073)
Virtual slides
Cytology description
  • Abnormalities in cervicovaginal cytology can be the first presenting sign of endometrial carcinoma
  • Presence of endometrial cells in pap smears of women over the age of 45 needs to be reported (Cytojournal 2017;14:22)
  • Diagnosis of adenocarcinoma in cervicovaginal cytology should prompt consideration for endometrial sampling
  • See endometrial carcinoma types for detailed description
Cytology images
Positive stains
Negative stains
Molecular / cytogenetics description
  • Determination of the tumor histologic type is critical for patient risk stratification and management
  • However, there is poor interobserver reproducibility in tumor type and grade among expert pathologists (Mod Pathol 2013;26:1594, Am J Surg Pathol 2013;37:874, Int J Gynecol Cancer 2011;21:654)
  • Recent studies have provided a comprehensive characterization of the genomic profiles of endometrial carcinoma:
    • In 2013, The Cancer Genome Atlas (TCGA) Research Network published an integrated genomic characterization of endometrial carcinoma based on genomic data from array and sequencing based technologies (Nature 2013;497:67)
    • It proposed a classification that separates endometrial carcinomas in 4 groups:
      • Copy number - high (frequently involving mutations of TP53); this group includes the vast majority of serous carcinomas and 25% of high grade endometrioid tumors
      • Copy number - low (frequently involving mutations of PTEN, PIK3CA, ARID1A and KRAS); this group is mostly composed of low grade endometrioid carcinomas
      • Microsatellite instability hypermutated (frequently involving alterations of mismatch repair protein genes)
      • Polymerase ε (POLE) ultramutated; this group is mostly composed of endometrioid cancers, which despite having a dramatically increased transversion mutation frequency and newly identified hotspot mutations in the POLE gene (which encodes the central catalytic subunit of DNA polymerase epsilon), appear to have a better prognosis than other groups (J Natl Cancer Inst 2014;107:402, Cancer 2015;121:386)
    • Molecular based classification correlates with clinical outcomes: survival rates are best in POLE mutated tumors, followed by copy number - low, microsatellite instability and copy number - high carcinomas (Nature 20132;497:67)
      • Thus, the molecular fingerprint can better assist in patient risk stratification and management
    • Ancillary testing using formalin fixed, paraffin embedded tumoral tissue can serve as a surrogate to detect its molecular alterations and determine the molecular group (J Pathol 2012;228:20)
    • PROMISE algorithm is based on POLE mutational analysis and immunohistochemistry for p53 and mismatch repair proteins (MLH1, MSH2, MSH6 and PMS2) as a valid surrogate to determine the molecular group (Br J Cancer 2015;113:299, Diagram 2)
      • When combined with clinicopathologic features, the molecular classifier is highly correlated with outcome and survival curves
    • Some carcinomas harbor more than one molecular classifying feature and are referred to as multiple classifier; recent evidence suggests that MMR deficiency and POLE mutations supersede p53 alterations in terms of clinical behavior (J Pathol 2020;250:312)
      • MMR deficient, p53 abnormal tumors should be categorized in the MMR deficient / microsatellite instable group
      • POLE mutant, p53 abnormal tumors should be classified in the ultramutated POLE mutant group
      • POLE mutant, MMR deficient should be classified in the ultramutated POLE mutant group (J Pathol 2020;250:323)
    • CTNNB1 mutations have been found to be an adverse prognostic feature in patients with low grade, low risk endometrial carcinoma (Mod Pathol 2017;30:1032)
    • Nuclear expression of beta catenin is usually associated with CTNNB1 mutations, although the correlation is not perfect (Mod Pathol 2018;31:1553)
Sample pathology report
  • Uterus, cervix, fallopian tubes and ovaries (total hysterectomy and bilateral salpingo-oophorectomy):
    • Endometrial carcinoma, ___ type (see synoptic report)
      • Tumor grade: Low versus high / FIGO grade 1 (low) versus 2 (low) versus 3 (high).
      • Myometrial invasion: Absent versus present (< 50% versus ≥ 50% of the myometrial thickness, __/__ mm, __% of the wall).
      • Uterine serosa: Uninvolved / involved.
      • Lymphovascular space invasion: Absent versus present (focal - one focus versus substantial - > 1 focus).
      • Cervical stromal invasion: Absent versus present (1/3 versus 2/3 versus 3/3 of the cervical wall, __/__ mm, __% of the cervical wall).
        • Distal mucosal margin: Negative / positive.
        • Parametrial margin: Negative / positive.
      • Ovaries and fallopian tubes: Uninvolved / involved.
      • POLE mutated MMR deficient versus p53 abnormal versus no specific molecular group.
Differential diagnosis
Board review style question #1
Which of the following prognostic variables modifies the FIGO / AJCC stage of endometrial carcinoma?

  1. Depth of myometrial invasion
  2. Involvement of adenomyosis
  3. Lymphovascular space invasion
  4. Molecular group
  5. Superficial (glandular) cervical involvement
Board review style answer #1
A. Depth of myometrial invasion. All the options have documented prognostic impact in patients with endometrial carcinoma; however, only depth of myometrial invasion modifies the pathologic stage.

Comment Here

Reference: Endometrial carcinoma - general
Board review style question #2
Which of the following variables modifies the FIGO / AJCC stage of endometrial carcinoma?

  1. Extranodal extension by carcinoma involving a lymph node
  2. Lower uterine segment involvement
  3. MELF pattern of invasion
  4. Number of intravascular tumor foci
  5. Size of lymph node tumor metastases
Board review style answer #2
E. Size of lymph node tumor metastases. Of all the included features, only the size of the lymph node alters the stage. Tumor metastasis now subclassifies the N (FIGO stage III) category in N0i+ (isolated tumor cells), N1 / N2mi (micrometastases) and N1 / N2a (macrometastases).

Comment Here

Reference: Endometrial carcinoma - general

Endometrial hyperplasia
Definition / general
  • Proliferation of endometrial glands with a resulting increase in gland to stroma ratio
  • Current system of classification (Kurman: WHO Classification of Tumours of the Female Reproductive Organs, 4th Edition, 2014):
    • Hyperplasia without atypia
    • Atypical hyperplasia / endometrioid intraepithelial neoplasia (AH / EIN)
      • Prior terminologies (simple and complex) are no longer included
  • AH / EIN is considered a premalignant condition
    • Increased risk of both progression to and simultaneous endometrial endometrioid adenocarcinoma
Essential features
  • Estrogen driven precursor lesion to endometrial endometrioid adenocarcinoma
  • Increase in gland to stroma ratio (> 3:1 glandular to stromal elements)
  • Divided into 2 groups: with or without atypia
  • Definitive treatment for AH / EIN is hysterectomy; progestin therapy for fertility preservation
Terminology
  • Obsolete terms:
    • Cystic hyperplasia
    • Adenomatous hyperplasia
    • Simple and complex hyperplasia
ICD coding
  • ICD-10: N85.00 - endometrial hyperplasia, unspecified
  • ICD-O: 8380/2 - endometrioid intraepithelial neoplasia
Epidemiology
Sites
Pathophysiology
  • Increased endogenous or exogenous estrogen, unopposed by progesterone (Semin Oncol Nurs 2019;35:157):
    • Initially, estrogen has mitogenic effect on both endometrial glands and stroma
    • Chronic estrogenic stimulation without progesterone affects glands to a greater extent → glandular overgrowth (hyperplasia)
Etiology
  • Premenopausal
    • Polycystic ovarian syndrome (PCOS): increased circulating androgens peripherally converted into estrogen
    • Chronic anovulation / infertility: dysregulated estrogen without opposing progesterone secretion → simultaneous proliferation and breakdown
  • Peri and postmenopausal
    • Exogenous estrogen:
      • Estrogen supplementation: systemic therapy to alleviate symptoms of menopause → endometrial proliferation
      • Tamoxifen: hormonal treatment for breast cancer acts as estrogen receptor antagonist in breast but agonist in endometrium
  • Any age
    • Obesity: aromatase (enzyme converting circulating androgens to estrogen) is found in adipose tissue → peripheral hyperestrogenism (Mod Pathol 2000;13:295, Am J Obstet Gynecol 2016;214:689.e1)
    • Ovarian pathology:
      • Stromal hyperplasia and hyperthecosis: stromal luteinization → hyperandrogenism → hyperestrogenism (BJOG 2003;110:690)
      • Hormone secreting stromal tumors: granulosa cell tumor, thecoma
Clinical features
Diagnosis
Laboratory
  • No validated biomarker for endometrial hyperplasia
Radiology description
Prognostic factors
  • Endometrial hyperplasia
    • Presence / absence of atypia is most important feature
    • AH / EIN associated with:
    • Hyperplasia without atypia: progression to endometrial endometrioid adenocarcinoma in up to 4.6% of cases after 20 year followup (J Clin Oncol 2010;28:788)
Treatment
  • Endometrial hyperplasia without atypia:
    • Hysterectomy too aggressive; risk of progression to or simultaneous endometrial endometrioid adenocarcinoma is low (refer to Prognostic factors)
    • Treatments outlined below for AH / EIN acceptable within appropriate clinical context
    • Endometrial hyperplasia without atypia arising in endometrial polyp: polypectomy curative if completely excised under hysteroscopic guidance
    • Endometrial ablation can be used (not adequate alternate therapy for AH / EIN or refractory endometrial hyperplasia without atypia) (Am J Obstet Gynecol 1998;179:569)
  • AH / EIN:
    • Hysterectomy with or without bilateral salpingo-oophorectomy is definitive treatment
    • If patient desires fertility or is not a surgical candidate:
Gross description
  • Usually not grossly appreciable
  • Florid to pseudopolypoid endometrium (similar to that of secretory phase)
Gross images

Images hosted on other servers:

Endometrial hyperplasia

Frozen section description
  • Not appropriate for diagnosing hyperplasia or atypia
  • Intraoperative consultation may be utilized for diagnosing adenocarcinoma in a patient with preoperative diagnosis of AH / EIN but this is not considered standard of care
Microscopic (histologic) description
  • Endometrial hyperplasia without atypia
    • Architecture:
      • Closely packed glands such that gland to stroma ratio is > 3:1 but stroma is still present between glandular basement membranes (however minimal)
      • Variation in gland size with cystic dilatation or irregular luminal contours (budding, angulation, invagination, outpouching, papillary projections)
      • Associated with stromal breakdown
      • Increased volume of endometrial tissue on biopsy / curetting is typical but NOT required for diagnosis
    • Cytologic features:
      • Reminiscent of normal proliferative endometrium with pseudostratified, mitotically active, elongated columnar cells
      • Can show mild cellular enlargement but retain smooth nuclear contours without distinct nucleoli
      • Metaplastic changes common (eosinophilic, papillary syncytial, squamous morular, mucinous, ciliated)
  • AH / EIN
    • Architecture:
      • Similar to the spectrum described above for hyperplasia without atypia
    • Cytologic features:
      • Enlarged, rounded and irregular nuclear contours
      • Prominent, enlarged nucleoli with coarse and vesicular chromatin
      • Occasionally, cytoplasmic eosinophilia imparts a distinct low power appearance
      • Stratified cells demonstrating loss of polarity with respect to basement membrane
      • Metaplastic changes can be seen
Microscopic (histologic) images

Contributed by Aarti Sharma, M.D.

Hyperplasia without atypia


AH / EIN


AH / EIN bordering on FIGO grade I endometrial endometrioid adenocarcinoma

Virtual slides

Images hosted on other servers:

AH / EIN

Immunohistochemistry & special stains
  • Not typically useful in differential diagnosis between normal endometrium and benign / malignant endometrial proliferations
  • Loss of PTEN or PAX2 (Int J Gynecol Pathol 2015;34:40, Cancer Res 2010;70:6225)
    • Most frequently mutated genes in endometrioid endometrial carcinoma and its precursors (tumor suppressor and transcription factor inactivation, respectively)
    • Helpful but neither sensitive nor specific for AH / EIN
Molecular / cytogenetics description
Sample pathology report
  • Endometrium, curettage:
    • Disordered proliferative endometrium with focus of hyperplasia without atypia
  • Endometrium, biopsy:
    • AH / EIN focally bordering on endometrial endometrioid adenocarcinoma (FIGO grade I) (see comment)
    • Comment: There are rare minute foci suspicious for a FIGO grade 1 endometrioid endometrial adenocarcinoma. Recommend additional sampling with endometrial curettage for a more definitive diagnosis.
Differential diagnosis
Benign:
  • Compression artifact:
    • Telescoping and pseudocompression of glands due to procedure / processing artifact may create appearance of packed and back to back glands
    • Absence of peripheral stromal elements to lesion in question is a clue to artificial density
  • Cystic atrophy:
    • Can have similar low power appearance to hyperplastic endometrium with closely apposed and cystically dilated glands but these do not have the irregular contours of hyperplasia
    • Glandular lining is low cuboidal to flattened without mitotic activity, in contrast to proliferative endometrium
    • Stroma is dense and resembles that of endometrium basalis
  • Endometrial polyp:
    • Similar low power appearance in biopsies (by definition - altered, disorganized or irregular glands)
    • Distinct densely fibrotic stroma
    • Thick walled blood vessels
    • Endometrial polyps can contain foci of AH / EIN
  • Disordered proliferative endometrium:
    • No well delineated criteria
    • Histologically considered as degree below hyperplasia without atypia on a shared morphologic spectrum and distinction is often not reproducible
    • Both have similar treatment (exogenous progestin)
  • Metaplastic changes:
    • Squamous and morular metaplasia
      • When involving nonhyperplastic glands, can create false appearance of solid crowding
      • As in endometrial endometrioid adenocarcinoma, squamous component should be subtracted in assessment of glandular architecture
    • Surface syncytial and eosinophilic metaplasia
      • Similar low power appearance due to cytoplasmic eosinophilia and epithelial proliferation
      • Metaplasia is usually cytologically bland
  • Endometrial stromal / glandular breakdown:
    • Menstrual endometrium may demonstrate altered cytology, such as loss of polarity due to nuclear piling and coarsening of chromatin
    • Collapse of glands creates artificial crowding without stromal scaffolding
    • Presence of glandular aggregation amidst necrotic predecidua can deceptively mimic carcinoma

Malignant:
  • Endometrioid adenocarcinoma, FIGO grade 1:
    • Degree of atypia between the two is usually similar
    • AH / EIN should NOT have:
      • Cribriforming, confluent glands
      • Labyrinthine intraluminal connections
      • Areas of purely solid epithelium
      • Stromal alteration suggesting invasion - desmoplasia (myofibroblasts, edema, inflammation) or necrosis (intervening endometrial stroma replaced by pools of neutrophilic debris)
Board review style question #1



The uterine lesion in the image above is commonly associated with which of the following?

  1. Anovulatory menstrual cycles
  2. Brown-red and firm, infiltrative gross appearance
  3. Intrauterine device is considered definitive therapy
  4. No increased risk of endometrial carcinoma
  5. Weak staining for WT1 and GATA3
Board review style answer #1
A. Anovulatory menstrual cycles. The photomicrograph shows an image of endometrioid intraepithelial neoplasia.

Comment Here

Reference: Endometrial hyperplasia / endometrioid intraepithelial neoplasia (EIN)
Board review style question #2
Which of the following features is required for a diagnosis of endometrial hyperplasia?

  1. Crowded glands with minimal residual intervening stroma
  2. Diffuse nuclear staining for p53
  3. Documentation of a PTEN mutation or loss of PTEN by IHC
  4. Glands with cribriforming architecture and cytologic alterations distinct from surrounding glands
  5. Loss of mismatch repair proteins
Board review style answer #2
A. Crowded glands with minimal residual intervening stroma

Comment Here

Reference: Endometrial hyperplasia / endometrioid intraepithelial neoplasia (EIN)

Endometrial metaplasia
Definition / general
  • Nonneoplastic cytomorphologic transformation of the epithelial lining of the uterine cavity from its usual endometrial phenotype to other subtypes of Müllerian epithelial differentiation
  • While rare, endometrial stromal transformation to other mesenchymal elements can also occur
Essential features
  • Epithelial metaplasia is replacement of the normal endometrium by a differentiated epithelium
  • Endometrial metaplasia can be associated with hyperestrogenism, inflammation, repeated irritation or endometrial polyps
  • Be sure to rule out a neoplastic process (endometrial hyperplasia or carcinoma)
  • Epithelial metaplasias include squamous, mucinous, tubal, eosinophilic, papillary, secretory and hobnail
  • Stromal metaplasias (while uncommon) include osseous, cartilaginous, myomatous, adipose and synovial-like
Terminology
  • Epithelial cellular change
  • Endometrial cytoplasmic change
Epidemiology
  • Wide age range; most common in perimenopausal to postmenopausal women
Sites
  • Endometrium, including endometrial polyps
Etiology
  • Hyperestrogenism
    • Exogenous hormone use / unopposed estrogens
    • Polycystic ovarian syndrome (PCOS)
    • Other causes of persistent anovulation
    • Ovarian tumors secreting hormones
    • Endometrial hyperplasia or carcinoma
  • Prolonged irritation
    • Mechanical
      • Intrauterine device
      • Previous hysteroscopy and manipulation
        • Curettage
        • Polypectomy
    • Infectious / inflammatory
      • Chronic endometritis
      • Pyometria
    • Physiologic
      • Breakdown
  • No obvious cause in a small percentage of cases
  • Reference: Virchows Arch 2018;472:907
Clinical features
  • Some epithelial metaplasias (e.g. squamous or mucinous) tend to coexist with endometrial hyperplasia or carcinoma; this is not true for other epithelial metaplastic changes, such as tubal metaplasia or for the stromal metaplasias (Histopathology 2008;53:325)
Diagnosis
  • Microscopic examination of endometrial biopsy / curettage or hysterectomy
Radiology description
  • Ultrasound
    • Generally no significant findings
    • If osseous stromal metaplasia → strongly echogenic endometrial plate with posterior shadowing (Clin Imaging 2018;52:260)
Radiology images

Images hosted on other servers:
Ultrasonographic features of osseous metaplasia

Ultrasonographic features of osseous metaplasia

Prognostic factors
Case reports
Frozen section description
  • Metaplasia might be mistaken for a hyperplasia on frozen section if the clinicians send a uterus to evaluate for carcinoma; awareness of this pitfall is important to avoid diagnosing an endometrial neoplasm on frozen section
Microscopic (histologic) description
  • Squamous metaplasia:
    • Subtypes:
      • Morular (Gynecol Endocrinol 2020;36:460):
        • Rounded to polygonal nests of immature squamous cells that fill the glandular lumen
        • Rarely demonstrates keratinization, mitoses and central necrosis
        • Generally cytologically bland but can obscure examination of the glandular architecture
        • Morular borders are not well defined
        • Characteristic intercellular bridges are not identified
        • More commonly associated with endometrioid adenocarcinoma
      • Squamous metaplasia:
        • Intercellular bridges
        • With or without histiocystic or giant cell reaction to the keratin
        • Ichthyosis uteri (Eur J Gynaecol Oncol 2011;32:699)
          • Extensive replacement of surface endometrium with mature, keratinizing, squamous epithelium as a result of chronic inflammation or cervical obstruction
          • Rarely becomes invasive squamous cell carcinoma (Eur J Gynaecol Oncol 2012;33:552)
  • Mucinous metaplasia (Gynecol Oncol 2015;136:389, Diagn Pathol 2017;12:39):
    • Cuboidal to columnar cells, most commonly resembling endocervical epithelium
      • Rarely, resembling gastric and intestinal type metaplasia
      • Generally cytologically bland
      • Subtypes (Mod Pathol 1999;12:1137):
        • Type A: single cell layer lining the epithelial surface, with or without glandular involvement
        • Type B: focal / patchy papillary / micropapillary changes
        • Type C: extensive papillary, cribriform or microacinar architecture
      • Mucinous metaplasia may be found in the ovaries or cervix of patients with endometrial mucinous metaplasia
  • Tubal metaplasia (Mod Pathol 2011;24:1254):
    • Resembling fallopian tube epithelium; pseudostratified with ciliated eosinophilic cells and clear round cells
  • Eosinophilic metaplasia:
    • Cuboidal to columnar cells with prominent eosinophilic cytoplasm, ranging from granular to dense
    • Round nuclei with or without degenerative atypia
  • Papillary metaplasia (Am J Surg Pathol 2013;37:167):
    • Subtypes:
      • Papillary syncytial: pseudostratified eosinophilic cells with indistinct cell borders and loss of nuclear polarity (Int J Gynecol Pathol 2012;31:206)
        • Sometimes termed as pseudopapillary due to the absence of fibrovascular cores
        • Generally located on the surface
        • Associated with stromal breakdown
      • Papillary proliferations: true papillae with fibrovascular cores (Am J Surg Pathol 2001;25:1347)
        • Simple / localized: short, nonbranching
        • Complex / diffuse: numerous branching with papillae and micropapillae
          • Analogous to endometrial intraepithelial neoplasia / atypical hyperplasia
  • Secretory metaplasia:
    • Resembling early to late secretory endometrium, with subnuclear to supranuclear vacuolization
  • Hobnail metaplasia:
    • Glandular, often eosinophilic cells with apically located nuclei that protrude into the lumen
      • Can have cytoplasmic vacuolization
    • Generally cytologically bland
  • Stromal metaplasias:
Microscopic (histologic) images

Contributed by Jessica L. Bentz, M.D.
Squamous morular metaplasia

Squamous morular metaplasia

Tubal and mucinous metaplasia Tubal and mucinous metaplasia

Tubal and mucinous metaplasia

Endometrioid adenocarcinoma with mucinous differentiation

Endometrioid adenocarcinoma with mucinous differentiation

Tubal (ciliated cell) metaplasia

Tubal (ciliated cell) metaplasia


Endometrioid adenocarcinoma with cilia

Endometrioid adenocarcinoma with cilia

Eosinophilic metaplasia in complex atypical hyperplasia Eosinophilic metaplasia in complex atypical hyperplasia

Eosinophilic metaplasia in complex atypical hyperplasia

Papillary syncytial metaplasia

Papillary syncytial metaplasia

Complex papillary metaplasia

Complex papillary metaplasia


Complex papillary metaplasia

Complex papillary metaplasia

Secretory metaplasia

Secretory metaplasia

Hobnail metaplasia

Hobnail metaplasia

Virtual slides

Images hosted on other servers:
Squamous morular metaplasia

Squamous morular metaplasia

Positive stains
Negative stains
Videos

Benign and premalignant lesions of the endometrium - Dr. Parra-Herran, from pathCast

Sample pathology report
  • Endometrium, biopsy:
    • Complex atypical hyperplasia (endometrial intraepithelial neoplasia) with metaplastic changes (see comment)
    • Comment: There are foci with cytologic atypia that exceed the allowable spectrum of atypical changes seen in endometrial metaplasia. Therefore, the findings are most consistent with atypical endometrial hyperplasia.
Differential diagnosis
  • For squamous metaplasia:
    • Squamous cell carcinoma:
      • Cytologic malignancy with invasion
      • Mitoses
      • Show immunophenotype typical of squamous differentiation
      • Diffuse p16 expression may suggest cervical origin of a squamous carcinoma
    • Reactive / reparative
    • Atypical polypoid adenomyoma:
      • Polypoid lesion with prominent myofibroblastic stroma and endometrial intraepithelial neoplasia
  • For mucinous metaplasia:
  • For tubal metaplasia:
  • For eosinophilic metaplasia:
  • For papillary metaplasia:
    • Surface repair
    • Serous carcinoma:
      • Nucleomegaly and marked atypia
      • Immunohistochemistry: p16+ diffuse, aberrant p53 expression (diffuse expression or null phenotype), high Ki67 proliferative index
  • For secretory metaplasia:
    • Clear cell carcinoma:
      • Malignant nuclear features
      • Clinically, may describe a mass lesion
    • Arias-Stella reaction:
      • Pregnancy, trophoblastic disease, progestin therapy
      • Decidualization of the stroma
      • Low nuclear to cytoplasmic ratio without stromal invasion
  • For hobnail metaplasia:
  • For stromal metaplasias:
    • Retained fetal parts
    • Heterologous elements in carcinosarcoma or adenosarcoma
    • If adipose stromal metaplasia → consider uterine perforation
Board review style question #1

A 53 year old woman presents to the gynecologist with chief complaint of vaginal bleeding. An endometrial biopsy is performed and shows fragments of ciliated inactive endometrial tissue with foci of cytologic atypia. Immunohistochemical studies show a wild type pattern of p53 expression, patchy expression of p16 and low Ki67 proliferative index. What is the most likely diagnosis?

  1. Atypical endometrial hyperplasia
  2. Endometrioid adenocarcinoma
  3. Hobnail metaplasia
  4. Serous endometrial intraepithelial carcinoma (SEIC)
  5. Tubal metaplasia
Board review style answer #1
E. Tubal metaplasia

Comment Here

Reference: Endometrial metaplasia
Board review style question #2
What subtype of carcinoma is most commonly associated with squamous metaplasia?

  1. Endometrioid adenocarcinoma
  2. Invasive squamous cell carcinoma of the cervix
  3. Ovarian high grade serous carcinoma
  4. Uterine carcinosarcoma
  5. Uterine serous carcinoma
Board review style answer #2
A. Endometrioid adenocarcinoma

Comment Here

Reference: Endometrial metaplasia

Endometrial polyp
Definition / general
  • Benign hyperplastic overgrowth of endometrial tissue that forms a localized projection into the endometrial cavity and is composed of a variable amount of glands and stroma
  • Polypoid / pedunculated mass composed of cystically dilated glands with fibrous stroma that contains thick walled blood vessels
  • May be related to hyperestrogenism, possibly originating as a localized hyperplasia of the endometrial basalis, secondary to hormonal influences
Essential features
  • Polypoid shape, attenuated surface endometrium on 3 sides
  • Large, thick walled blood vessels
  • Fibrous stroma with spindled fibroblast-like cells, sometimes hyalinized
  • Glandular architectural abnormality different from the surrounding endometrium, with cystic change, sometimes glandular crowding
Terminology
  • Uterine polyp
ICD coding
  • ICD-10: N84.0 - polyp of corpus uteri
Epidemiology
  • Can affect up to 25% of females presenting with abnormal uterine bleeding (Case Rep Obstet Gynecol 2014;2014:518398)
  • Prevalence in asymptomatic females: 10 - 15%
  • Peak incidence in fifth decade
  • Incidence in asymptomatic females with infertility: 10 - 32%
  • Increased incidence with hormone replacement therapy, either estrogen only or combined preparations (Am J Obstet Gynecol 2011;205:535.e1)
Sites
  • Can arise anywhere in the uterine cavity including lower uterine segment
  • Most common in the posterior wall of the uterine cavity, followed by anterior, lateral and uterotubal junction (Fertil Steril 2008;90:180)
  • Can be multiple
  • Rare incidence of endometrial polyps embedded in the fetal membranes have been reported (Int J Gynecol Pathol 2020;39:e13)
Etiology
  • Thought to represent a hyperplastic response of normal endometrial tissue, possibly originating as a localized hyperplasia of the endometrial basalis, secondary to unopposed estrogen
  • Monoclonal stromal overgrowth with secondary induction of polyclonal benign glands
  • Overexpression of endometrial aromatase suggests a role of this enzyme in pathogenesis
  • Associated with tamoxifen therapy (Am J Obstet Gynecol 2011;205:535.e1)
  • Long term consumption of phytoestrogens, produced by plants and found in abundance in spice, herbs and food, can lead to endometrial polyps (Cureus 2021;13:e12789)
  • Late menopause, hormone replacement therapy and obesity all increase risk (Maturitas 2005;50:231)
  • In postmenopausal females, the presence of metabolic syndrome is a predictor (Menopause 2016;23:759)
Clinical features
  • Small polyps are usually asymptomatic
  • Larger polyps are associated with abnormal vaginal bleeding in 13 - 50% of cases and occasionally infertility
  • Rarely, giant polyps may fill the uterine cavity and extend into the endocervical canal
  • Generally unresponsive to progesterone stimulation
  • Polyps associated with tamoxifen therapy for breast cancer are characteristically multiple, large and fibrotic (J Cases Obstet Gynecol 2017;4:55)
  • Most polyps persist if left untreated; however, a small percentage may spontaneously regress (Best Pract Res Clin Obstet Gynaecol 2017;40:89)
Diagnosis
Prognostic factors
Case reports
Treatment
  • Premenopausal: polypectomy for symptomatic polyps, multiple polyps, polyps > 1.5 cm, prolapsed polyps or those associated with infertility
  • Postmenopausal: polypectomy or hysterectomy (Eur J Obstet Gynecol Reprod Biol 2021;260:70)
  • Hysteroscopic removal or morcellation (Gynecol Obstet Fertil 2015;43:104)
  • Excision is curative if the circumscribed foci of endometrial hyperplasia are in polyp with no background hyperplasia
Gross description
Gross images

Contributed by Monira Haque, M.D.

Excised polyp from uterine fundus



Images hosted on other servers:
Missing Image

Fundic polyp expands uterine cavity

Missing Image

Polyp fills the uterine cavity

Missing Image

Partially cystic polyp

Missing Image

Small fundic polyp

Missing Image

Large endometrial polyp

Microscopic (histologic) description
  • Polypoid fragments of endometrial tissue lined by epithelium on 3 sides
  • Surface epithelium:
    • May be atrophic but often is proliferative even in postmenopausal women
    • Papillary proliferations with fibrovascular cores occasionally occur on the surface of an endometrial polyp or within cystically dilated glands
    • May exhibit a degree of atypia, often with degenerated appearing nuclei and sometimes, hobnail cell change
    • Reactive surface changes can be seen, including breakdown (shedding) and hemorrhage
  • Stroma:
    • Fibrous stroma, rich in collagen with abundant extracellular connective tissue
    • Can contain a variable amount of edema, occasional myxoid change and hemosiderin pigment laden macrophages
    • Collections of thick walled blood vessels are common; ectatic thin walled vessels may be seen
    • Bundles of smooth muscle within the stroma may be present, often in close proximity to thick walled blood vessels
    • Foci of decidua may be present (usually reflecting exogenous progestin use or pregnancy)
    • Mitotic figures in the stromal cells may be present
    • Rarely, sex cord-like areas have been described (Int J Gynecol Pathol 2006;25:170)
      • Conventional endometrial, endocervical, or adenomyomatous pedunculated, or sessile lesion with histologic features diagnostic of polyp
  • Glands:
    • Glandular architecture out of phase with the background endometrium
    • Angulated, tubular or cystically dilated
    • Usually endometrioid in type: inactive, proliferative or functional
    • May exhibit metaplastic changes, including ciliated, eosinophilic, mucinous and squamous metaplasia
    • Proliferative activity in a polyp in a postmenopausal woman is of no clinical importance (if present in the nonpolypoid endometrium, it is useful to comment on)
    • Polyps originating at the junction of the upper endocervix / lower uterine segment contain both endocervical and ciliated lower uterine segment type glands
    • If present, the parallel arrangement of the long axis of the endometrial glands to the surface epithelium is a useful diagnostic finding; this feature is especially helpful in premenopausal women (Am J Surg Pathol 2004;28:1057)
  • Tamoxifen associated endometrial polyps:
  • Extensively necrotic polyps can be seen, secondary to torsion or if polyps outgrow their blood supply; vascular thrombosis and surface atypia may be seen in such cases
  • Variants:
    • Endometrial polyp with atypical stromal cells:
      • Presence of bizarre / atypical stromal cells in endometrial polyps and nonpolypoid endometrium has been documented in case reports and few series (Am J Surg Pathol 2015;39:116)
      • Stromal cells show markedly atypical symplastic-like nuclei, resembling those seen elsewhere in the female genital tract (e.g., fibroepithelial stromal polyps of the vulva and vagina) (Int J Surg Pathol 2016;24:320, Am J Surg Pathol 2002;26:505)
      • Mitotic figures are absent (useful feature to differentiate from adenosarcoma)
      • No prominent nucleoli
      • Chromatin distribution is uniform with a smudged appearance (instead of coarse)
      • Multinucleate forms may be seen
      • Atypical cells are usually focally scattered throughout the polyp, predominantly beneath the surface epithelium or at the base
      • Immunohistochemically, the atypical cells express vimentin, estrogen, progesterone and androgen receptors and may express desmin, CD10 and muscle specific actin (Am J Surg Pathol 2002;26:505)
    • Atrophic: usually seen in postmenopausal women; low columnar to cuboidal epithelial lining, cystic dilatation of the glands and fibrotic stroma
    • Functional: if secretory features are present, they are underdeveloped features compared with the background endometrium
    • Myomatous: abundant smooth muscle in the stroma (versus adenomyoma: see Differential diagnosis) (Int J Gynecol Pathol 2000;19:195)
    • Mixed polyps or polyp of mixed endometrial endocervical type
  • Other rare findings include:
  • Must be carefully examined for foci of endometrial hyperplasia and carcinoma:
    • Diagnosis of nonatypical hyperplasia should not be made in a polyp, since proliferative activity with glandular dilatation is a feature
    • Atypical hyperplasia:
      • Diagnostic features similar to those in nonpolypoid endometrium
      • If these are confined to the polyp, it should be specified in the diagnosis (may not require additional therapy)
      • However, in women with atypical hyperplasia in polyps, atypical hyperplasia and carcinoma in the background surrounding endometrium is not uncommon (it is useful to comment on if present in the nonpolypoid endometrium) (Int J Gynecol Pathol 2008;27:45, BJOG 2007;114:944)
    • Endometrial carcinoma:
      • Most commonly serous and endometrioid
      • Carcinomas found in polyps (tamoxifen related and unrelated) may be confined to the polyp or be part of a multifocal endometrial hyperplasia
      • Stains may be helpful to diagnose serous carcinoma
  • Recently, it has been reported that most minimal uterine serous carcinomas arise in an endometrial polyp (Hum Pathol 2021;118:1)
Microscopic (histologic) images

Contributed by Monira Haque, M.D. and Yuri Tachibana, M.D.

Polyp with pedicle

Atrophic epithelium

Stromal edema and myxoid changes

Variably cellular stroma and stromal hemorrhage

Marked stromal hyalinization


Dense stroma

Irregular dilated glands and thickened blood vessels

Highly thickened vessels with fewer glands

Thick walled blood vessels

Variable gland morphology

Glands lined by proliferative type endometrium


Cystically dilated glands

Markedly dilated glands

Squamous metaplasia


Glandular disarray and endometrioid adenocarcinoma

Endometrioid adenocarcinoma

Serous carcinoma

Positive stains
Negative stains
Molecular / cytogenetics description
Sample pathology report
  • Endometrium, biopsy:
    • Fragments of endometrial polyp
    • Negative for carcinoma
Differential diagnosis
  • Adenomyoma:
    • Typically not polypoid
    • Contains prominent stromal smooth muscle
    • Distinction is somewhat arbitrary, based on the amount of smooth muscle present
  • Adenosarcoma:
    • Usually, bulky polypoid mass
    • Leaf-like or club-like architecture, broad papillae lined by surface epithelium and intraglandular stromal projections and > 3 mitoses/10 high power fields with overall architecture resembling a phyllodes tumor of the breast
    • Typically, no cystically dilated glands or thick walled blood vessels
    • Stroma is more cellular with increased mitotic activity and a degree of nuclear atypia, especially immediately surrounding the glands
    • With multiple recurrent endometrial polyps, adenosarcoma should be suspected since the morphological features may be subtle
  • Atypical polypoid adenomyoma:
    • Stroma exhibits more extensive smooth muscle differentiation
    • More complex glandular architecture
    • Often shows extensive squamous morular formation
  • Endocervical polyp:
    • Dilated glands are endocervical (mucous) in type
    • Stroma is edematous, inflamed and usually fibrotic
    • Polyps originating at the junction of the upper endocervix and lower uterine segment contain both endocervical and ciliated lower uterine segment type glands (the term polyp of mixed endometrial endocervical type may be used in such cases)
  • Endometrial hyperplasia:
    • Usually not polypoid
    • Usually a diffuse process, involving the entire endometrium
    • May be confused with hyperplasia arising in an endometrial polyp; will see background endometrial hyperplasia and lack of thick walled vessels
  • Endometrial stromal tumor:
    • Uniform oval small cells, enveloped by reticulin fibers
    • Small vessels characteristically encircled by reticulin fibers
    • Does not have dilated glands or thick walled blood vessels
  • Endometritis:
    • Typical morphological features of polyp are absent, such as polypoid shape and thick walled vessels
  • Leiomyoma (intracavitary / submucosal):
    • Smooth muscle with fascicular growth pattern
    • Dilated glands and thick walled vessels are not seen
    • On hysteroscopy, generally white and firm, with surface blood vessels
  • Lower uterine segment endometrium:
    • Polypoid shape and thick walled blood vessels are typically not present
  • Secretory endometrium:
    • Secretory endometrium corresponds to a cyclical endometrium (secretory in type) which can be seen in the background endometrium

  • Endometrial polyp with atypical stromal cells may have to be differentiated from the following:
    • Adenosarcoma:
      • Extent of stromal atypia is greater (usually diffuse or multifocal)
      • Atypia is uniformly severe
      • Stromal hypercellularity with periglandular stromal cuffing and cambium layer underneath the glandular component
      • Leaf-like projections into glandular lumina
      • > 3 mitoses/10 high power fields
    • Trophoblastic cells:
      • Location is important (implantation site nodule or recent implantation site, instead of a polyp)
      • Cells dispersed individually or in loose aggregates, surrounded by fibrinoid material
      • Mononuclear trophoblast cells have polygonal shape, abundant pale eosinophilic cytoplasm and round uniform nuclei
      • Syncytiotrophoblast is characteristically multinucleated
    • Endometrial stromal sarcoma:
      • Expansile proliferation of spindle cells resembling endometrial stroma (usually producing a mass effect, instead of atypical cells in polyps which are usually incidental)
      • Tightly packed uniform spindled cells without significant glandular elements
      • Smaller capillary type vessels distributed evenly
      • Infiltration into myometrium
    • Malignant mixed Müllerian tumor (MMMT):
      • Malignant appearing epithelial component
      • Stromal component shows diffuse severe atypia
      • High mitotic activity in stroma
Board review style question #1

Which chromosomal abnormality is likely to be associated with the histologic findings from endometrial biopsy of a 35 year old woman with abnormal uterine bleeding?

  1. 1p aberrations
  2. 6p21 aberrations
  3. 9q34 aberrations
  4. Loss of 7q
Board review style answer #1
B. 6p21 aberrations. The histology is consistent with endometrial polyp. The most common cytologic abnormality associated with endometrial polyps are aberrations of chromosome 6p21. Aberrations of chromosome 1p, 9q34 and loss of 7q are associated with uterine leiomyoma.

Comment Here

Reference: Endometrial polyp
Board review style question #2
Tamoxifen related endometrial polyps are frequently associated with which of the following?

  1. Epithelial and stromal metaplasia
  2. Hypercellular stroma
  3. Malignant transformation in up to 10%
  4. Unique cytogenetic profile
Board review style answer #2
A. Epithelial and stromal metaplasia. Tamoxifen related endometrial polyps are generally larger, sessile with bizarre stellate shapes and frequent epithelial and stromal metaplasia. They may show stromal fibrosis and periglandular stromal condensation. Malignant transformation can be seen in up to 3% of cases. Interestingly, their cytogenetic profile is similar to noniatrogenic lesions.

Comment Here

Reference: Endometrial polyp

Endometrial stromal nodule
Definition / general
  • Benign tumor composed of cells reminiscent of proliferative phase endometrial stroma with absent or minimal myometrial invasion (< 3 mm and < 3 protrusions) and lacking vascular invasion
Essential features
  • Benign tumor composed of cells reminiscent of proliferative phase endometrial stroma with absent or minimal myometrial invasion (< 3 mm and < 3 protrusions) and lacking vascular invasion
  • Can be cystic, have necrosis and hemorrhage
  • Excellent prognosis if completely excised
ICD coding
  • ICD-10: D26.1 - other benign neoplasm of corpus uteri (endometrial - stromal)
Epidemiology
  • Rare tumor
  • Occurs at any age (mostly in the fifth and sixth decades)
  • Associated with hypoestrogenism, tamoxifen therapy, radiation
Sites
  • Corpus > cervix > ovary
Pathophysiology
  • Unknown
Etiology
  • Unknown
Clinical features
  • Usually asymptomatic / incidental
  • Abnormal vaginal bleeding
  • Pelvic mass
Diagnosis
Radiology description
Prognostic factors
  • Excellent prognosis if completely excised (hysterectomy)
Case reports
Treatment
  • Hysterectomy if fertility is complete or not desired
  • If fertility preservation is desired, conservative excision followed by hysteroscopy to monitor for regrowth
  • Conservative excision may be adequate but usually limits ability to sample margins
Gross description
  • Well circumscribed, nonencapsulated, soft, fleshy yellow, solitary lesion with size ranging from 1 - 12 cm
  • Tumors usually grow in an expansile, noninfiltrating pattern with a smooth margin
  • Rarely, can show some irregularities but no intravascular component
  • Usually located in the endometrial cavity (polypoid mass) but can also be seen in myometrium
  • Can be cystic
  • Can have necrosis and hemorrhage
Gross images

Contributed by Devi Jeyachandran, M.D.

Polypoid uterine tumor



Images hosted on other servers:

Uterine tumor

Frozen section description
  • If sent for frozen, it may be difficult to differentiate with other spindle cell lesions
    • Can be reported as spindle cell proliferation; final diagnosis deferred to permanent
Microscopic (histologic) description
  • Monotonous proliferations of bland endometrial stromal cells
  • Expansive growth pattern (not infiltrating) at the margin
  • Infiltration, if present, should be < 3 mm and < 3 protrusions
  • Usually prominent proliferative type arterioles and can sometimes show hyalinized walls
  • Collagen bands, plaques, infarct-like necrosis, hemorrhage and degenerative changes (cholesterol clefts, myxoid change and histiocytes) can be present
  • Large, thick blood vessels are uncommon; if present, usually are seen at tumor - myometrium interface
  • May have sex cord-like differentiation, epithelioid morphology, rhabdoid features, clear cells, granular cytoplasm, pseudopapillary, glandular element, fat cells, multinucleated giant cells, bizarre cells, myxoid change
  • No angiolymphatic invasion should be present
  • Mitotic activity is usually < 10 per 10 high power fields
  • Note: foci of smooth muscle metaplasia within the tumor should not be interpreted as myometrial invasion at the edge of the tumor
Microscopic (histologic) images

Contributed by Devi Jeyachandran, M.D.

Bland stromal cells

Expansive growth

Proliferative type arterioles

Bland stromal cells

CD10 expression

ER expression


PR expression

WT1 expression

SMA expression

AE1 / AE3 expression

Desmin expression

Positive stains
Negative stains
  • Desmin, h-caldesmon (but can be positive if smooth muscle or sex cord-like differentiation is present)
  • CD34 (rarely positive)
Molecular / cytogenetics description
  • t(7;17) most common JAZF1-SUZ12
  • PHF1 gene rearrangements (especially if sex cord-like areas are present)
Sample pathology report
  • Uterus, cervix, fallopian tubes and ovaries, total hysterectomy and bilateral salpingo-oophorectomy:
    • Endometrial stromal nodule (see comment)
    • Comment: The mass is extensively sampled and the microscopic examination shows bland spindle cell proliferation resembling endometrial stroma with rare mitosis and proliferative type arterioles. Given the morphology and noninfiltrative growth pattern along with strong CD10 positivity, the above diagnosis is rendered.
  • Endometrium, curettage:
    • Spindle cell proliferation, final classification deferred to resection specimen (see comment)
    • Comment: The biopsy shows bland spindle cell proliferation resembling endometrial stroma with rare mitosis and proliferative type arterioles. These features favor a low grade endometrial stromal tumor but distinction between an endometrial stromal nodule versus low grade endometrial stromal sarcoma cannot be made on a limited sampling.
Differential diagnosis
Board review style question #1
The diagnosis of endometrial stromal nodule versus low grade endometrial stromal sarcoma is based on

  1. Depth of invasion (superficial versus deep)
  2. Infiltrative growth pattern and angiolymphatic invasion
  3. Presence and absence of molecular alteration
  4. Size of the tumor
Board review style answer #1
B. Infiltrative growth pattern and angiolymphatic invasion

Comment Here

Reference: Endometrial stromal nodule
Board review style question #2

A 56 year old woman presents with abnormal uterine bleeding and a polypoid mass in the uterus. The tumor is well circumscribed with bland spindled cells and delicate vasculature. No infiltrative pattern, mitoses, atypia, necrosis or angiolymphatic invasion were identified. No myometrial invasion is identified. The tumor cells are strongly and diffusely positive for CD10, ER and SMA and negative for desmin, caldesmon and MelanA. What type of tumor does this likely represent?

  1. Endometrial polyp
  2. Endometrial stromal nodule
  3. Endometrial stromal sarcoma, low grade
  4. Gland poor adenomyosis
  5. Leiomyoma
Board review style answer #2
B. Endometrial stromal nodule

Comment Here

Reference: Endometrial stromal nodule

Endometrioid carcinoma
Definition / general
  • Endometrial endometrioid carcinoma arises in younger women and is considered to be estrogen dependent with a defined precursor lesion
Essential features
  • Estrogen driven carcinoma of the endometrium that has a well defined precursor lesion - atypical hyperplasia / endometrioid intraepithelial neoplasia
  • Back to back glands lacking intervening stroma, usually with mild to moderate but occasionally marked atypia
  • Major prognostic factors are FIGO grade and stage
Terminology
  • Endometrial endometrioid adenocarcinoma refers to a tumor arising from the endometrium, which resembles proliferative type endometrial glands
ICD coding
  • ICD-O: 8380/3 - Endometrioid adenocarcinoma, NOS
  • ICD-10: C54.1 - Malignant neoplasm of endometrium
Epidemiology
Sites
  • Most common site is the uterine corpus - endometrium, endometrial polyps or adenomyosis
  • Primary cervical endometrioid adenocarcinomas are extraordinarily rare and likely develop from cervical endometriosis (Histopathology 2020;76:112)
    • Drop metastasis or contiguous extension from corpus should be ruled out in these cases
  • Technically, any tissue involved by endometriosis
Pathophysiology
  • Increased endogenous or exogenous estrogen, unopposed by progesterone:
    • Initially, estrogen has mitogenic effect on both endometrial glands and stroma
    • Chronic estrogenic stimulation without progesterone affects glands to a greater extent → glandular overgrowth (hyperplasia) → adenocarcinoma
Etiology
  • Premenopausal:
    • Polycystic ovarian syndrome (PCOS, Stein-Leventhal syndrome):
      • Increased circulating androgens peripherally converted into estrogen
      • Anovulatory cycles
    • Chronic anovulation: dysregulated estrogen without opposing progesterone secretion → simultaneous proliferation and breakdown
  • Peri and postmenopausal:
    • Exogenous estrogen:
      • Estrogen supplementation: systemic therapy to alleviate symptoms of menopause → endometrial proliferation
      • Tamoxifen: hormonal treatment for breast cancer acts as estrogen receptor antagonist in breast but agonist in endometrium
  • Any age:
Clinical features
  • Abnormal, dysfunctional or postmenopausal uterine bleeding
  • Pelvic pain or mass / compression effect on adjacent structures
  • Abdominal bloating
  • Dyspareunia, dysuria
  • General stigmata of malignancy, i.e. weight and appetite loss, malaise, fatigue
  • Rare cases asymptomatic
  • Reference: Eur J Cancer 2001;37:64
Diagnosis
  • Pelvic / transvaginal ultrasound
  • Endometrial biopsy
  • Hysteroscopy with endometrial curettage
  • Hysterectomy:
    • Incidental finding in specimens removed for benign pathology (up to 0.7% including other endometrial histotypes) (Int J Gynecol Cancer 2008;18:1065)
    • Observed in 43% of specimens removed for atypical hyperplasia / endometrioid intraepithelial neoplasia (Cancer 2006;106:812)
  • Incidental finding on cervical cytology screening or endocervical curettings
Laboratory
Radiology description
  • Pelvic / transvaginal ultrasound
    • Thickened endometrial stripe with heterogenous echotexture, increased vascularity and ill defined endomyometrial interface
  • MRI
    • Hypointense mass or heterogenous thickening of endometrium
    • Best modality to detect integrity of endomyometrial junction
  • CT abdomen / pelvis
    • Enlarged uterus
    • Hypoattenuating, hypoechoic mass in endometrial cavity
    • Used mostly for staging of advanced disease (i.e. lymph node status and distant metastases)
  • Reference: Indian J Radiol Imaging 2015;25:137
Prognostic factors
  • Prognosis largely dependent on FIGO / TNM stage:
    • Presence and extent of myometrial invasion (< 50% or > 50%)
    • Cervical stromal involvement
    • Serosal / vaginal / adnexal involvement
    • Nodal metastases
  • FIGO grade:
    • Low grade (FIGO grades 1 and 2) have excellent survival compared with high grade (FIGO grade 3) tumors, the prognosis of which is similar to that of endometrial serous carcinoma
  • However, other parameters such as age, tumor size, histologic features (lymphovascular invasion, microcystic elongated and fragmented glands / MELF pattern invasion) and most recently, molecular features (see Molecular / cytogenetics description) are implicated in prognosis (Int J Gynecol Pathol 2019;38:S93)
Case reports
Treatment
  • Primary treatment is surgical (hysterectomy and bilateral salpingo-oophorectomy with staging), unless patient desires fertility
    • Hormonal therapy (progesterone, leuprolide) alone can lead to complete remission in early stage, low grade tumor for women who want to preserve fertility but long term follow up studies not available (J Clin Oncol 2007;25:2798)
  • Adjuvant chemo / radiotherapy largely dependent on postoperative surgical stage and histologic grade but incorporates other factors (lymphovascular invasion, age, tumor size and involvement of lower uterine segment / surface cervical glands)
  • Reference: NCCN: Clinical Practice Guidelines in Oncology (NCCN Guidelines®) [Accessed 7 August 2020]
Gross description
  • Mass arising from endometrial surface with varied appearances / sizes but usually exophytic and friable in texture
  • Tumor / myometrial interface usually vaguely demarcated, which is useful to grossly assess depth of invasion during intraoperative evaluation
  • Occasionally, no grossly appreciable mass, in which case the entire endometrium must be submitted for histologic evaluation (if prior biopsy showed carcinoma / atypical hyperplasia)
  • Reference: Int J Gynecol Pathol 2019;38:S9
Gross images

Contributed by Aarti Sharma, M.D. and Ricardo R. Lastra, M.D.
Fundic mass

Fundic mass

Mass occupying entire endometrium

Mass occupying entire endometrium

Frozen section description
  • Intraoperative consultation is not appropriate for:
    • Diagnosing adenocarcinoma in a patient with a preoperative diagnosis of atypical hyperplasia / endometrioid intraepithelial neoplasia
    • Diagnosing hyperplasia or atypia
  • Section entire endometrium / mass to assess and freeze area of deepest apparent invasion
  • Key information to report to surgeon (influences subsequent lymphadenectomy) (original Mayo Criteria: Am J Obstet Gynecol 2000;182:1506)
    • FIGO grade of tumor (*1 or 2)
    • Largest tumor dimension (*< 2 cm)
    • Whether tumor is 1) *endometrium confined, 2) *< 50% myoinvasive or 3) > 50% myoinvasive
    • *Cases meeting all criteria do not merit pelvic lymphadenectomy
Microscopic (histologic) description
Overall
  • Architecture
    • Key feature is confluent or back to back glands lacking intervening stroma
    • Cribriform or microacinar configurations
    • Complex papillary, micropapillary or villoglandular structures
  • Cytologic features
    • Resembles proliferative type endometrium with varying features / degrees of atypia but cytology must differ from that of surrounding nonneoplastic glands
      • Cellular / nuclear enlargement
      • Nuclear rounding (rather than elongation) with large nucleoli
      • Loss of polarity
      • Cytoplasmic eosinophilia
    • Sharp glandular luminal borders
    • Foamy histiocytes in residual stroma

Myometrial invasion
  • Conventional pattern:
    • Traverses beyond confines of typically irregular endomyometrial junction without intervening rim of benign marker glands or endometrial stroma
      • Rounded, smooth pushing invasive front, or
      • Infiltrative extension of neoplastic glands
    • Stromal response at invasive front variably consists of fibroblastic proliferation, edema and inflammatory cells
      • This response may be absent
    • Ratio of myoinvasion is crucial to staging:
      • Numerator: depth of furthest invasion (endomyometrial junction to deepest focus of invasive glands)
      • Denominator: myometrial thickness (distance from endomyometrial junction to uterine serosa)
  • Notable patterns of invasion
    • Microcystic, ELongated and Fragmented (MELF):
      • Generally associated with low grade (FIGO 1 - 2); associated with higher rate of lymphovascular invasion and lymph node metastases but not overall survival
      • Fragmented microcystic, elongated glands lined by flattened or histiocytoid epithelium, which can lead to depth of invasion underestimation
      • Distinctive fibromyxoid stromal reaction with acute inflammation
    • Adenoma malignum:
      • Diffusely infiltrative carcinomatous glands with irregular contours, invading myometrium in clusters without or with minimal associated stromal response
      • Can lead to depth of invasion underestimation
      • Distinguish from carcinoma involving adenomyosis, which should not be interpreted as invasion
        • Will have nonneoplastic endometrial glands or stroma at periphery and conventional adenomyosis in other areas
  • Myoinvasion: notable caveats
    • Adenomyosis (Int J Gynecol Pathol 2019;38:S93):
      • Important distinction between 1) carcinoma involving adenomyosis and 2) carcinoma involving adenomyosis with invasion from that focus
      • Regarding 1: depth of invasion = distance from endomyometrial junction to deepest point of invasion elsewhere (the nonmyoinvasive carcinoma within the adenomyotic focus is not considered invasion)
      • Regarding 2: depth of invasion = distance from endomyometrial junction to the point of invasion arising in that specific focus of adenomyosis (irrespective of the deep or superficial location of that focus of adenomyosis within the myometrial wall)
    • Polyp:
      • Only invasion into underlying myometrium should be considered in depth of invasion from endomyometrial junction, not invasion into the polyp stroma itself
    • Exophytic tumors:
      • Thickness of exophytic component should not be considered, only invasion from endomyometrial junction
    • Leiomyomas:
      • Invasive carcinomas overlying / extending into a leiomyoma: wall thickness should incorporate (not subtract) the leiomyoma; unless there is a greater percentage of invasion elsewhere

FIGO grading system (based primarily on architecture)
  • Grade 1: 5% or less nonsquamous solid growth pattern
  • Grade 2: 6 - 50% nonsquamous solid growth pattern
  • Grade 3: > 50% nonsquamous solid growth pattern
  • Nuclear atypia exceeding that expected for the architectural grade increases FIGO grade by 1
    • Glandular variant of endometrial serous carcinoma or component thereof, must be excluded

Morphologic variants
  • Altered differentiation / metaplasia:
    • Squamous, morular and mucinous differentiation are characteristically associated to endometrioid type adenocarcinomas; generally not observed in serous, clear cell or other histotypes
    • Squamous or "squamous" morular: usually banal but occasionally cytologically malignant; former can be glycogenated which imparts appearance of clear cytoplasm
    • Mucinous: intracytoplasmic mucin (intraluminal mucin pooling does not qualify)
    • Secretory: sub / supranuclear vacuolization
    • Ciliated / tubal: resembles fallopian tube lining; scattered cells with apical terminal bars and ciliation
  • Papillary type variants:
    • Villoglandular
    • Small nonvillous papillae
    • Micropapillae
  • Microglandular hyperplasia-like: microcystic, microacinar glands with intraluminal neutrophils
  • Spindled: bland spindling of carcinomatous cells merging with epithelioid carcinomatous component
  • Corded and Hyalinized Endometrial Carcinoma (CHEC): linear cords of carcinoma cells molded by an abundant myxohyaline background
  • Mixed endometrial carcinoma:
    • Defined as combination of at least 2 endometrial histologic subtypes (most commonly endometrioid and serous), the minor component of which must constitute at least 5% of tumor volume on resection specimen (WHO 2014)
    • Distinction important as prognosis is similar to that of the higher grade component (i.e. serous, clear cell, neuroendocrine)
  • Dedifferentiated endometrial carcinoma:
    • Abrupt transition from well differentiated (FIGO 1 - 2) to undifferentiated carcinoma
Microscopic (histologic) images

Contributed by Aarti Sharma, M.D. and Ricardo R. Lastra, M.D.
FIGO 1 FIGO 1

FIGO 1

Solid growth Solid growth

Solid growth

Squamous metaplasia Squamous metaplasia

Squamous metaplasia


Morular metaplasia Morular metaplasia

Morular metaplasia

Mucinous metaplasia Mucinous metaplasia

Mucinous metaplasia

MELF pattern of invasion MELF pattern of invasion

MELF pattern of invasion


CHEC variant CHEC variant

CHEC variant

Virtual slides

Images hosted on other servers:

Uterus, endometrioid carcinoma

MLH1

Cytology description
  • Not typically or ideally a diagnosis made on cytologic specimens but can be identified incidentally on standard Papanicolaou smears
  • No histologic criteria to differentiate between well to moderately differentiated tumor in which atypia is usually not prominent and (Obstet Gynecol 1988;71:242, Obstet Gynecol 1990;76:1000):
    • Normal proliferative endometrium
    • Cells exfoliated from endometrial polyp
    • Endometrial hyperplasia
  • Cytologically unremarkable endometrial cells in cervical Pap of a woman ≥ 45 years is considered abnormal and should be reported (Cancer Cytopathol 2015;123:271)
  • Positive pelvic washings or Pap smears have no effect on staging but are considered adverse prognosticators
Positive stains
Negative stains
  • p53 wild type (patchy and weak expression in scattered nuclei)
    • To be considered aberrant (also known as mutation type), p53 must be either 1) diffusely positive in > 80% of nuclei, 2) completely negative (null type mutant pattern) or 3) any amount of unequivocal cytoplasmic staining (blush does not qualify)
    • Caveat: high grade (FIGO 3) tumors can show aberrant expression (Gynecol Oncol 2004;94:449, Am J Surg Pathol 2009;33:1504)
  • p16 mosaic (patchy and weak nuclear / cytoplasmic expression)
  • CK20, CEA, HNF-1B, Napsin A (rarely positive in endometrioid adenocarcinoma with clear cell change) (Am J Surg Pathol 2015;39:1061)
Molecular / cytogenetics description
  • The Cancer Genome Atlas (TCGA, 2013) defined 4 molecular and prognostic subgroups of endometrial carcinoma (endometrioid (EEC) and serous histotypes) (Nature 2013;497:67):
    • POLE mutated (ultramutated): very high mutational burden, microsatellite stability (~10% of EEC)
      • Comprised by a majority of grade 3 EEC
      • Best prognosis of all subgroups (despite high grade histology)
    • Microsatellite instability (MSI) (hypermutated): high mutational burden with aberrant mismatch repair protein (MMR) IHC
    • Copy number low: low mutational burden, low copy number alterations, no alterations in TP53 gene and wild type p53 IHC
      • Comprised by a majority of grade 1 - 2 EEC
      • Intermediate prognosis similar to hypermutated group, correlated with copy number alteration level
    • Copy number high (serous-like): low mutational burden with elevated copy number alteration rate and high frequency of TP53 gene mutations with aberrant p53 IHC
      • Comprised by a majority of serous and a minor proportion of grade 3 endometrioid carcinomas
      • Worst prognosis of all subgroups, correlated with high copy number alteration level
  • Hereditary nonpolyposis colon cancer / Lynch syndrome
    • Universal screening of all endometrial carcinomas for Lynch syndrome (endometrioid and clear cell histotypes, but testing endometrial serous carcinomas is controversial):
      • Endometrial carcinoma (not colonic) is more frequently the presenting neoplasm for female patients with Lynch syndrome
      • MMR IHC (MLH1, PMS2, MSH2, MSH6) validated on both EEC biopsies / curettings and resections (Gynecol Oncol 2018;149:570)
      • Loss of any component of MSH2 / MSH6 complex → likely Lynch syndrome, refer for genetic testing
      • Loss of MLH1 / PMS2 complex → likely sporadically derived → reflex hypermethylation testing of MLH1
        • Hypermethylation of MLH1 negative → likely Lynch syndrome, refer for genetic testing
    • Histologic features of Lynch syndrome associated endometrial carcinoma:
      • Most are of endometrioid histotype and frequently arise in lower uterine segment
      • Associated with tumor infiltrating lymphocytes and peritumoral lymphocytes
      • Association with dedifferentiated / undifferentiated histotypes
Sample pathology report
  • Endometrium, curettings:
    • Endometrial endometrioid adenocarcinoma, FIGO grade 2, with squamous differentiation
    • Background endometrium with extensive atypical hyperplasia / endometrioid intraepithelial neoplasia
  • Uterus and cervix, hysterectomy:
    • Endometrial endometrioid adenocarcinoma, FIGO grade 1, with deep myometrial invasion (> 50%), focal lymphovascular invasion and extension to lower uterine segment (see synoptic report and comment)
    • Comment: Immunohistochemical stains for mismatch repair proteins (with appropriate controls) demonstrate loss of MLH1 and PMS2, with retention of MSH2 and MSH6. Molecular testing for MLH1 promotor hypermethylation has been requested and results will be issued as an addendum.
Differential diagnosis
Benign
Malignant / premalignant
  • Atypical hyperplasia / endometrioid intraepithelial neoplasia (AH / EIN):
    • Distinction challenging on small biopsies
    • Controversial but 1) cribriforming, 2) solid growth, 3) complex labyrinthine architecture should occupy a diameter ≤ 2.1 mm (half of one 4x low power field) (Cancer 1982;49:2547)
  • Atypical polypoid adenomyoma (APA):
    • Distinction especially challenging with and should not be made on biopsy material, where lobular / polypoid architecture of APA can be lost
    • Atypical, angulated endometrial glands with centrally necrotic squamous morules embedded in fibromyomatous stroma are usual features of APA
    • Appearance of characteristic APA glands in muscular stroma can mimic myometrial invasion by well differentiated EEC
  • Endometrial serous carcinoma (ESC):
    • Both can have papillary architecture
    • Prominent pleomorphism and mitotic activity
    • Arises in background of atrophy and often within endometrial polyps
    • Apical borders are hobnailed and exfoliative, i.e. they appear to shed cells
    • Positive for both p16 (diffuse, block-like) and p53 (aberrant patterns, see Negative stains)
    • Mimics:
      • EEC with papillary architecture (villoglandular, small nonvillous papillae, micropapillae):
        • Less cellular atypia and mitotic activity
        • Arises in background of hyperplasia rather than atrophy or polyps
        • Sharply delineated apical borders (not exfoliative)
        • Wild type p53 and patchy / focal p16
      • EEC with ciliated / tubal metaplasia:
        • Hyperchromatic metaplastic cells lack prominent mitotic activity
        • Appear atypical as they are cytologically distinct from adjacent epithelium and hence mimic ESC (or its precursor - serous endometrial intraepithelial carcinoma)
        • ESC usually not ciliated
  • Endometrial clear cell carcinoma (CCC):
    • Clear to oxyphilic cytoplasm, uniform but moderate atypia and distinct hobnail appearance often with prominent nucleoli and hyaline globules
    • Papillary cores are often hyalinized
    • HNF-1B+ and Napsin A+ and ER / PR-
    • Mimics:
      • EEC with glycogenated squamous metaplasia:
        • Apart from cytoplasmic clearing, lacks other cellular features of CCC
        • Lacks hyalinized, edematous papillary structures
        • ER / PR+ (but may be variably lost in cells with cytoplasmic clearing); HNF-1B-, typically Napsin A-; glycogenated squamous metaplastic component should be at least focally p63+
      • EEC with secretory features:
        • Resembles progestational endometrium with well differentiated, nonstratified columnar cells and prominent cytoplasmic vacuolization (Cancer 1982;49:1511)
        • Caution - this variant is not necessarily restricted to premenopausal females
        • ER / PR+; HNF-1B-, Napsin A-
  • Endocervical usual type adenocarcinoma (ECA):
    • Prominent apical mitoses and basal apoptotic bodies
    • HPV associated ECA are CEA and p16+, vimentin and ER / PR-
    • Typically mucin decreased / depleted
    • Mimics:
      • EEC with mucinous metaplasia
        • Lacks characteristic luminal (floating) mitoses and basal apoptosis
        • EEC with mucinous metaplasia has qualitatively more intraepithelial mucin than a usual type ECA
        • ER / PR and vimentin+, CEA and p16-
      • Microglandular hyperplasia-like EEC:
  • Uterine carcinosarcoma (CS):
    • Mesenchymal component is unequivocally malignant, with cellular atypia and mitotic activity
    • Sarcomatous component usually negative for cytokeratins, EMA and beta catenin (latter may help distinction from CHEC pattern EEC)
    • Often heterologous sarcomatous differentiation (rhabdo, osteo, chondrosarcoma)
    • Mimics:
      • Corded and Hyalinized Endometrial Carcinoma (CHEC):
        • Lacks prominent sarcomatous atypia and mitotic activity
        • Vague sertoliform or trabeculated growth (not patternless like CS) in abundant hyaline matrix
        • Cells may show nuclear expression of beta catenin (i.e. positive)
      • Spindled variant of EEC:
        • Lacks sarcomatous atypia and mitotic activity
        • Spindled cells merge with epithelioid cells (i.e. no sharp transition)
        • Spindled areas usually CK / EMA+
Board review style question #1

The endometrial lesion in the image above is most likely associated with which of the following ovarian tumors?

  1. Fibroma
  2. Granulosa cell tumor
  3. Immature teratoma
  4. Sertoli-Leydig cell tumor
  5. Yolk sac tumor
Board review style answer #1
B. Granulosa cell tumor

Comment Here

Reference: Endometrioid carcinoma
Board review style question #2
Which of the following immunophenotypes is consistent with a well differentiated endometrial endometrioid adenocarcinoma?

  1. PAX8+, CK7+, CK20+, ER / PR+, wild type p53, patchy / focal p16
  2. PAX8+, CK7+, CK20-, ER / PR+, wild type p53, strong / diffuse p16
  3. PAX8+, CK7+, CK20-, ER / PR-, wild type p53, strong / diffuse p16
  4. PAX8+, CK7+, CK20-, ER / PR+, wild type p53, patchy / focal p16
  5. PAX8+, CK7-, CK20+, ER / PR+, wild type p53, patchy / focal p16
Board review style answer #2
D. PAX8+, CK7+, CK20-, ER / PR+, wild type p53, patchy / focal p16

Comment Here

Reference: Endometrioid carcinoma

Endometritis
Definition / general
  • Inflammatory process involving the endometrium
Essential features
  • Endometrial stromal plasma cells required for the diagnosis of chronic endometritis
  • Chronic endometritis is implicated in infertility and recurrent pregnancy loss
  • Acute endometritis typically represents ascending infection from lower genital tract
  • Xanthogranulomatous endometritis usually associated with cervical stenosis
ICD coding
  • ICD-10:
    • N71.1 - chronic inflammatory disease of uterus
    • N71.9 - inflammatory disease of uterus, unspecified
Epidemiology
  • Typically premenopausal women
Sites
  • Endometrium
Pathophysiology
  • Unknown
Etiology
Clinical features
Diagnosis
  • Microscopic examination is the gold standard
  • Hysteroscopic scoring systems have been proposed (J Minim Invasive Gynecol 2020;27:1127)
    • Features: endometrial hyperemia (focal or diffuse), hemorrhagic spots, dilated endometrial vessels, micropolyps, endometrial polyp
Prognostic factors
Case reports
Treatment
Microscopic (histologic) description
  • Chronic endometritis
  • Acute endometritis
    • Neutrophils infiltrating and destroying endometrial epithelium
    • Neutrophils filling gland lumina
    • With or without microabscess formation
  • Granulomatous endometritis
    • Clusters of epithelioid histiocytes rimmed by lymphocytes
    • Cytopathic effects if viral etiology (herpes simplex virus, cytomegalovirus)
    • Tuberculosis: necrotizing granulomas with multinucleated giant cells, usually in superficial functional endometrium (Am J Obstet Gynecol 2020;223:737.e1)
    • Sarcoidosis: nonnecrotizing granulomatous inflammation, more commonly in myometrium
  • Xanthogranulomatous endometritis
    • Abundant foamy histiocytes, siderophages, neutrophils, plasma cells, lymphocytes
    • With or without fibrosis, calcification
Microscopic (histologic) images

Contributed by Stephanie L. Skala, M.D. and Yuri Tachibana, M.D.

Chronic endometritis
Spindled endometrial stroma

Spindled endometrial stroma

Endometrial stromal plasma cells

Endometrial stromal plasma cells

Eosinophils and plasma cells

Eosinophils and plasma cells

Plasma cells

Plasma cells

CD138

CD138

MUM1 highlights plasma cells

MUM1 highlights plasma cells


Xanthogranulomatous endometritis
Foamy histiocytes and eosinophils

Foamy histiocytes and eosinophils

Lymphoid follicles and histiocytes

Lymphoid follicles and histiocytes

 
Acute endometritis
Prominent intraluminal neutrophils

Prominent intraluminal neutrophils

Positive stains
Sample pathology report
  • Endometrium, biopsy:
    • Chronic endometritis
Differential diagnosis
Board review style question #1

What clinical scenario is most likely associated with the endometrial biopsy findings shown above?

  1. Cervical stenosis
  2. Chlamydia
  3. Tuberculosis
  4. Use of intrauterine device
Board review style answer #1
A. Cervical stenosis

Comment Here

Reference: Endometritis
Board review style question #2
Which of the following conditions is thought to be associated with infertility and recurrent pregnancy loss?

  1. Acute endometritis
  2. Chronic endometritis
  3. Granulomatous endometritis
  4. Xanthogranulomatous endometritis
Board review style answer #2
B. Chronic endometritis

Comment Here

Reference: Endometritis

Exogenous hormones
Definition / general
  • Exogenous hormones taken for various reasons may have an effect on the morphology of endometrium, endometrial stroma and myometrial lesions, such as leiomyomas
Essential features
  • Exogenous hormones taken for various indications may affect women of any age
  • Morphologic changes are secondary to the effect of exogenous hormones on estrogen or progesterone receptors in the endomyometrium
  • Subsequent estrogenic or progestogenic effects are variable, ranging from benign (decidual, secretory, inactive or mixed patterns, polyps, stromal changes) to preneoplastic (endometrial hyperplasia) or neoplastic (endometrial carcinoma) changes
  • Appearance of myometrial lesions, such as leiomyomas, may also be affected
  • Clinical history of recent use of exogenous hormones is critical for accurate morphologic assessment, including endometrial dating
Epidemiology
  • Females of any age
Sites
  • Endometrium, myometrium
Pathophysiology
  • Morphologic changes arise secondary to the effect of exogenous hormones on estrogen or progesterone receptors in the endomyometrium (Expert Rev Clin Pharmacol 2020;13:1103, Menopause 2018;25:1033, Contraception 2015;91:360)
  • Contraceptives:
    • Progestogen only: oral, subdermal implant, intramuscular injection or intrauterine device (IUD) containing a progestin (e.g., levonorgestrel or medroxyprogesterone acetate)
      • Results in thickening of cervical mucus, thinning of endometrium (low and high dose) or inhibition of follicular development and ovulation (high dose)
    • Combined: estrogen (ethinyl estradiol) and progestogen (progestin)
      • Suppresses release of gonadotropins, thereby inhibiting follicular development and preventing ovulation
  • Hormone replacement therapy (HRT):
    • Estrogen only: leads to endometrial proliferation with increased risk for endometrial hyperplasia and carcinoma
    • Combined: progestogens have a protective effect on endometrium by downregulating estrogen receptors and thus limiting response to estrogens
  • Androgens:
    • Danazol: weak androgen and weak progestin effects due to main metabolite, ethisterone
    • Gestrinone: combined androgenic, antiestrogenic and antiprogestogenic effects
    • Tibolone: estrogenic and progestogenic activity with weak androgenic effect
    • Testosterone: androgenic effect on endometrium
  • Selective estrogen receptor modulators (SERM): competitive inhibitors of estrogen binding to estrogen receptors
    • Tamoxifen (first generation): estrogen antagonist (breast) or agonist (endometrium), effect depends on estradiol concentration, dose and duration of use as well as menopausal status
    • Clomiphene (first generation): estrogenic or antiestrogenic activity, competitively binding to hypothalamic estrogen receptors and causing an increase in follicle stimulating hormone (FSH) and luteinizing hormone (LH), which induces ovulation
    • Raloxifene (second generation): estrogen antagonist in uterus
    • Bazedoxifene (third generation): estrogen antagonist in uterus
  • Selective progesterone receptor modulators (SPRM): bind to progesterone receptors with agonist, antagonist or mixed agonist / antagonist activity
  • Gonadotropin releasing hormone (GnRH) agonists: reduce gonadotropin levels which suppresses ovulation and secretion of ovarian hormones causing hypoestrogenism
Etiology
  • Exogenous hormones, including estrogens, progestogens and androgens
  • Hormone-like medications:
    • Selective estrogen receptor modulators: tamoxifen, clomiphene, raloxifene, bazedoxifene (when combined with conjugated estrogen termed tissue selective estrogen complex)
    • Selective progesterone receptor modulators: ulipristal acetate, mifepristone, asoprisnil
    • Gonadotropin releasing hormone agonists: buserelin, goserelin, leuprolide (Lupron), nafarelin
Clinical features
  • Indications for estrogen or progestogen therapy include contraception, irregular vaginal bleeding, infertility, polycystic ovarian syndrome, hirsutism, postmenopausal symptoms, adjuvant treatment for breast and endometrial carcinoma, gender dysphoria and congenital abnormal sexual development
  • Androgens:
    • Danazol: treatment of endometriosis, menorrhagia, symptomatic leiomyomas
    • Gestrinone: treatment of endometriosis
    • Tibolone: menopausal symptoms, endometriosis, prevention of osteoporosis
    • Testosterone: gender affirming hormone therapy for gender transition
  • Selective estrogen receptor modulators:
    • Tamoxifen: treatment of hormone receptor positive breast cancer in premenopausal women, breast cancer prevention in patients with ductal carcinoma in situ
    • Clomiphene: ovulation induction
    • Raloxifen: treatment and prevention of postmenopausal osteoporosis
    • Bazedoxifene: treatment of osteoporosis
  • Selective progesterone receptor modulators:
  • Gonadotropin releasing hormone agonists: used for ovulation induction, treatment of endometriosis, pre-endometrial ablation and preoperative treatment of large leiomyomas
Diagnosis
  • Histologic examination of biopsy or resection specimens
Case reports
  • 32 year old woman with uterine fibroids treated with ulipristal acetate prior to in vitro fertilization (Eur Rev Med Pharmacol Sci 2016;20:202)
  • Woman in her late 30s treated with ulipristal acetate for uterine fibroids, diagnosed with paratubal endometriosis showing a morphology similar to progesterone receptor modulator associated endometrial changes (Pathol Res Pract 2017;213:79)
  • 44 year old woman with posterior reversible encephalopathy syndrome after leuprolide injection for uterine leiomyoma (Obstet Gynecol Sci 2019;62:69)
  • 53 year old woman on tamoxifen with atypical endometrial stromal cells in an endometrial polyp and osteoclastic-like giant cells in leiomyoma (Acta Biomed 2019;90:572)
Microscopic (histologic) description
  • Contraceptives:
    • Progestogen only: 3 morphologic patterns of response, dependent on dose and duration of progestin as well as endogenous estrogen levels, which may be coexistent and overlapping (Expert Rev Clin Pharmacol 2020;13:1103)
      • Decidual (pregnancy-like) pattern:
        • Abundant tissue, frequently polypoid
        • Mostly inactive glands, rarely with marked secretory features (Arias-Stella-like)
        • Decidualized stroma (large cells, abundant cytoplasm, prominent cell borders) with occasional stromal mitoses and lymphocytes
        • Ectatic venules with or without markedly thickened spiral arterioles
        • In advanced cases, thrombosis of venules, breakdown and bleeding
        • Stromal lymphocyte and mononuclear cell infiltrate, especially with levonorgestrel IUD; may mimic chronic endometritis but no plasma cells or gland infiltration
        • IUD also associated with stromal macrophages, reactive surface papillary epithelial pattern, less often stromal edema, hemosiderin, calcification and necrosis
        • Neutrophils may be associated with tissue necrosis in areas of breakdown
        • Rarely, decidual cells with vacuolated cytoplasm and eccentric nuclei resulting in signet ring cell appearance
        • Rare pseudosarcomatous stromal features with nuclear pleomorphism, hyperchromasia and prominent nucleoli
      • Secretory (luteal phase-like) pattern:
        • Scant to moderate tissue
        • Mildly tortuous secretory glands lined by columnar cells with basal nuclei and scant dense eosinophilic intraluminal secretions
        • Predecidual stromal cells with pale cytoplasm, ovoid nuclei and scattered mitoses
        • Ectatic vessels with or without thrombosis
        • Appearance of glands and stroma inconsistent with days of normal menstrual cycle
      • Inactive pattern:
        • Scant tissue
        • Widely spaced, small, inactive tubular glands lined by low columnar cells with scant cytoplasm and scant to absent luminal secretions
        • Plump to spindled stromal cells with scant to moderate cytoplasm and inconspicuous mitotic activity
        • More abundant stroma compared with physiologic postmenopausal atrophy
        • Ectatic vessels
    • Combined:
      • Widely spaced, narrow, straight glands lined by cuboidal epithelial cells lacking nuclear pseudostratification or mitotic activity
      • Sparsely cellular pseudodecidualized stroma
      • Thin walled, ectatic stromal vessels, sometimes with thrombi
    • Apoplectic leiomyomas (Am J Surg Pathol 2016;40:563):
      • Related to progestational effect
      • Multiple ovoid to stellate zones with central hemorrhage, necrosis, hyalinization or cystic change and hypercellular periphery
      • Intracellular and extracellular edema of hydropic or alveolar type
      • Spindled and rounded cells in hypercellular areas with eosinophilic cytoplasm and often nuclear pyknosis
      • No to mild cytological atypia
      • Increased mitotic activity may be found in hypercellular zone (0 – 14 mitoses/10 high power fields)
  • Hormone replacement therapy:
    • Estrogen only: weakly to markedly proliferative endometrium, stromal breakdown, squamous morules
    • Progestogen only: scant literature, secretory pattern (mildly tortuous glands with basal nuclei and scant luminal secretions) not always present
    • Combined:
      • Sequential:
        • Weakly proliferative (small tubular glands with occasional mitoses)
        • Poorly developed secretory pattern during progestin administration phase
        • Scant stroma
        • Polyps
      • Combined:
        • Mostly inactive or atrophic endometrium
        • Less commonly secretory pattern
        • Rarely proliferative pattern
  • Androgens:
    • Danazol:
      • Secretory pattern with hypercellular stroma
      • Inactive glands after prolonged therapy
      • Sometimes vascular ectasia, glandular and stromal proliferative activity
    • Tibolone: inactive endometrium
    • Testosterone:
  • Selective estrogen receptor modulators:
    • Tamoxifen:
      • Diffuse endometrial thickening; gross Swiss cheese appearance due to cystically dilated glands
      • Polyps common in postmenopasual women after prolonged treatment
        • Large, multiple, recurrent
        • Staghorn shaped, cystic or hyperplastic glands, polarized along long axis of polyp with various epithelial metaplasias (mucinous, squamous, papillary eosinophilic and clear cell)
        • Stroma myxoid, edematous or fibrous
        • Glandular and stromal proliferative activity common
        • Focal periglandular stromal condensation
      • Endometrial hyperplasia and carcinoma
    • Clomiphene: secretory pattern is subtle and difficult to appreciate
    • Raloxifene: usually inactive endometrium
  • Selective progesterone receptor modulators: progesterone receptor modulator associated endometrial change (Contraception 2015;91:360, Int J Gynecol Pathol 2018;37:575, Int J Gynecol Pathol 2020;39:146)
    • Dyssynchronous endometrium (secretory glands with mitotic activity and apoptosis)
    • Large cystically dilated glands lined by flattened epithelium
    • Edematous or fibroblastic stroma without predecidual change
    • Abnormal blood vessels, including thin walled ectatic, thick walled clusters or anastomosing capillaries
  • Gonadotropin releasing hormone agonists:
    • Endometrial atrophy or secretory pattern if used in conjunction with a progestin
    • Leuprolide (Lupron) related changes in leiomyomas
      • Central geographic areas of edema and hydropic degeneration
      • Subsequent hyaline degeneration
      • Necrosis with surrounding hypercellularity (may vary from infarct type, coagulative or uncertain type)
      • Lymphoid aggregates
Microscopic (histologic) images

Contributed by Sakinah Ahmad Thiryayi, M.D. and Gulisa Turashvili, M.D., Ph.D.

Decidual pattern

Secretory pattern

Inactive pattern

Variable progestin changes


Contraceptive effect

Testosterone for gender transition

Tamoxifen therapy

Tamoxifen therapy


Lupron treated leiomyoma

Necrosis with Lupron

Sample pathology report
  • Endometrium, biopsy:
    • Inactive endometrium with progestational effect (see comment)
    • Comment: The clinical history of Mirena intrauterine device is noted.
  • Uterus, myomectomy:
    • Leiomyomas with degenerative changes (see comment)
    • Comment: The leiomyomas exhibit degenerative changes including large areas of edema, hydropic to hyaline degeneration and necrosis associated with lymphoid aggregates. Most foci of necrosis appear infarct type, although some foci are difficult to differentiate from coagulative necrosis and would be best classified as uncertain type; however, given the recent history of Lupron therapy, these changes are most likely attributed to such therapy.
Differential diagnosis
Board review style question #1

A 45 year old woman, who has been treated with Lupron for longstanding endometriosis and fibroids, underwent total hysterectomy. Most leiomyomas showed morphologic changes depicted in this figure. What additional features would be required to diagnose treatment related degenerative changes?

  1. Marked cytologic atypia and 15 mitoses per 10 high power fields
  2. Marked cytologic atypia and 5 mitoses per 10 high power fields
  3. Mild to absent cytologic atypia and 20 mitoses per 10 high power fields
  4. Mild to absent cytologic atypia and 2 mitoses per 10 high power fields
Board review style answer #1
D. Mild to absent cytologic atypia and 2 mitoses per 10 high power fields

Comment Here

Reference: Exogenous hormones
Board review style question #2

A 32 year old woman has been on a combined oral contraceptive for 2 years. Which of the following statements is true regarding morphologic changes seen in her endometrial biopsy?

  1. Mildly tortuous glands lined by secretory endometrium and pseudodecidualized stroma, consistent with early secretory phase
  2. Straight glands lined by proliferative endometrium and proliferative type endometrial stroma, consistent with early proliferative phase
  3. Tortuous glands lined by secretory endometrium, with intraluminal secretions and pseudodecidualized stroma, consistent with late secretory phase
  4. Widely spaced, narrow, straight glands lined by inactive endometrium and pseudodecidualized stroma, consistent with changes associated with combined oral contraceptives
Board review style answer #2
D. Widely spaced, narrow, straight glands lined by inactive endometrium and pseudodecidualized stroma, consistent with changes associated with combined oral contraceptives

Comment Here

Reference: Exogenous hormones

Grossing & features to report
Procedure (tumor)
  • Measure, weigh, describe, photograph
  • Open along lateral sides
  • Ink serosa adjacent to tumor, vaginal margin of resection
  • Section from endocervical canal to superior aspect
  • If cervical tumor present / suspected, amputate cervix and process as cone biopsy
Sections to obtain
No significant gross pathology:
  • Anterior and posterior cervix (important for identifying carcinoma in situ) (J Clin Pathol 2002;55:629)
  • Anterior and posterior endomyometrium
  • Lower uterine segment
  • Each ovary (cortex, hilar region) and fallopian tube
  • Any gross lesions, including leiomyomas

Suspected tumor:
  • Above, plus:
  • With tumor, full thickness endomyometrium and serosa to assess depth of invasion
  • 1 section per 1 cm of tumor, minimum 3 sections
  • All grossly different appearing regions of tumor
  • High endocervical canal or lower isthmus
  • Ink and obtain margins of resection, including vaginal mucosa

  • Notes: in laparoscopic hysterectomies, pathologists may create artifactual vascular invasion by mechanically transporting tumor into vascular spaces during the grossing process (Am J Surg Pathol 2009;33:298)
  • May be difficult to identify specific cervical anatomic regions in morcellated specimens (Indian J Pathol Microbiol 2010;53:634)
Gross images

Images hosted on other servers:

IARC: various images for grossing protocol

Morcellated specimen:
endometrium
identified as a
slit-like space

Diagrams / tables

Images hosted on other servers:

IARC: various images for grossing protocol

Features to report: recommended sectioning

Features to report

  • Features to report for endometrial carcinoma (J Clin Pathol 2008;61:241):
    • Specimen type, procedure, morcellation or intact
    • Tumor size and location
    • Histologic type
    • Histologic grade
    • Depth of invasion (maximum depth of myoinvasion, measured from endomyometrial junction)
    • Angiolymphatic invasion
    • Cervical involvement
    • Involvement of other organs / tissues
    • Features of uninvolved uterus (hyperplasia, metaplasia, etc.)
    • Margin involvement
    • Nodal involvement (site of nodes, # positive nodes, # total lymph nodes)
    • TNM stage

High grade endometrial stromal sarcoma
Definition / general
  • Malignant uterine mesenchymal tumor originating from the endometrial stroma
Essential features
  • Cellular tumor with high grade round or spindle cells, sometimes with an associated low grade component
  • Variable positivity for BCOR, cyclin D1 and CD10, depending on molecular alteration present; often negative or only focally positive for smooth muscle markers and hormone receptors
  • Major subtypes include sarcomas with YWHAE::NUTM2A/B fusions, BCOR rearranged and BCOR internal tandem duplication (ITD)
Terminology
  • Undifferentiated endometrial sarcoma, uniform type
  • High grade endometrial stromal sarcoma with YWHAE::FAM22 fusions
ICD coding
  • ICD-O: 8930/3 - endometrial stromal sarcoma, high grade
  • ICD-10: C54.1 - malignant neoplasm of endometrium
Epidemiology
Sites
Pathophysiology
Etiology
  • Unknown
Clinical features
  • Pelvic pain, pelvic mass, abnormal uterine bleeding
Diagnosis
  • Hysterectomy needed for definitive diagnosis
  • Rarely diagnosed on endometrial biopsy / curettage
Radiology description
Radiology images

Images hosted on other servers:
MRI findings

MRI findings

Prognostic factors
  • Intermediate prognosis between low grade endometrial stromal sarcoma and undifferentiated uterine sarcoma
  • Limited number of cases reported with limited followup information to identify favorable / unfavorable prognostic factors
Case reports
Treatment
Gross description
Gross images

Contributed by Ondrej Ondič, M.D., Ph.D. and Ayse Ayhan, M.D., Ph.D.
Pale yellow-white, soft and friable masses

Pale yellow-white, soft and friable masses

Missing Image

Necrotic polypoid mass

Frozen section description
  • High grade sarcoma with spindled and epithelioid cells, defer to permanent sections and ancillary (immunohistochemical and molecular) studies
Microscopic (histologic) description
  • YWHAE::NUTM2A/B fusions (Am J Surg Pathol 2012;36:641, Mod Pathol 2013;26:1390, Histopathology 2015;67:1, Pathology 2018;50:162):
    • Permeative (tongue-like) invasion
    • Vaguely nested growth of round cells with scant (small round blue cell appearance) to moderate eosinophilic cytoplasm and uniform nuclear atypia
    • Brisk mitoses (> 10/10 high power fields)
    • Tumor cell necrosis and lymphovascular invasion common
    • Delicate network of arborizing or curvilinear vessels
    • 50% associated with a low grade spindle cell component resembling low grade endometrial stromal sarcoma
      • Admixed or occasionally well demarcated
      • Monomorphic bland spindle cells
      • Fibrous to fibromyxoid matrix
      • Low mitoses (≤ 3/10 high power fields)
      • Typically lacks necrosis
    • Variant features include pseudoglandular or pseudopapillary appearance, sex cord-like differentiation, rosette-like formations
    • Overt nuclear pleomorphism rare
  • BCOR rearranged (Mod Pathol 2018;31:674, Pathology 2018;50:162):
    • Often involves endometrium and myometrium
    • Tongue-like invasion or broad front with infiltrative borders
    • Haphazard fascicles of uniformly atypical spindle cells with scant to moderate and eosinophilic or abundant and blue-gray cytoplasm
    • Extensive myxoid stroma
    • Collagen plaques in ~50%
    • Brisk mitoses (≥ 10/10 high power fields)
    • Small arterioles without perivascular whorling of tumor cells
    • Lymphovascular invasion in ~50%
    • Infarct type necrosis more common than tumor cell necrosis
    • Overt nuclear pleomorphism rare
    • No low grade component present
  • BCOR ITD (Mod Pathol 2017;30:1251, Am J Surg Pathol 2018;42:335, Oncology 2019;96:101, Am J Surg Pathol 2019;43:662):
    • Tongue-like invasion
    • High grade round cell component with uniform atypia, high grade spindle cell component with uniform atypia, low grade fibromyxoid spindle cell component
    • Myxoid matrix
    • Brisk mitoses (≥ 10/10 high power fields)
    • Lymphovascular invasion and tumor cell necrosis
    • Overt nuclear pleomorphism rare
  • High grade transformation of low grade endometrial stromal sarcoma (Mod Pathol 2020;33:1861):
    • High grade transformation may occur in the primary, metastatic or recurrent tumor
    • High grade component usually easily recognized from low grade component on low power magnification but occasionally is a gradual transition
    • High grade component:
      • Comprises 10 - 90% of tumor
      • Sharply demarcated nodules often associated with sclerotic or less commonly myxoid stroma
      • Rounded epithelioid cells with increased atypia (often uniform) and brisk mitoses (median, 16/10 high power fields)
      • Delicate vasculature without perivascular tumor cell whorls
      • Tumor cell necrosis uncommon
    • Low grade component:
      • Conventional low grade endometrial stromal sarcoma
      • May show variant morphology (fibroblastic, smooth muscle differentiation, myxoid matrix, sex cord-like differentiation)
Microscopic (histologic) images

Contributed by Elizabeth Kertowidjojo, M.D., Ph.D., M.P.H., Jennifer A. Bennett, M.D., Ondrej Ondič, M.D., Ph.D. and Ayse Ayhan, M.D., Ph.D.
Mitotically active spindle cells

Mitotically active spindle cells

Myxoid stroma

Myxoid stroma

Collagen plaques

Collagen plaques

Round cell component

Round cell component

Low grade spindle cell component

Low grade spindle cell component


High grade transformation

High grade transformation

Spindle cell morphology

Spindle cell morphology

High grade component

High grade component

Signet ring-like change

Myxoid stroma and signet ring-like change


Tongue-like infiltration

Tongue-like infiltration

Small round cells

Small round cells

Missing Image

Undifferentiated small round cell component

Missing Image

Cyclin D1

Virtual slides

Images hosted on other servers:
YWHAE::NUTM2A/B high grade endometrial stromal sarcoma YWHAE::NUTM2A/B high grade endometrial stromal sarcoma

YWHAE::NUTM2A/B high grade endometrial stromal sarcoma

Positive stains
Negative stains
Molecular / cytogenetics description
Sample pathology report
  • Uterus, fallopian tubes and ovaries, hysterectomy and bilateral salpingo-oophorectomy:
    • ZC3H7B::BCOR rearranged high grade endometrial stromal sarcoma (14 cm) (see comment and synoptic report)
    • Comment: The tumor shows tongue-like myometrial invasion and is composed of enlarged, hyperchromatic spindle cells percolating within a myxoid stroma. Mitoses number up to 15 per 10 high power fields. It is positive for BCOR, cyclin D1 and CD10 but negative for desmin, caldesmon and SMA. Molecular testing confirmed a ZC3H7B::BCOR fusion.
Differential diagnosis
Board review style question #1

A 51 year old woman presents with vaginal bleeding. Histologic sections of the hysterectomy reveal an infiltrative spindle cell neoplasm. Immunohistochemically, the tumor is positive for CD10, cyclin D1 and BCOR but negative for smooth muscle actin, desmin and caldesmon. Next generation sequencing study reveals ZC3H7B::BCOR gene fusion. What is the correct diagnosis?

  1. High grade endometrial stromal sarcoma
  2. Inflammatory myofibroblastic tumor
  3. Leiomyosarcoma
  4. Low grade endometrial stromal sarcoma
  5. Undifferentiated uterine sarcoma
Board review style answer #1
A. High grade endometrial stromal sarcoma

Comment Here

Reference: High grade endometrial stromal sarcoma
Board review style question #2
A subclassification of high grade endometrial stromal sarcoma as per WHO 2020 is based on

  1. Identification of specific gene rearrangement involving YWHAE or BCOR or internal tandem duplication of BCOR gene
  2. Immunohistochemical coexpression of cyclin D1 and BCOR
  3. Immunohistochemical expression of BCOR
  4. Presence of myxoid stromal change
Board review style answer #2
A. Identification of specific gene rearrangement involving YWHAE or BCOR or internal tandem duplication of BCOR gene

Comment Here

Reference: High grade endometrial stromal sarcoma

Histology of specimens from gender affirming surgery in individuals assigned female at birth
Definition / general
  • Hysterectomy is part of gender affirming surgery for individuals assigned female at birth (AFAB)
  • Vaginectomy is a rarely performed gender affirming surgery for individuals AFAB
  • Surgery is typically performed following long periods of preoperative androgen administration
Essential features
ICD coding
  • ICD-10:
    • Z87.890 - personal history of sex reassignment
    • N85.9 - noninflammatory disorder of uterus, unspecified
    • N83.0 - follicular cyst of ovary
    • N89.9 - noninflammatory disorder of vagina, unspecified
Epidemiology
  • Any age, typically 20 - 45 years old
Sites
  • Uterus, cervix, vagina, ovary
Pathophysiology
  • Long term androgenic medication administration
  • Androgen receptor (AR) is present in endometrial cells and when activated, regulates genes involved in cytoskeletal organization and cell progression (Reprod Sci 2014;21:372)
  • Ectocervical transitional cell metaplasia has been described in postmenopausal uteruses (Int J Gynecol Pathol 1997;16:89)
  • Prostatic metaplasia and transitional cell metaplasia of the vagina is associated with long term androgen administration (Am J Surg Pathol 2020;44:1040)
  • Abnormally high androgen levels are known to suppress normal follicular development (Hormones (Athens) 2015;14:190)
Etiology
  • Histopathologic alterations seen in the uterus, vagina and ovary of individuals assigned female at birth are attributed to long term preoperative androgenic medication administration
Diagnosis
  • Diagnosis is typically evident based on the information received from gynecologist (e.g., gender dysphoria)
Case reports
  • 23 year old patient assigned female at birth with adrenogenital syndrome, presenting with transitional cell metaplasia and ectopic prostatic tissue in the uterine cervix and vagina (Int J Gynecol Pathol 2004;23:182)
  • 3 cases of testosterone induced virilization of mesonephric duct remnants and cervical squamous epithelium in patients assigned female at birth (Int J Gynecol Pathol 2017;36:328)
  • 12 patients assigned female at birth between 20 and 32 years old with uterine and ovarian changes during testosterone administration (Taiwan J Obstet Gynecol 2016;55:686)
  • 13 patients assigned female at birth with prostatic metaplasia of the vagina and uterine cervix (Am J Surg Pathol 2020;44:1040)
  • 27 patients assigned female at birth between 20 and 46 years old with histologic features of hysterectomy specimens (Int J Gynecol Pathol 2019;38:520)
  • 55 patients assigned female at birth between 18 and 56 years old with histologic features of hysterectomy specimens (Arch Pathol Lab Med 2022;146:742)
Gross description
  • Typically, gross appearance is unremarkable
Microscopic (histologic) description
Microscopic (histologic) images

Contributed by Mahmoud A. Khalifa, M.D., Ph.D.

Plump stromal cells and gland paucity

Transitional cell metaplasia of ectocervix

Follicular cysts with high follicular density

Prostatic metaplasia of vagina

NKX3.1 stain

Positive stains
  • Prostatic markers NKX3.1, PSA and P501S show positive staining in areas of prostatic metaplasia
Negative stains
  • p16 immunostaining in the cervix is negative and is used to distinguish transitional cell metaplasia from high grade squamous intraepithelial lesion
Sample pathology report
  • Uterus, fallopian tubes and ovaries, hysterectomy with bilateral salpingo-oophorectomy:
    • Inactive / atrophic endometrium with areas of focal decidualization and gland paucity
    • Myometrium with no significant histologic abnormality
    • Cervix with transitional cell metaplasia of ectocervix
    • Ovaries with follicular cysts; follicular density is higher than expected for the patient's age

  • Vagina, vaginectomy:
    • Vaginal tissue with epithelial atrophy with focal transitional cell metaplasia and prostate-like glands / prostatic metaplasia
Differential diagnosis
  • High grade squamous intraepithelial lesion (HSIL):
    • Loss of polarity, cytologic atypia and mitotic figures
    • Immature cells with high N:C ratio, irregular nuclear contour and coarse chromatin
    • Superficial cells may show koilocytotic atypia and viral cytopathic changes
    • p16 immunohistochemical staining shows diffuse and strong nuclear and cytoplasmic staining
Additional references
Board review style question #1

In a hysterectomy from a 27 year old individual with gender dysphoria, the cervical section exhibited the area depicted in the photomicrograph above. Which of the following is true about this field?

  1. Lack of epithelial maturation is consistent with high grade squamous intraepithelial lesion
  2. Occasional nuclear grooves and the presence of umbrella-like cells are interpreted as transitional cell metaplasia
  3. p16 immunohistochemistry will not be of value in this case since it is not reliable in cases of low grade squamous intraepithelial lesions
  4. Plump nuclei and prominent nucleoli suggest reactive inflammatory atypia and regeneration
  5. The presence of occasional perinuclear halos is in keeping with human papillomavirus infection
Board review style answer #1
B. Occasional nuclear grooves and the presence of umbrella-like cells are interpreted as transitional cell metaplasia

Comment Here

Reference: Histology of specimens from gender affirming surgery in individuals assigned female at birth
Board review style question #2
Which of the following is a known outcome of long term administration of androgen to a young individual who was assigned female at birth?

  1. Atypical endometrial hyperplasia
  2. Endocervical polyp
  3. Follicular cysts of the ovary
  4. Leiomyomata
  5. Serous carcinoma of the uterus
Board review style answer #2
Board review style question #3
Which epithelial change is often encountered in transgender vaginectomy specimens?

  1. Endometriosis
  2. Mucinous metaplasia
  3. Prostatic metaplasia
  4. Tubal metaplasia
  5. Vaginal adenosis

Inflammatory myofibroblastic tumor
Definition / general
  • Uncommon mesenchymal neoplasm of myofibroblastic / fibroblastic origin with variable amounts of myxoid stroma and lymphoplasmacytic inflammation; often characterized by ALK fusions
Essential features
  • Best classified as a neoplasm of uncertain malignant potential, as histologically bland uterine confined tumors may recur
  • Comprised predominantly of spindle cells with hypercellular (fascicular / storiform) and hypocellular (myxoid rich) areas, admixed with a variably prominent lymphoplasmacytic infiltrate
  • Often positive for ALK and smooth muscle / stromal markers but staining is of variable intensity and distribution
  • Most harbor ALK fusions but rearrangements involving ROS1, RET and ETV6::NTRK3 have also been reported
Terminology
  • Inflammatory pseudotumor
  • Plasma cell granuloma
ICD coding
  • ICD-10:
    • D39.0 - neoplasm of uncertain behavior of uterus
    • C54.9 - malignant neoplasm of corpus uteri, unspecified
  • ICD-11:
    • 2F76 - neoplasms of uncertain behaviour of female genital organs
    • 2C76.Z - malignant neoplasms of corpus uteri, unspecified
Epidemiology
Sites
  • Uterine corpus is the most common site in the gynecologic tract
    • Rarely arises in the cervix, placenta, ovary, fallopian tube, cul de sac, broad ligament
Pathophysiology
  • Fusion occurs between the promoter of the 5' partner gene and the 3' ALK kinase domain, resulting in ligand independent dimerization and constitutive activation of ALK (Nat Rev Cancer 2013;13:685)
    • Other tyrosine kinase receptors (ROS, RET, ETV6::NTRK3) are less frequently involved
  • TIMP3 and THBS1 are genes involved in endometrial remodeling during implantation and pregnancy (Am J Surg Pathol 2020;44:970)
    • Common 5' partner genes in pregnancy associated tumors
Etiology
  • Unknown
Clinical features
  • Generally present with nonspecific gynecologic symptoms (abnormal uterine bleeding, abdominopelvic pain) or presumed fibroids
  • May be incidentally discovered at cesarean section
Diagnosis
  • Hysterectomy or myomectomy
  • Rarely endometrial sampling
Radiology description
  • No defining features to distinguish from other uterine mesenchymal neoplasms on imaging
Radiology images

Images hosted on other servers:

CT shows intrauterine mass

Axial MRI shows local lesion

Prognostic factors
Case reports
Treatment
  • Surgery (hysterectomy, myomectomy) is first line therapy, as many are presumed leiomyomas
  • Tyrosine kinase inhibitors (i.e., crizotinib) have shown durable responses of at least 12 months in one study (Gynecol Oncol Rep 2021;37:100852)
Gross description
Gross images

Images hosted on other servers:

Polypoid lesion within endometrial cavity

Microscopic (histologic) description
  • Borders range from well circumscribed, to focally irregular, to infiltrative
  • 3 main growth patterns: myxoid, fascicular / compact, hyalinized (Mod Pathol 2017;30:1489)
    • Myxoid: loosely arranged spindle cells (nodular fasciitis-like) in a myxoid background, hypocellular
    • Fascicular / compact: densely arranged spindle cells with fascicular or storiform architecture; may have a smooth muscle appearance or myxoid stroma, hypercellular
    • Hyalinized (infrequent): sparsely cellular collagen resembling a scar
  • Variable degree of lymphoplasmacytic inflammation, occasionally lymphocyte predominant
  • Lymphoid aggregates, foamy histiocytes, neutrophils, eosinophils and Touton giant cells may be seen
  • Thin walled and elongated vessels, occasional thick walled (leiomyoma-like) or staghorn vessels (Mod Pathol 2017;30:1489, Am J Surg Pathol 2019;43:64)
  • Spindle cells have open vesicular nuclei with variable cytologic atypia (rarely severe)
  • Ganglion-like cells (abundant eosinophilic cytoplasm, eccentric nuclei, prominent nucleoli) may be seen and typically comprise a minority of the tumor
  • Spindle cells may show a decidualized appearance during pregnancy (Mod Pathol 2017;30:1489, Am J Surg Pathol 2020;44:970, Hum Pathol 2020;106:62)
  • Mitotic index is usually low and tumor cell necrosis is uncommon
Microscopic (histologic) images

Contributed by Jennifer Bennett, M.D.

Hyper and hypocellular areas

Myxoid pattern

Acellular myxoid areas

Myxoid areas resemble nodular fasciitis

Plasma cell aggregates

Compact pattern


Severe nuclear atypia

Ganglion-like cells

Pregnancy associated IMT

ALK

Positive stains
Negative stains
Electron microscopy description
  • Spindle cells show features of myofibroblasts and are focally surrounded by basal lamina-like material (Int J Gynecol Pathol 1987;6:275)
    • Nondilated rough endoplasmic reticulum
    • Thin filaments with peripheral dense bodies
    • Pinocytic vesicles, occasional Golgi bodies, lipid droplets
Molecular / cytogenetics description
Molecular / cytogenetics images

Images hosted on other servers:

FISH: classic ALK rearrangement

Sample pathology report
  • Uterus, hysterectomy:
    • Inflammatory myofibroblastic tumor (see comment)
    • Comment: The tumor consists of spindled cells in a myxoid stroma admixed with lymphoplasmacytic inflammation. The cells show minimal cytologic atypia, without appreciable mitoses or tumor cell necrosis. The tumor is positive for ALK (granular cytoplasmic staining), desmin (patchy) and CD10 (patchy) but negative for BCOR and caldesmon. The morphology and immunohistochemical profile is consistent with an inflammatory myofibroblastic tumor. If clinically indicated, FISH or RNA fusion analysis can be performed.
Differential diagnosis
Board review style question #1

A 44 year old woman presents with an infiltrative spindle cell neoplasm in the myometrium that has prominent myxoid stroma and a dense lymphoplasmacytic infiltrate. It is positive for desmin, smooth muscle actin and ALK by immunohistochemistry and shows ALK rearrangement by FISH. What is the most likely diagnosis?

  1. BCOR rearranged endometrial stromal sarcoma
  2. Inflammatory myofibroblastic tumor
  3. Myxoid endometrial stromal sarcoma
  4. Myxoid leiomyosarcoma
  5. Postoperative spindle cell nodule
Board review style answer #1
B. Inflammatory myofibroblastic tumor

Comment Here

Reference: Inflammatory myofibroblastic tumor
Board review style question #2
Which gene is rearranged in the majority of uterine inflammatory myofibroblastic tumors?

  1. ALK
  2. BCOR
  3. RANBP2
  4. RRBP1
  5. STAT6
Board review style answer #2

Intravenous and diffuse leiomyomatosis
Definition / general
  • Intravenous leiomyomatosis
    • Rare entity characterized by an intravenous proliferation of benign smooth muscle cells; may coexist with leiomyoma
    • May present with pelvic and extrapelvic spread along venous vasculature
  • Diffuse leiomyomatosis
    • Rare entity characterized by a symmetrical uterine enlargement with innumerable poorly defined, confluent smooth muscle nodules that replace most of the myometrium
Essential features
  • Intravenous leiomyomatosis
    • Intravascular proliferation of tumor plugs composed by bland smooth muscle cells
    • Most cases are confined to the uterus but may present with pelvic and extrapelvic extension through the venous circulatory system
    • Clinical presentation is varied, nonspecific and dependent on tumor extension
    • Despite being a benign tumor, it may cause remarkable systematic complications and has a relatively high risk of recurrence
  • Diffuse leiomyomatosis
    • Diffuse, symmetrical thickening of the myometrium with innumerous poorly defined, coalescent nodules of bland smooth muscle cells
    • Clinical presentation is varied and nonspecific
    • Benign tumor, patients usually do not experience recurrence
Terminology
  • Intravenous leiomyomatosis: intravascular leiomyomatosis
  • Diffuse leiomyomatosis: diffuse uterine leiomyomatosis
ICD coding
  • Intravenous leiomyomatosis
    • ICD-O: 8890/1 - intravenous leiomyomatosis
    • ICD-11: 2E86.0 & XH60C2 - leiomyoma of uterus & intravascular leiomyomatosis
  • Diffuse leiomyomatosis
    • ICD-O: 8890/0 - leiomyoma, NOS
    • ICD-11: 2E86.0 - leiomyoma of uterus
Epidemiology
Sites
Pathophysiology
Etiology
Clinical features
  • Intravenous leiomyomatosis
    • Clinical manifestations are usually diverse, nonspecific and dependent on its location and extension
    • According to one group of authors, clinically, intravenous leiomyomatosis can be divided into 4 stages (Medicine (Baltimore) 2016;95:e4902)
      • Stage I: tumors invading the uterine vein and limited to the pelvis
      • Stage II: tumors extending to the abdominal cavity but without involvement of the renal vein
      • Stage III: tumors invading the renal vein and inferior vena cava and extending further up to the right atrium but without reaching the pulmonary arteries
      • Stage IV: tumors reaching the pulmonary arteries or metastasizing to the lungs
    • ~29.7% of patients reported in literature present in stages I / II and 65.6% in stages III / IV but patients with early lesions or asymptomatic patients may be underreported (Front Surg 2023;9:1020004)
    • When confined to the uterus, most patients are asymptomatic (J Obstet Gynaecol Res 2021;47:4357 Mod Pathol 2016;29:500, Medicine (Baltimore) 2016;95:e4902)
    • Some patients present with nonspecific symptoms suggestive of a pelvic mass and similar to uterine leiomyomas: hypermenorrhea, menostaxis, irregular vaginal bleeding, abdominal swelling or pelvic pain (J Obstet Gynaecol Res 2021;47:4357 Mod Pathol 2016;29:500, Medicine (Baltimore) 2016;95:e4902)
    • Cases with extrapelvic extension may present with edema and heaviness of the lower extremities (J Obstet Gynaecol Res 2021;47:4357)
    • In cases with cardiac and pulmonary extension, chest discomfort, dyspnea and syncope, congestive heart failure, pulmonary embolism and sudden death have been reported (J Obstet Gynaecol Res 2021;47:4357)
  • Diffuse leiomyomatosis
Diagnosis
Radiology description
  • Intravenous leiomyomatosis
    • The most typical CT features of intravenous leiomyomatosis described are (Clin Radiol 2018;73:503.e1, BMC Cancer 2016;16:73)
      • Continuous pelvic and intravenous masses, with heterogeneous enhancement and occasionally, presence of blood vessels in the lesion
      • Lesions in the inferior vena cava that are continuous and shaped like sausages
      • In cases with involvement of the right cardiac chambers, the cardiac imaging findings show a snake head or walking stick head appearance
      • No invasion or adhesion to the vessel wall
      • May present with calcifications
    • Cases invading the inferior vena cava may have a characteristic feature on contrast enhancing CT and MRI: the growth pattern and the small vessels within the tumor create an appearance similar to a sieve on axial images and a luffa sponge on coronal images (Clin Radiol 2018;73:503.e1)
  • Diffuse leiomyomatosis
    • Imaging techniques reveal symmetrical and uniform enlargement of the uterus, with innumerable coalescing nodules of different sizes with indistinct borders in the myometrium (World J Clin Cases 2022;10:8797)
    • Typical leiomyomas are well circumscribed masses with an asymmetrical involvement of the uterus and usually demonstrate low / intermediate signal intensity on T2 weighted images, which can be difficult to appreciate in diffuse leiomyomatosis (Radiol Case Rep 2022;17:1536, Eur Radiol 2018;28:3125)
Radiology images

Images hosted on other servers:

Intravenous leiomyomatosis
Intravenous leiomyomatosis - Contrast enhancing CT

Contrast enhancing CT

Intravenous leiomyomatosis - Magnetic resonance imaging

MRI

 
Diffuse leiomyomatosis
Diffuse leiomyomatosis - Magnetic resonance imaging

MRI

Prognostic factors
Case reports
  • Intravenous leiomyomatosis
    • 31 year old woman with a history of myomectomy presented with abnormal uterine bleeding, anemia and multiple uterine nodules (Radiol Case Rep 2022;17:4203)
    • 49 year old woman presented with dyspnea, abdominal distension and a large uterine mass extending to the right parauterine veins, right ovarian vein and the inferior vena cava (Taiwan J Obstet Gynecol 2021;60:367)
    • 52 year old woman presented with lower abdominal pain, a 270 mm uterine mass and a 78 x 47 mm mass in the right atrium (Diagn Pathol 2020;15:4)
  • Diffuse leiomyomatosis
    • 27 year old woman presented with heavy vaginal bleeding and a regularly enlarged uterus (World J Clin Cases 2022;10:8797)
    • 36 year old woman presented with a 1 year history of lower abdominal discomfort and dysmenorrhea and a markedly enlarged uterus replaced by innumerable nodules (Radiol Case Rep 2022;17:1536)
    • 38 year old nulliparous woman presented with abdominal distension and an enlarged uterus of size 250 x 200 x 130 mm (Cureus 2022;14:e29595)
Treatment
  • Intravenous leiomyomatosis
    • Surgery is the main treatment option (J Obstet Gynaecol Res 2021;47:4357, Medicine (Baltimore) 2021;100:e24228, J Int Med Res 2020;48:300060519896887, Front Surg 2023;9:1020004)
      • Complete removal of the lesion is crucial to obtain a favorable prognosis
      • Controversy surrounding the selection of the different surgical approaches
      • Hysterectomy is recommended for most patients
      • Bilateral oophorectomy has also been recommended for patients who are menopausal / perimenopausal or with extrauterine extension but recent studies questioned its usefulness
      • Myomectomy has been proposed as an option for young women who wish to preserve fertility but it has been associated with an increased risk of recurrence
      • Multidisciplinary surgical approach is required in cases of extrapelvic extension
      • 1 stage or 2 stage surgery may be employed, depending on several factors, such as patient's tolerance and tumor extension
      • Complete tumor resection is achieved in 56 - 92.1% of patients
    • Use of postoperative hormone treatment is controversial (J Obstet Gynaecol Res 2021;47:4357, Medicine (Baltimore) 2021;100:e24228)
      • Gonadotropin releasing hormone agonist was previously recommended as a postoperative approach to reducing the risk of recurrence in cases with preservation of both ovaries or impossibility of complete resection but recent studies have questioned its effectiveness
      • Use of tamoxifen and aromatase inhibitors has also been reported
  • Diffuse leiomyomatosis
Gross description
Gross images

Contributed by Sabrina Croce, M.D., Ph.D.

Intravenous leiomyomatosis
Tumor plugs

Tumor plugs

 
Diffuse leiomyomatosis
Diffuse myometrial expansion

Diffuse myometrial expansion

Numerous coalescing nodules

Numerous coalescing nodules

Symmetrical myometrial enlargement

Symmetrical myometrial enlargement



Images hosted on other servers:

Intravenous leiomyomatosis
Intravenous leiomyomatosis Intravenous leiomyomatosis

Tumor plugs

Involvement of myometrial vessels

Microscopic (histologic) description
Microscopic (histologic) images

Contributed by Sabrina Croce, M.D., Ph.D.

Intravenous leiomyomatosis
Tumor plugs Tumor plugs

Tumor plugs

Tumor plugs

Tumor plugs

Plexiform appearance

Plexiform appearance


Coexisting leiomyoma

Coexisting leiomyoma

Estrogen receptor

Estrogen receptor

Desmin

Desmin

Desmin

CD34



Diffuse leiomyomatosis
Numerous confluent nodules Numerous confluent nodules

Numerous confluent nodules

Poorly defined nodules Poorly defined nodules

Poorly defined nodules


Well defined contours

Well defined contours

Progesterone receptor

Progesterone receptor

Progesterone receptor

Caldesmon

Virtual slides

Images hosted on other servers:
Intravenous leiomyomatosis

Intravenous leiomyomatosis

Diffuse leiomyomatosis

Diffuse leiomyomatosis

Positive stains
Negative stains
Molecular / cytogenetics description
  • Intravenous leiomyomatosis
    • Some cases of intravenous leiomyomatosis demonstrate balanced translocations t(12;14)(q14-15;q23-24), commonly observed in uterine leiomyomas (Hum Pathol 2022;120:18, Mod Pathol 2016;29:500, Cancer Med 2020;9:4581)
      • Breakpoint on 12q14-15 typically results in overexpression of HMGA2, as evidenced by high levels of HMGA2 staining
      • HMGA2 overexpression and t(12;14)(q15;q24) likely have important roles in tumorigenesis but additional genetic alterations might explain the quasimalignant behavior of intravenous leiomyomatosis
    • Recurrent 22q and 1p regional losses and 12q gains have been reported (Cancers (Basel) 2022;14:2907, Mod Pathol 2016;29:500, Int J Gynecol Pathol 2015;34:169)
    • Unlike leiomyomas, MED12 mutations have been rarely reported in intravenous leiomyomatosis (Cancers (Basel) 2022;14:2907, Mod Pathol 2016;29:500, Int J Gynecol Pathol 2015;34:169)
    • X chromosome inactivation analysis confirmed the monoclonal origin of intravenous leiomyomatosis (Mod Pathol 2002;15:351, Mod Pathol 2016;29:500)
    • Comparative genomic hybridization analysis on a benign metastasizing leiomyoma of the lung and uterine intravenous leiomyomatosis suggested that these entities may represent different points from the same neoplastic continuum (Pathol Res Pract 2018;214:871, Mod Pathol 2002;15:351)
    • RNA sequencing shows that intravenous leiomyomatosis appears to have higher expression of angiogenesis and antiapoptotic factors when compared to uterine leiomyomas (Cancer Med 2020;9:4581)
      • These factors may be associated with the characteristic continuous extension growth and proliferation rate of intravenous leiomyomatosis
      • Gene expression profile of intravenous leiomyomatosis is more complex than that of leiomyomas
    • Comparative proteomic profiling study of intravenous leiomyomatosis and other smooth muscle tumors described the selective enrichment of coregulated splicing factors, which may be associated with distinct biological pathways (Cancers (Basel) 2022;14:2907)
  • Diffuse leiomyomatosis
Videos

Intravenous leiomyomatosis

Sample pathology report
  • Uterus, hysterectomy:
    • Intravenous leiomyomatosis (see comment)
    • Comment: Microscopic examination reveals several tumor plugs inside dilated vessels. The tumor plugs are comprised of smooth muscle cells with a fascicular and plexiform architecture and are lined by endothelial cells. The mitotic activity is estimated at 2 mitoses/10 high power fields. No cytologic atypia or tumor cell necrosis is observed. The tumor plugs present with extension to the broad ligament vessels. Additionally, an intramural uterine leiomyoma is observed. Intravenous leiomyomatosis is a benign smooth muscle tumor, albeit with the potential for locoregional and distant recurrence. Clinical and imagiological correlation to determine the presence of intravenous extension beyond the uterus and broad ligament, as well as postoperative residual lesions, is advised. Close clinical follow up is recommended.

  • Uterus, hysterectomy:
    • Diffuse leiomyomatosis (see comment)
    • Comment: Microscopic examination reveals numerous poorly circumscribed, coalescent nodules within the myometrium, composed of smooth muscle cells, organized into interlacing fascicles. The mitotic activity is estimated at 1 mitosis/10 high power fields. No cytologic atypia or tumor cell necrosis is observed. The morphology is consistent with diffuse leiomyomatosis.
Differential diagnosis
  • Intravenous leiomyomatosis
    • Uterine lymphangioleiomyomatosis:
    • Leiomyoma with vascular invasion:
    • Dissecting (cotyledonoid) leiomyoma:
      • Characterized by tongues and broad fascicles of smooth muscle cells, dissecting the myometrium
      • May have varying degrees of hydropic change and numerous blood vessels (Hum Pathol 2011;42:1240)
      • No atypia or necrosis
      • No intravascular growth (Hum Pathol 2011;42:1240)
    • Uterine leiomyosarcoma:
      • Moderate / marked cytologic atypia, presence of tumor cell necrosis or numerous mitotic figures, including atypical mitosis
      • Recurrences of intravenous leiomyomatosis may lead to suspicion of a missed diagnosis of a leiomyosarcoma
        • Bland nuclear features, low mitotic activity and lack of tumor cell necrosis favor intravenous leiomyomatosis
        • History of hysterectomy / myomectomies and histological review of previous uterine specimens (if possible) may also aid in the diagnosis
        • Consultation services should be considered for difficult cases
    • Low grade endometrial stromal sarcoma:
      • Composed of cells resembling proliferative phase endometrium
      • Infiltrative / permeative growth in the myometrium
      • May be associated with lymphovascular invasion, with numerous intravascular tumor plugs, grossly detected
      • Most cases show diffuse, strong expression of ER, PR, CD10 and IFITM1
      • Immunohistochemical panel of at least 2 smooth muscle markers (desmin and caldesmon) and CD10 may be helpful in the differential diagnosis
      • May show areas with smooth muscle differentiation, which may be positive for caldesmon, desmin, ER, PR and variably positive for CD10
        • Searching for areas with the classical morphological and immunohistochemical features and molecular testing may aid in the diagnosis
      • Differential diagnosis with intravenous leiomyomatosis may be challenging, especially with intravenous adenomyomatosis comprised only of endometrial stroma
  • Diffuse leiomyomatosis
    • Multiple leiomyomas:
      • Asymmetrical involvement of the myometrium and sharp circumscription of the individual leiomyomas, unlike diffuse leiomyomatosis
    • Intravascular leiomyomatosis:
      • May also have a multinodular appearance
      • Presence of coalescing nodules without associated intravascular involvement allows the diagnosis of diffuse leiomyomatosis
      • Some cases of diffuse leiomyomatosis may present with concomitant intravenous leiomyomatosis
    • Endometrial stromal sarcoma:
      • Multinodular appearance and permeative growth may raise this differential diagnosis
      • Morphological, immunohistochemical and molecular features allow for this diagnosis
    • PEComatosis:
Board review style question #1

A 42 year old woman with metrorrhagia, abdominal pain and imaging findings suggestive of multiple leiomyomas was submitted to a hysterectomy. A representative microscopic image is shown above. Which of the following hypotheses is true?

  1. CD10 is consistently negative in these tumors
  2. High mitotic rates are frequently observed in these tumors
  3. These are benign tumors, without risk of spread beyond the uterus
  4. These lesions are associated with tuberous sclerosis complex
  5. These lesions may coexist with leiomyomas
Board review style answer #1
E. These lesions may coexist with leiomyomas. Presence of a coexisting uterine leiomyoma has been estimated to be 38.9 - 61.5% of cases. Answer D is incorrect because intravenous leiomyomatosis has not been associated with tuberous sclerosis complex, unlike lymphangioleiomyomatosis, an important differential diagnosis. Answer A is incorrect because CD10 is weakly positive in some cases and strongly positive in cases of intravascular adenomyomatosis. Answer B is incorrect because these tumors present with no or occasional mitotic figures (< 2/10 high power fields). Answer C is incorrect because intravenous leiomyomatosis is a benign tumor that may present with pelvic and extrapelvic spread along venous vasculature.

Comment Here

Reference: Intravenous and diffuse leiomyomatosis
Board review style question #2
Which of the following observations is true regarding diffuse uterine leiomyomatosis?

  1. Cytologic atypia and high mitotic counts are typical of this tumor
  2. It is a benign condition with a relatively high risk of recurrence
  3. It is often characterized by an asymmetrical thickening of the myometrium
  4. Numerous well defined, confluent nodules are present in the myometrium
  5. X chromosome inactivation analysis suggests a polyclonal origin of the different nodules
Board review style answer #2
E. X chromosome inactivation analysis suggests a polyclonal origin of the different nodules. X chromosome inactivation analysis supports the hypothesis of an independent clonal origin of the different nodules (Hum Pathol 2000;31:1429). Answers C and D are incorrect because diffuse leiomyomatosis classically is characterized by a diffuse, symmetrical thickening of the myometrium with numerous poorly defined, confluent nodules. Answer B is incorrect because patients usually do not experience recurrence (only 2 cases of recurrence after conservative surgery have been reported). Answer A is incorrect because diffuse leiomyomatosis does not show significant cytologic atypia and only demonstrates low mitotic activity.

Comment Here

Reference: Intravenous and diffuse leiomyomatosis

Leiomyoma-general
Definition / general
Essential features
  • Benign mesenchymal tumor derived from smooth muscle
  • 90% of leiomyomas are the conventional type:
    • Conventional / usual leiomyoma:
      • Monotonous spindle cells with indistinct borders arranged in intersecting fascicles
      • Mitoses: rare (in general < 5/10 high power fields)
    • Subtypes:
      • Cellular
      • Leiomyoma with bizarre nuclei
      • Fumarate hydratase (FH) deficient
      • Mitotically active
      • Hydropic
      • Apoplectic
      • Lipoleiomyoma
      • Epithelioid
      • Myxoid
      • Dissecting leiomyoma
      • Diffuse leiomyomatosis
ICD coding
  • ICD-O: 8890/0 - leiomyoma, NOS
  • ICD-11: 2E86.0 - leiomyoma of uterus
Epidemiology
Sites
  • Uterine corpus
  • Less common in vulva, vagina, cervix, broad ligament, ovary
Pathophysiology
  • Leiomyomas have a clonal origin
  • Several alterations identified (Table 1) (Clin Obstet Gynecol 2016;59:25)
  • Risk decreased by oral contraceptives
  • May occur in hereditary leiomyomatosis and renal cancer syndrome (HLRCC) and Alport syndrome
Etiology
  • Mitotically active leiomyoma: associated ischemia or hormonal stimulation (endogenous or exogenous) (Am J Surg Pathol 2016;40:563)
  • Apoplectic leiomyoma: progesterone (or analogues) exposure (contraception or pregnancy and postpartum)
Diagrams / tables

Contributed by Sabrina Croce, M.D., Ph.D.

Table 1: molecular alterations



Table 2: differential diagnosis in uterine spindle cell smooth muscle lesions
Tumor cell necrosis
Atypia
Mitoses*
Diagnosis
- + ≥ 10 Leiomyosarcoma
+ + ≥ 10
+ + < 10
- - > 15** Smooth muscle tumor of
uncertain malignant
potential (STUMP)
- + < 10
+ - < 10
- - ≥ 6 and ≤ 15** Mitotically active leiomyoma
* Mitoses/mm2 (mitoses/10 HPF of 0.55 mm in diameter)
** > 15 or ≥ 15, cutoff value varies according to the references


Table 3: differential diagnosis in uterine myxoid smooth muscle lesions
Myxoid leiomyoma
Myxoid STUMP
Myxoid leiomyosarcoma
Infiltrative borders - -
1 or more of these criteria:
  • Mitoses: ≥ 2
  • Tumor cell necrosis
  • Atypia
Atypia - -
Tumor cell necrosis - -
Mitoses* - 1
* Mitoses/mm2 (mitoses/10 HPF of 0.55 mm in diameter)


Table 4: differential diagnosis in uterine epithelioid smooth muscle lesions
Epithelioid leiomyoma
Epithelioid STUMP
Epithelioid leiomyosarcoma
Atypia - - 1 or more of these criteria:
  • Mitoses: ≥ 4
  • Tumor cell necrosis
  • Atypia
Tumor cell necrosis - -
Mitoses* < 2 ≥ 2 and < 4
* Mitoses/mm2 (mitoses/10 HPF of 0.55 mm in diameter)
Clinical features
  • 25% symptomatic; remainder asymptomatic
  • Symptoms depend on size and location
  • Menorrhagia and pelvic pain in 33% of patients
  • Reference: Nat Rev Dis Primers 2016;2:16043
Benign metastasizing leiomyoma
  • Rare, benign appearing smooth muscle tumor in lung; may represent hematogenous spread of a uterine leiomyoma or a metastasis of a well differentiated leiomyosarcoma of low malignant potential (Mod Pathol 2006;19:130)
  • Usually women 36 - 64 years, mean 44 years, with history of uterine leiomyoma
  • Lung is most common site, sparing bronchus and pleura; also reported in lymph nodes, retroperitoneum, skin, bone, spine, skull base, heart
  • Usually multiple nodules, up to a few centimeters in size
  • Lesions tend to regress during pregnancy or after oophorectomy and stabilize or grow slowly in postmenopausal women
  • Usually asymptomatic but may present with dyspnea, cough, hemoptysis, chest pain
  • Chest Xray: diffuse, bilateral nodular opacities; rarely associated with miliary pattern, cavitary lesions, multiloculated cysts, interstitial lung disease (J Thorac Dis 2014;6:E92)
  • Surveillance acceptable treatment for indolent, asymptomatic disease
Diagnosis
  • Can be established by resection of the whole uterus (hysterectomy), by resection of the leiomyoma if accessible by curetting (if submucosal) or by myomectomy (if subserosal)
  • Ultrasound or magnetic resonance imaging (MRI) (guided core biopsy of the leiomyoma is a promising new procedure (Int J Gynecol Cancer 2020;30:A113)
Prognostic factors
  • Uterine leiomyomas are benign tumors that may recur, especially after myomectomy
  • Fumarate hydratase deficient leiomyomas:
    • Recur more frequently
    • Genetic counseling should be recommended
Case reports
Treatment
  • Asymptomatic: does not require therapy
  • Symptomatic leiomyoma:
    • Surgery (hysterectomy or myomectomy)
    • Hysteroscopic resection
    • Medical treatment - progestins / levonorgestrel intrauterine system, gonadotropin releasing hormone (GnRH) analogs or aromatase inhibitors
      • Ulipristal acetate; widely used in conservative treatment of uterine leiomyomas but rejected by the FDA due to risk of liver toxicity
    • Interventional treatments - uterine artery embolization or radiofrequency myolysis
  • Reference: J Obstet Gynaecol Can 2015;37:157
Gross description
  • Location in the uterus: intramural, submucosal and subserosal
  • Often multiple
  • Typically well circumscribed but nonencapsulated
  • On cut surface: white or tan-white, whorled, firm, bulging
  • Hemorrhage and infarction can be present in large tumors
  • Calcifications can be present
  • Apoplectic change (foci of hemorrhage) associated with progesterone therapy
  • Extensive sampling (to exclude malignancy)
    • Especially in leiomyomas that lack the classic gross appearance
    • Myxoid areas to exclude myxoid leiomyosarcoma
  • Reference: Mod Pathol 2016;29:S104
Gross images

Contributed by Sabrina Croce, M.D., Ph.D. and @Andrew_Fltv on Twitter

Submucosal leiomyoma

Intramural leiomyoma

Subserosal leiomyomas

Apoplectic leiomyoma


Conventional / usual leiomyoma

Cystic hydropic leiomyoma

Cystic hydropic leiomyoma

Microscopic (histologic) description
  • Conventional / usual leiomyoma (spindle):
    • Well defined borders
    • Normocellular
    • Intersecting fascicles of monotonous spindle cells with indistinct borders, eosinophilic cytoplasm, cigar shaped nuclei (with tapered ends) and small nucleoli
    • Atypia: absent or mild
    • Mitoses: rare (in general < 5/10 high power fields)
    • Blood vessels with thick walls
    • With or without infarct type necrosis, hyalinization, calcification, cystic change
  • Subtypes:
    • Cellular
      • Increased cellularity (more cellular than background myometrium)
      • Scant cytoplasm without increased mitotic activity and atypia
      • May have irregular borders
      • Highly cellular leiomyoma is not a WHO diagnosis
    • Leiomyoma with bizarre nuclei:
    • Fumarate hydratase deficient leiomyoma:
      • Alveolar edema
      • Staghorn or hemangiopericytoma-like vessels
      • Chain-like growth of tumor cells
      • Spindle or epithelioid cells with ovoid nuclei and prominent eosinophilic nucleoli surrounded by perinucleolar halos
      • Rhabdoid / eosinophilic cytoplasmic inclusions
      • May include multinucleated cells and cells with bizarre nuclei
    • Mitotically active:
      • Spindle cell leiomyoma without atypia or tumor cell necrosis
      • Increased mitotic activity (the mitotic cutoff varies according to the authors 6 - 14 mitoses/10 high power fields or 6 - 15 mitoses/10 high power fields)
    • Hydropic:
      • Tumor cells separated by watery or eosinophilic and proteinaceous fluid, resulting in a trabecular, nested architecture
    • Apoplectic:
      • Central zone of hemorrhage and necrosis with increased mitotic activity in its periphery or myxoid changes (zonation phenomenon)
      • Usual appearance away from the central necrosis
    • Lipoleiomyoma:
      • Tumor composed of smooth muscle cells mixed with mature adipocytes (variable quantity)
    • Epithelioid:
      • Round or polygonal cells with eosinophilic or clear cytoplasm (in general, ≥ 50% of tumor cells)
      • Nested or trabecular architecture
      • No cytologic atypia or tumor cell necrosis
      • Mitotic count is < 2 mitoses/10 high power fields
    • Myxoid:
      • Hypocellular tumor
      • Cells separated by myxoid matrix composed of glycosaminoglycans (Alcian blue+) occupying ≥ 50% of the overall tumor volume
      • Well circumscribed borders (most important criteria)
      • No cytologic atypia, mitoses or tumor cell necrosis
    • Dissecting leiomyoma:
      • Nodules of smooth muscle cells dissecting the myometrium; occasionally hydropic changes and intravenous extension can be seen
      • Called cotyledonoid leiomyoma if extends outside the uterus
    • Diffuse leiomyomatosis:
      • Diffuse, poorly circumscribed, innumerable tumor nodules in the myometrium
      • No atypia and tumor cell necrosis
      • Low mitotic count
Microscopic (histologic) images

Contributed by Sabrina Croce, M.D., Ph.D., Kristina Doytcheva, M.D., Jennifer A. Bennett, M.D. (Case #508) and @Andrew_Fltv on Twitter

Conventional / usual leiomyoma

Cellular leiomyoma


FH deficient leiomyoma


Leiomyoma with bizarre nuclei


Hydropic leiomyoma

Apoplectic leiomyoma

Epithelioid leiomyoma

Lipoleiomyoma


Myxoid leiomyoma

Fumarate hydratase deficient (FH-d) leiomyoma


Benign metastasizing leiomyoma

Cystic hydropic leiomyoma Cystic hydropic leiomyoma Cystic hydropic leiomyoma

Cystic hydropic leiomyoma


Virtual slides

Images hosted on other servers:

Leiomyoma

Leiomyoma with bizarre nuclei

Highly cellular leiomyoma

Lipoleiomyoma

Positive stains
Negative stains
  • p16 negative or patchy (except in leiomyomas with bizarre nuclei, where it can be diffusely positive)
  • p53 typically wildtype (except in leiomyomas with bizarre nuclei, where it can show aberrant expression)
Molecular / cytogenetics description
Sample pathology report
  • Uterus, total hysterectomy:
    • Conventional leiomyoma: 5.0 cm (see comment)
    • Comment: Microscopic examination reveals a smooth muscle tumor composed of spindle, cigar shaped cells arranged in fascicular pattern without cytologic atypia and tumor cell necrosis. Mitoses are rare (4 mitoses/10 high power fields). Tumor borders are well circumscribed. By immunohistochemistry the tumor cells are positive for desmin, h-caldesmon, ER and PR.
Differential diagnosis
Board review style question #1

A 23 year old woman with multiple recurrent leiomyomas is diagnosed with a FH deficient leiomyoma. Which of the following is true?

  1. FH deficiency is a risk factor for uterine leiomyosarcoma
  2. Staghorn vessels, alveolar oedema, macronuclei with perinucleolar halos and cytoplasmic eosinophilic globules are typical morphologic features of FH deficient leiomyoma
  3. Bizarre nuclei and FH deficient leiomyoma are mutually exclusive
  4. All FH deficient leiomyomas are 2SC positive and FH negative
  5. All FH deficient leiomyomas are associated with a germline mutation
Board review style answer #1
B. Staghorn vessels, alveolar oedema, macronuclei with perinucleolar halos and cytoplasmic eosinophilic globules are typical morphologic features of FH deficient leiomyoma

Comment Here

Reference: Leiomyoma-general
Board review style question #2

A 44 year old woman presents with 4 cm leiomyoma with bizarre nuclei. Which of the following is true?

  1. The presence of atypia alone is a sufficient criterion warrant the diagnosis of leiomyosarcoma
  2. Significant number of leiomyomas with bizarre nuclei harbor FH biallelic inactivation
  3. Leiomyomas with bizarre nuclei are malignant
  4. p16 staining is a reliable marker of leiomyomas with bizarre nuclei
  5. Arteriolarization of vessels is a typical feature of bizarre nuclei leiomyoma
Board review style answer #2
B. Significant number of leiomyomas with bizarre nuclei harbor FH biallelic inactivation

Comment Here

Reference: Leiomyoma-general

Leiomyosarcoma
Definition / general
  • Rare, malignant mesenchymal tumor derived from myometrial smooth muscle
  • Most common sarcoma of the gynecologic tract
Essential features
ICD coding
  • ICD-O:
    • 8890/3 - leiomyosarcoma, NOS
    • 8891/3 - epithelioid leiomyosarcoma
    • 8896/3 - myxoid leiomyosarcoma
  • ICD-10: C55 - malignant neoplasm of uterus, part unspecified
  • ICD-11: 2B58.1 - leiomyosarcoma of uterus
Epidemiology
Sites
  • Uterus corpus
  • Cervix, rare
Pathophysiology
  • Derived from smooth muscle
  • Vast array of associated cytogenetic abnormalities but none are consistent or diagnostic
  • Most frequently mutated genes: TP53 (~30%), ATRX (~25%) and MED12 (~20%) (Proc Natl Acad Sci U S A 2021;118:e2025182118)
Etiology
Clinical features
  • Nonspecific symptoms:
    • Abnormal uterine bleeding, pelvic or abdominal pain
  • Rapidly growing uterine mass in a postmenopausal woman
  • Usually an incidental finding, identified in 0.13% of hysterectomies for benign indication and 0.39% of hysterectomies for uterine leiomyomas (Am J Obstet Gynecol 2019;220:179.e1)
Diagnosis
  • Myomectomy, hysterectomy
Laboratory
  • No laboratory values are diagnostic:
    • Some studies suggest leiomyosarcomas have higher levels of lactate dehydrogenase (LDH) versus uterine leiomyoma (BMC Cancer 2020;20:514)
    • Variable success as a predictive marker
Radiology description
  • No pathognomonic findings
  • Difficult to distinguish from benign smooth muscle tumors (Curr Opin Oncol 2021;33:464)
  • TVUS:
    • Most common initial imaging modality
      • Successfully detects uterine leiomyomas
      • Does not differentiate between leiomyomas and leiomyosarcomas
  • CT: not indicated for assessing uterine masses
    • May show irregular central zones of low attenuation, suggesting necrosis and hemorrhage
  • MRI, conventional techniques:
    • Ill defined borders
    • Central nonenhancement
    • T1 weighted images with hyperintensity associated with tumoral hemorrhage or necrosis
    • T2 weighted images show heterogeneous intermediate to high signal intensities
  • Advanced imaging modalities show potential diagnostic improvement:
Radiology images

Images hosted on other servers:

Large intrauterine mass

Heterogeneous mass with irregular borders

Prognostic factors
  • Most important prognostic factor is stage
  • Additional prognostic measures include age, tumor size, mitotic rate and lymphovascular invasion
  • High recurrence rate (50 - 70%) regardless of stage at initial diagnosis
  • Poor prognosis even if the tumor is confined to the uterus
    • More favorable prognosis if tumor is < 5 cm and confined to the uterus
  • 5 year overall survival (OS) rate for all stages is poor, ranging from 15 to 25%
    • OS rates are more favorable at low stages (1 - 2), ranging from 40 to 70%
  • Morcellation is associated with significantly increased risk of recurrence (Gynecol Oncol 2021;160:99)
  • Risk stratification model, including mitoses > 25 per 2.4 mm2 (10 high power fields), atypical mitoses, coagulative necrosis, lymphovascular invasion and serosal abutment, is significantly associated with disease free and disease specific survival in stage I tumors (Mod Pathol 2022;35:794):
    • 3 risk groups include low risk (0- 2 points), intermediate risk (3 - 5 points) and high risk (6 - 13 points)
    • Serosal abutment and lymphovascular invasion can be omitted for myomectomy or morcellated specimens
Case reports
Treatment
  • Surgical resection (hysterectomy) is standard treatment for patients with localized leiomyosarcoma:
    • Adjuvant chemotherapy for early stage disease is controversial
    • Bilateral salpingo-oophorectomy is reasonable in peri and postmenopausal patients
  • Lymphadenectomy is only indicated if there is evidence of concerning lymph nodes (J Adv Pract Oncol 2022;13:70, StatPearls: Leiomyosarcoma [Accessed 26 October 2022])
  • Radiotherapy is indicated for palliative care purposes in advanced or metastatic disease (Cancer Med 2022;11:2906):
    • No evidence for increased overall survival
  • Adjuvant chemotherapy is indicated for metastatic / recurrent tumors
  • Immunotherapy is a potential option for MSI high uterine leiomyosarcoma (Gynecol Oncol Rep 2021;35:100701)
  • Hormonal therapy may be an option in hormone receptor positive tumors
Clinical images

Images hosted on other servers:

Uterus with dilated cervix

Large exophytic intrauterine mass

Large mass

Gross description
  • Often a solitary, bulky, fleshy mass within the myometrium - intramural (majority), submucosal, subserosal or pedunculated:
    • Hemorrhagic, necrotic and cystic areas upon sectioning
    • Grossly invasive / infiltrative
  • Rarely may arise in the cervix (5%)
  • Average diameter is 10 cm:
    • ~25% are < 5 cm
  • Myxoid leiomyosarcoma:
    • Gelatinous cut surface
    • Friable
  • References: Oncol Res Treat 2018;41:680, Arch Pathol Lab Med 2008;132:595
Gross images

Contributed by Ashley Monsrud, M.D.
Formalin fixed, intracavitary leiomyosarcoma

Formalin fixed, intracavitary leiomyosarcoma



Images hosted on other servers:

Large mass

Frozen section description
  • Not usually performed
  • If done, assess:
    • Cellularity
    • Significant cytologic atypia
    • Number of unequivocal mitotic figures
    • Tumor cell necrosis:
      • Karyorrhexis, perivascular cuffs of viable tumor cells in a background of necrosis and ghost cells are diagnostic clues for tumor cell necrosis (Am J Surg Pathol 2021;45:1179)
  • If malignant criteria are met:
    • Best practice is to call "malignant mesenchymal tumor”
Microscopic (histologic) description
  • Conventional / spindle cell type:
    • Essential diagnostic criteria:
      • Requires 2 of 3 histologic features:
        • Marked cytologic atypia
        • ≥ 10 mitoses / 10 high power fields
        • Tumor cell necrosis
          • Identified by abrupt transition from viable tumor cells to necrotic cells (ghost cells, apoptotic bodies may be seen)
          • Granulation tissue surrounding necrosis is absent
    • Growth pattern:
      • Cellular tumor comprised of long intersecting or haphazard fascicles
      • Infiltrative border (common)
      • Rarely, may arise from background leiomyoma
    • Cytologic features:
      • Spindle / elongated cells
      • Eosinophilic cytoplasm
      • Hyperchromatic nuclei often with moderate to severe nuclear pleomorphism (can be deceptively bland and uniform)
      • Atypical mitoses are frequently identified
      • Multinucleated and osteoclast-like giant cells may be seen
  • Myxoid leiomyosarcoma:
    • Diagnosis based on ≥ 1 of the following:
      • Moderate to severe cytologic atypia
      • Coagulative tumor cell necrosis
      • ≥ 2 mitosis / 10 high power fields
      • Infiltrative borders / irregular margins
    • Growth pattern:
      • Hypocellular tumor with abundant myxoid stroma
      • Fascicular or nodular patterns are uncommon
      • Myxoid stroma may be difficult to differentiate from hydropic change in small / limited samples
    • Extensive sampling is generally required for diagnosis
  • Epithelioid leiomyosarcoma:
    • Diagnosis based on ≥ 1 of the following:
      • Moderate to severe cytologic atypia
      • Tumor cell necrosis
      • ≥ 4 mitoses / 10 high power fields
    • Growth pattern:
      • Arranged in nests, cords or sheets
      • May show pseudoglandular spaces
    • Cytologic features:
      • > 50% of round or polygonal cells with eosinophilic or clear cytoplasm
      • Rarely, extensive hyalinization
    • Alternative criteria include ≥ 2 of the following features: moderate or severe atypia, ≥ 4 mitoses / 2.4 mm2 and tumor cell necrosis (Am J Surg Pathol 2022;46:464)
  • Classification systems used:
    • FIGO
    • TNM
  • References: Arch Pathol Lab Med 2008;132:595, StatPearls: Leiomyosarcoma [Accessed 26 October 2022]
Microscopic (histologic) images

Contributed by Ashley Monsrud, M.D. and Paulette Mhawech-Fauceglia, M.D.
Spindle cell leiomyosarcoma Spindle cell leiomyosarcoma

Spindle cell leiomyosarcoma

Tumor cell necrosis Tumor cell necrosis Tumor cell necrosis Tumor cell necrosis

Tumor cell necrosis


Atypical mitotic figures

Atypical mitotic figures

Hypercellular tumor

Nuclear atypia

Numerous mitoses


Myxoid mesenchymal, infiltrating border

Myxoid leiomyosarcoma, infiltrating border

Myxoid leiomyosarcoma Myxoid leiomyosarcoma

Myxoid leiomyosarcoma

Mitotic figures in myxoid mesenchymal neoplasm

Mitotic figures in myxoid leiomyosarcoma

Epithelioid mesenchymal neoplasm

Epithelioid leiomyosarcoma


Solid pattern and nuclear atypia

Epithelioid leiomyosarcoma

Infiltration of benign smooth muscle

Infiltration of adjacent myometrium

Solid growth pattern

Solid growth pattern

Nuclear atypia

Nuclear atypia

Epithelioid subtype

Epithelioid subtype

Virtual slides

Images hosted on other servers:

Leiomyosarcoma with extensive lymphovascular invasion

Expanding pelvic mass in 60 year old woman

Positive stains
Negative stains
Molecular / cytogenetics description
Molecular / cytogenetics images

Images hosted on other servers:

Somatic mutational landscape

Videos

Review of uterine leiomyosarcoma

Sample pathology report
  • Uterus and cervix, total hysterectomy:
    • Myometrium:
      • Leiomyosarcoma
      • Tumor size: 15 cm
      • Cytologic atypia: diffuse, marked
      • Coagulative tumor cell necrosis: present
      • Mitotic count: 25 per 10 high power fields
      • Lymphovascular invasion: negative
      • Margin status: negative
      • Other findings: leiomyomata
      • See synoptic report
    • Endometrium: inactive
    • Uterine serosa: benign
    • Cervix: benign
Differential diagnosis
  • Endometrial stromal sarcoma:
    • Low grade endometrial stromal sarcoma:
      • Tumor cells resemble proliferative type endometrial stroma
      • Minimal cytologic atypia and low mitotic index
      • Diffusely positive for CD10 and ER / PR but h-caldesmon typically negative or weak expression
      • Cyclin D1 (focal)
      • ~66% harbor gene fusions:
        • JAZF1::SUZ12 (most common), followed by JAZF1::PHF1, EPC1::PHF1 and MEAF6::PHF1
    • High grade endometrial stromal sarcoma:
      • Round or spindle high grade cells with brisk mitoses and necrosis
      • Infiltrative growth pattern
      • Molecular alterations are: YWHAE::NUTM2A / YWHAE::NUTM2B fusion, ZC3H7B::BCOR and BCOR internal tandem duplication (ITD)
  • Leiomyoma variants:
    • Mitotically active leiomyoma:
      • No cytologic atypia or coagulative tumor cell necrosis
      • 6 - 14 mitoses / 10 HPF
    • Leiomyoma with apoplectic changes:
      • Zonation phenomena (benign smooth muscle away from necrotic areas)
    • Cellular leiomyoma:
      • Increased cellularity but no cytologic atypia or tumor necrosis
    • Leiomyoma with bizarre nuclei:
      • Scattered or diffuse bizarre nuclei with adjacent areas of classic leiomyoma
    • Myxoid leiomyoma:
      • Usually focal myxoid change within a conventional leiomyoma, no coagulative tumor cell necrosis, cytologic atypia or mitotic activity
    • Epithelioid leiomyoma:
        No coagulative tumor cell necrosis, cytologic atypia or mitotic activity
  • Smooth muscle tumors of uncertain malignant potential (STUMP):
    • Some but not all criteria are met for uterine leiomyosarcoma:
      • Spindled smooth muscle tumors: focal / multifocal or diffuse cytologic atypia and 2 - 4 mitoses / mm2 (6 - 9 mitoses / 10 high power fields, 0.55 mm field of diameter, 0.24 mm2 in area) but lacking coagulative necrosis; unequivocal coagulative necrosis but lacking cytologic atypia or elevated mitoses; elevated mitoses at > 6 mitoses / mm2 or > 15 mitoses / 10 high power fields (FD = 0.55, 0.24 mm2 in area) but lacking coagulative necrosis or cytologic atypia; diffuse cytologic atypia and uncertain mitotic count, often due to prominent karyorrhexis but lacking coagulative necrosis
      • Epithelioid smooth muscle tumors: epithelioid morphology with 2 - 3 mitoses / 10 high power fields (FD = 0.55, 0.24 mm2 in area) but lacking moderate to severe cytologic atypia and coagulative necrosis
      • Myxoid smooth muscle tumors: myxoid morphology but lacking mitotic activity, moderate to severe cytologic atypia, coagulative necrosis and infiltrative / irregular borders
  • Inflammatory myofibroblastic tumor:
    • Spindle cell neoplasm with myxoid stroma and associated lymphoplasmacytic inflammatory cells
    • Atypia can range from mild to severe
    • ALK staining is seen in most tumors and ALK rearrangement is seen in 80% of cases (Mod Pathol 2017;30:1489)
  • Perivascular epithelioid cell tumor:
    • Composed of epithelioid or spindled cells with eosinophilic to clear cytoplasm with variable cytologic atypia and mitoses
    • Cells may be organized in a perivascular fashion, short fascicles, sheets or nests
    • Melanin rarely present
    • Typically strongly HMB45 positive and often MelanA or MITF positive
    • Cathepsin K staining
    • A subset harbor TSC1 and TSC2 alterations or TFE3 fusion (Mod Pathol 2022;35:515)
Board review style question #1

What mitotic count per 10 high power fields is used to aid in diagnosing spindle cell leiomyosarcoma of the uterus?

  1. > 1 mitosis per 10 high power fields
  2. ≥ 5 mitoses per 10 high power fields
  3. ≥ 10 mitoses per 10 high power fields
  4. Not a diagnostic criterion
Board review style answer #1
C. ≥ 10 mitoses per 10 high power fields. Uterine spindle cell leiomyosarcomas are diagnosed based on the presence of 2 of 3 morphologic features: a mitotic count of ≥ 10 mitoses per 10 high power fields, moderate to marked cytologic atypia and coagulative tumor cell necrosis.

Comment Here

Reference: Leiomyosarcoma
Board review style question #2

A 62 year old woman underwent a hysterectomy for a uterine mass. Immunohistochemical stains show that the tumor is positive for h-caldesmon and desmin and negative for CD10 and ALK. What is the most likely diagnosis?

  1. Endometrial stromal sarcoma
  2. High grade endometrial adenocarcinoma
  3. Inflammatory myofibroblastic tumor
  4. Leiomyosarcoma, myxoid subtype
Board review style answer #2
D. Leiomyosarcoma, myxoid subtype. The image shows a myxoid mesenchymal tumor with infiltrative borders. The morphologic findings combined with the immunophenotype is consistent with leiomyosarcoma, myxoid subtype.

Comment Here

Reference: Leiomyosarcoma

Low grade endometrial stromal sarcoma
Definition / general
  • Malignant mesenchymal tumor comprised of cells resembling proliferative phase endometrial stroma with infiltrative growth or lymphovascular invasion
Essential features
  • Histologic features include permeative tongue-like islands of tumor cells composed of monotonous oval to spindle cells with minimal cytologic atypia, often demonstrating whorling around blood vessels; smooth muscle and sex cord-like differentiation are common
  • Diagnosis may require extensive sampling of the tumor myometrial interface to evaluate for invasion and exclude endometrial stromal nodule
  • Recurrent rearrangements involving JAZF1 and PHF1 are common, though absence does not preclude the diagnosis
Terminology
  • Endolymphatic stromal myosis
ICD coding
  • ICD-O: 8931/3 - endometrial stromal sarcoma, low grade
  • ICD-11: 2B5C & XH1S94 - endometrial stromal sarcoma, primary site and endometrial stromal sarcoma, low grade
Epidemiology
Sites
  • Uterus: more commonly the corpus than the cervix
  • Rarely extrauterine, usually associated with endometriosis
Clinical features
  • Abnormal uterine bleeding (most common), pelvic pain, uterine mass
  • Occasionally, patients present with metastases (most commonly lung, adnexal or nodal) (Adv Anat Pathol 2000;7:257)
Diagnosis
  • Radiologic findings on magnetic resonance imaging (MRI) (see radiology description)
  • Endometrial sampling may obtain diagnostic material in ~30%
Radiology description
Radiology images

Images hosted on other servers:

T2 and T1 weighted images

MRI of low grade endometrial stromal sarcoma

Prognostic factors
  • Typically indolent, with an overall 5 year survival of > 90% and a 10 year survival of 75%
  • Factor affecting prognosis: stage
  • Controversial factors: older age (> 50 years), mitotic index, necrosis (Oncology 2006;71:333)
  • Factors of uncertain significance: tumor size, lymphovascular invasion, hormonal status, ploidy (Adv Anat Pathol 2010;17:113)
Case reports
Treatment
Gross description
  • Poorly circumscribed soft yellow-tan to white nodules extending from the endometrium and invading into the myometrium
  • Worm-like plugs of tumor may be seen in the myometrium or lymphovascular channels (Am J Surg Pathol 1990;14:415)
  • Often with a polypoid endometrial component
  • May appear deceptively well circumscribed: extensive sampling of the tumor myometrial interface is necessary to rule out endometrial stromal nodule (Int J Gynecol Pathol 2014;33:374)
  • Hemorrhage and necrosis may be seen
Gross images

Contributed by Elizabeth Kertowidjojo, M.D., Ph.D., M.P.H. and Ayse Ayhan, M.D., Ph.D.
Tan-yellow uterine mass

Tan-yellow uterine mass

Fleshy lobulated mass

Fleshy lobulated mass

Missing Image

Multinodular, tan to yellow mass



Images hosted on other servers:

Yellow-brown
tumor mass and
additional infiltrating
tumor nodules

Macroscopically,
sarcoma resembles
uterine leiomyomas

Yellow tumor nodule with necrosis

Frozen section description
  • Spindle to oval cells with permeative growth into the myometrium
  • May be associated with a polypoid growth
  • Minimal cytologic atypia and low mitotic count
  • May display smooth muscle or sex cord-like differentiation
Frozen section images

Contributed by Elizabeth Kertowidjojo, M.D., Ph.D., M.P.H.
Polypoid mass

Polypoid mass

Low grade spindle cells

Low grade spindle cells

Infiltrative mass

Infiltrative mass

Microscopic (histologic) description
  • Irregular cellular islands, forming permeative tongue-like pattern of myometrial invasion with frequent vascular invasion
  • Monotonous oval to spindle cells with minimal cytologic atypia, vesicular chromatin and scant cytoplasm
  • Mitotic count is usually low (< 5/10 high power fields), necrosis is usually absent
  • Tumor cells may whorl around delicate arteriolar type vessels, reminiscent of proliferative phase endometrial stroma
  • May have admixed collagen bands / plaques and foamy histiocytes
  • May have smooth muscle differentiation, particularly in a starburst morphology, with collagen bands radiating towards the periphery of the nodule
  • Other reported types of differentiation: fibromyxoid / fibrous, sex cord-like, epithelioid, rhabdoid, endometrioid glands, pseudopapillae, clear cells, bizarre cells, adipose tissue (Mod Pathol 2016;29:S92)
Microscopic (histologic) images

Contributed by Elizabeth Kertowidjojo, M.D., Ph.D., M.P.H. and Ayse Ayhan, M.D., Ph.D.
Tongue-like invasion

Tongue-like invasion

Monotonous tumor cells

Monotonous tumor cells

Whorling around vasculature

Whorling around vasculature

Smooth muscle differentiation

Smooth muscle differentiation

Sex cord-like growth

Sex cord-like growth


Endometrial curetting Endometrial curetting

Endometrial curetting

Missing Image

Mild nuclear atypia

Missing Image

Small blue cells, scant cytoplasm

Missing Image

Unusual features in endometrial stromal sarcoma


Desmin

Desmin

Positive for CD10

Positive for CD10

Missing Image Missing Image

CD10

Missing Image

Estrogen receptor (alpha isoform)

Missing Image

Progesterone receptor

Cytology description
  • Resembles benign endometrial stromal cells
  • Moderate to marked cellularity composed of single cells and clusters of bland cells with scant cytoplasm, small round to spindle nuclei with fine chromatin (Acta Cytol 2007;51:461)
  • Interspersed delicate blood vessels may be present
  • Distinction between low grade endometrial stromal sarcoma and endometrial stromal nodule cannot be made based on cytology, as it requires evaluation of the tumor myometrium interface
  • Distinction between low grade endometrial stromal sarcoma and other monomorphic spindle cell neoplasm is difficult on cytology, especially without immunohistochemistry
Cytology images

Contributed by Elizabeth Kertowidjojo, M.D., Ph.D., M.P.H.
Lung fine needle aspiration

Lung fine needle aspiration

Lung fine needle aspiration cell block

Lung fine needle aspiration cell block

Positive stains
Negative stains
Molecular / cytogenetics description
Molecular / cytogenetics images

Images hosted on other servers:

JAZF1-SUZ12 dual fusion probe

MBTD1-CXorf67 dual fusion probe

Videos

Endometrial stromal sarcoma

Uterine mesenchymal neoplasms

Sample pathology report
  • Uterus, hysterectomy:
    • Endometrial stromal sarcoma, low grade (see synoptic report)
    • Tumor size: 9 cm
    • Lymphovascular invasion identified
    • Surgical margins uninvolved
Differential diagnosis
  • Endometrial stromal nodule:
    • Absent to minimal myometrial invasion (≤ 3 protrusions, each measuring < 3 mm) and no lymphovascular invasion
  • High grade endometrial stromal sarcoma:
    • Presence of a high grade component with cytologic atypia and elevated mitotic count
    • Frequently positive for cyclin D1 and BCOR while negative for ER
    • YWHAE-NUTM2A/B fusion, BCOR fusions or internal tandem duplication
  • Cellular leiomyoma:
    • Fascicular growth, large thick walled blood vessels, cleft-like spaces
    • Lack JAZF1 or PHF1 fusions
  • Leiomyosarcoma:
    • Marked cytologic atypia, mitotic activity and necrosis
    • Large thick walled blood vessels
    • Lack JAZF1 or PHF1 fusions
  • Uterine tumor resembling ovarian sex cord tumor:
    • Absence of any conventional endometrial stromal component
    • ESR1 or GREB1 fusions in a subset
  • Endometrial polyp:
    • No expansile growth or displacement of adjacent endometrium
  • Gland poor adenomyosis:
    • No confluent growth or displacement of myometrium
Board review style question #1


The uterine tumor illustrated in the figures above is best diagnosed as

  1. Cellular leiomyoma
  2. Endometrial stromal nodule
  3. Endometrioid carcinoma
  4. Low grade endometrial stromal sarcoma
  5. Uterine tumor resembling ovarian sex cord tumor
Board review style answer #1
D. Low grade endometrial stromal sarcoma. The tumor shows a polypoid component, as well as a characteristic permeative tongue-like invasion pattern seen on low power. The presence of invasion rules out an endometrial stromal nodule (answer B) and cellular leiomyoma (answer A). The lack of any glandular component rules out endometrioid carcinoma (answer C). While low grade endometrial stromal sarcoma can have sex cord-like differentiation, the presence of conventional low grade endometrial stromal sarcoma rules out uterine tumor resembling ovarian sex cord tumor (answer E).

Comment Here

Reference: Low grade endometrial stromal sarcoma
Board review style question #2
Which of the following can reliably diagnose low grade endometrial stromal sarcoma?

  1. Endometrial biopsy
  2. Hysterectomy
  3. Myomectomy
  4. Presenting symptoms and clinical history
  5. Ultrasound
Board review style answer #2
B. Hysterectomy. The presenting symptoms and clinical findings in low grade endometrial stromal sarcoma are nonspecific, including abnormal uterine bleeding, pelvic pain and uterine mass. While ultrasound can detect a uterine mass, distinguishing low grade endometrial stromal sarcoma from other entities such as endometrial stromal nodule and leiomyoma can be difficult. While biopsy and myomectomy may obtain diagnostic material, low grade endometrial stromal sarcoma is distinguished from endometrial stromal nodule by its invasive growth pattern, which cannot be appreciated on limited sampling.

Comment Here

Reference: Low grade endometrial stromal sarcoma

Mesonephric-like adenocarcinoma (uterus / ovary)
Definition / general
  • Mesonephric-like adenocarcinoma (MLA) is a rare, recently recognized but frequently misdiagnosed subtype of gynecologic malignancy arising in the ovary and uterine corpus
  • MLA shares morphologic, immunophenotypic and molecular characteristics with mesonephric adenocarcinoma (MA) except that mesonephric remnants are not identified in the former
  • Entity is included in the 5th edition World Health Organization (WHO) classification of female genital tumors
Essential features
  • Rare subtype of gynecologic malignancy with high risk of recurrence and increased tendency to metastasize
  • Morphologic hallmark is the combination of architectural patterns in a tumor without squamous or mucinous differentiation; intraluminal dense eosinophilic secretions can be seen
  • Low grade morphology with negative ER and PR should prompt the pathologist to perform additional stains
  • PAX8, GATA3, TTF1, ER and PR are the main immunohistochemical stains that can be used to support the diagnosis
  • Usually mismatch repair (MMR) proficient and p53 wild type
  • KRAS mutation is commonly present
ICD coding
  • ICD-O: 9111/3 - mesonephric-like adenocarcinoma
  • ICD-11:
    • 2C76.Y - other specified malignant neoplasms of corpus uteri
    • 2C73.Y - other specified malignant neoplasms of the ovary
Epidemiology
Sites
  • Uterine corpus and ovary
Pathophysiology
  • Controversial whether MLA originates from mesonephric structures or represents Müllerian tumors that have acquired mesonephric characteristics
  • Evidence supporting Müllerian origin with mesonephric transdifferentiation
    • Endometrial MLA arises from the endometrium rather than myometrium
    • MLA is not associated with mesonephric remnants
    • Ovarian MLA has been found to coexist with other Müllerian neoplasms such as low grade serous carcinoma (Int J Gynecol Pathol 2020;39:84)
    • MA and MLA share KRAS mutations but concurrent PIK3CA mutations, which are described in endometrioid adenocarcinoma, are also documented in MLA
    • NRAS mutations may occur (Int J Gynecol Pathol 2018;37:448)
Etiology
  • Unknown
Clinical features
  • Patients with endometrial MLA may present with abnormal vaginal bleeding
  • Patients with ovarian MLA may present with pelvic or abdominal pain and may be associated with endometriosis and other benign, borderline and malignant lesions of Müllerian origin (Am J Surg Pathol 2021;45:498)
Diagnosis
  • Definitive diagnosis of MLA requires a surgical specimen
  • Limited literature on the cytologic features of these tumors; may be identified on Pap test as atypical glandular cells (AGC) (Am J Surg Pathol 2021;45:498)
  • MLA of the uterine corpus was correctly diagnosed on initial biopsy in only 32% of the cases in one study (Am J Surg Pathol 2021;45:498)
Laboratory
Prognostic factors
  • Factors associated with development of metastasis in mesonephric adenocarcinoma of the uterine corpus (Am J Surg Pathol 2019;43:12)
    • Large tumor size (> 4 cm)
    • Ill defined tumor border
    • Advanced FIGO stages (III - IV)
    • Presence of coagulative tumor cell necrosis
    • High mitotic activity (> 10/10 high power fields)
    • Lymphovascular invasion (Am J Surg Pathol 2019;43:12)
  • Associated with aggressive clinical course (J Clin Med 2021;10:698)
    • Tends to present with advanced stage (FIGO II or more)
    • Increased risk of recurrent disease
    • Increased tendency to metastasize to the lungs even for tumors with stage I disease
  • Compared with other endometrial adenocarcinomas (Am J Surg Pathol 2021;45:498)
    • Better overall survival than carcinosarcoma and serous carcinoma
    • Equal overall survival to endometrioid grade 3
    • Worse overall survival than endometrioid grade 1 - 2 carcinomas
Case reports
  • 32 year old woman with coexistent endometrial mesonephric-like adenocarcinoma and endometrioid carcinoma (Diagn Pathol 2019;14:54)
  • 58 year old woman with endometrial mesonephric-like adenocarcinoma presenting as an ocular lesion (Int J Gynecol Pathol 2022;41:161)
  • 69 year old woman with synchronous ovarian and uterine mesonephric-like carcinoma that potentially arose from endometrioid adenofibroma (J Obstet Gynaecol Res 2023;49:1052)
  • 70 year old woman with mesonephric-like adenocarcinoma arising from the uterine body and mimicking follicular thyroid carcinoma (Histopathology 2019;74:651)
  • 71 year old woman with corded and hyalinized mesonephric-like adenocarcinoma of the uterine corpus (Hum Pathol 2019;86:243)
Treatment
  • Surgical approach with total hysterectomy and bilateral salpingo-oophorectomy, with or without pelvic and para-aortic lymph node dissection, is the primary therapy
  • Adjuvant chemotherapy and radiation therapy have been used
  • Hormone therapy has been reported in 2 cases (Am J Surg Pathol 2020;44:429)
  • No tumor specific treatment options have been elucidated for MLA
Gross description
  • No distinctive macroscopic appearance compared to other endometrial or ovarian tumors
  • Areas of necrosis and hemorrhage can be present
  • Ovarian MLA is usually unilateral, ranging in size from 4 - 32 cm, with solid or mixed solid cystic, grey-white or yellow-tan appearance (Histopathology 2016;68:1013)
Microscopic (histologic) description
  • Variety of histologic patterns that may be present within the same tumor
    • Most frequently small tubules with ductal / glandular growth
    • Papillary, solid growth, trabecular, retiform, sex cord-like, sieve-like, glomeruloid and spindle cell areas have all been described
  • Luminal eosinophilic secretions are characteristic but not always identified
  • Tumor cells can be flattened, cuboidal or columnar with mild to moderate cytological atypia
    • Clear cell features can be seen but are less common
    • High grade cytological atypia is usually not a predominant feature
  • Nuclei show vesicular chromatin and nuclear grooves
  • Sarcomatoid transformation has been seen in rare instances
  • Squamous, ciliated or mucinous differentiation (metaplasia) are not present and there are no associated mesonephric remnants (J Clin Med 2021;10:698)
Microscopic (histologic) images

Contributed by Daniel Graham, M.D., Adele Wong, M.B., B.Ch., B.A.O. and Lucy Ma, M.D.
Low power morphologic appearance

Glandular growth pattern

Variety of histologic patterns

Variety of histologic patterns

Low power appearance

Variable growth patterns

Medium power appearance Medium power appearance

Glandular morphology

High power

Glands


Multiple growth patterns

Multiple growth patterns

Glandular and cystic areas

Glandular and cystic areas

Micropapillary architecture

Micropapillary architecture

Mitotic activity Mitotic activity

Mitotic activity

Papillary architecture

Papillary architecture


Variable morphology

Variable morphology

PAX8 PAX8

PAX8

GATA3 GATA3

GATA3

GATA3

GATA3


TTF1 TTF1

TTF1

TTF1

TTF1

CD10

CD10

CD10

CD10


CD10

CD10

Estrogen receptor ER

Estrogen receptor

Progesterone receptor

Progesterone receptor

p53

p53

Ki67

Ki67


WT1

WT1

Varied morphology Varied morphology Varied morphology

Varied morphology

PTC-like nuclear features

PTC-like nuclear features

Virtual slides

Images hosted on other servers:
MLA H&E and IHC MLA H&E and IHC

MLA H&E and IHC

Positive stains
  • PAX8: usually diffusely positive
  • GATA3 and TTF1: focal or diffuse with inverse staining pattern described in several studies in the most recent WHO classification; cells positive for GATA3 are negative for TTF1 and vice versa (Am J Surg Pathol 2018;42:1596)
  • CD10: focal and apical / luminal
  • p53 wild type
  • MMR proficient
Negative stains
Molecular / cytogenetics description
  • Majority of MLA shows KRAS mutation, most commonly G12V and G12D (Am J Surg Pathol 2020;44:429)
  • Concurrent ARID1A and PIK3CA mutations are quite common
  • PTEN mutation has been reported
  • CTNNB1 hotspot mutations have been reported
  • Copy number analysis: copy number gain of 1q and 10 are the most common, some have 1p loss (Mod Pathol 2021;34:1570)
  • Usually does not have the common molecular abnormalities described in endometrioid (p53, MMR and POLE) (Mod Pathol 2021;34:1570)
  • NRAS mutations may occur (Int J Gynecol Pathol 2018;37:448)
Videos

Mesonephric-like adenocarcinoma by Dr. Lewis Hassell

Sample pathology report
  • Uterus and cervix, bilateral fallopian tubes and ovaries, total hysterectomy and bilateral salpingo-oophorectomy:
    • Endometrial adenocarcinoma, most consistent with mesonephric-like adenocarcinoma (see comment)
    • Comment: The tumor exhibits various growth patterns including small tubular, glandular and papillary areas. Tumor cells show diffuse immunoreactivity for PAX8 and focal immunoreactivity for GATA3 and TTF1. Despite the predominant glandular architecture and low grade appearance, the tumor is negative for ER and PR. The cervix is benign and mesonephric remnants are not identified. All these findings are most consistent with endometrial adenocarcinoma, mesonephric-like subtype.
Differential diagnosis
Board review style question #1

A 55 year old woman was recently diagnosed with endometrioid intraepithelial neoplasia on endometrial curettage. The entire endometrium is submitted for histologic evaluation. Histologic details from one section are shown in the image above. The tumor cells are diffusely positive for GATA3 and PAX8 and focally positive for TTF1. ER and PR are negative. Which of the following statements is true regarding this entity?

  1. KRAS mutations, particularly G12V and G12D, are commonly identified
  2. MMR deficiency is present in half of the cases
  3. The tumor frequently shows p53 overexpression
  4. The tumor has better prognosis compared to low grade endometrioid adenocarcinoma
Board review style answer #1
A. KRAS mutations, particularly G12V and G12D, are commonly identified. Despite the low grade morphology commonly encountered in mesonephric-like adenocarcinoma, the tumor has worse prognosis compared to low grade endometrioid adenocarcinoma. They do not show abnormal p53 expression and are usually MMR proficient. KRAS mutations are commonly seen, with G12V and G12D being the most common alterations identified.

Comment Here

Reference: Mesonephric-like adenocarcinoma
Board review style question #2
Which of the following statements is true regarding mesonephric-like adenocarcinoma occurring in the uterus?

  1. The tumor commonly expresses GATA3 and TTF1 immunohistochemically
  2. The tumor expresses hormone receptors similar to those seen in conventional endometrioid carcinoma
  3. The tumor is most often seen in the myometrium without a surface connection to the endometrium
  4. The tumor often shows DNA polymerase epsilon (POLE) mutation
Board review style answer #2
A. The tumor commonly expresses GATA3 and TTF1 immunohistochemically. The finding of a hormone receptor negative, PAX8, GATA3 and TTF1 positive tumor is highly supportive of the diagnosis of mesonephric-like adenocarcinoma. While mesonephric remnants and mesonephric derived tumors are often found in paracervical or adnexal locations, this is not true of the mesonephric-like tumors in the endometrium, which have a surface component in virtually all cases. Most such tumors are of no specific molecular type.

Comment Here

Reference: Mesonephric-like adenocarcinoma

POLE ultramutated endometrial carcinoma
Definition / general
  • Clinically significant, molecularly defined subclass of endometrial carcinoma with favorable prognosis
  • Endometrial carcinoma harboring pathogenic mutations in the exonuclease domain of the POLE gene
Essential features
  • Carries excellent prognosis compared to the other molecular subgroups, as defined by The Cancer Genome Atlas (TCGA) program
  • Detected POLE mutation must be pathogenic, involving the exonuclease domain and correlated with ultramutated phenotype
  • Predominantly endometrioid or mixed morphology but is represented across all endometrial carcinoma histotypes
Terminology
  • POLE mutant endometrial carcinoma
ICD coding
  • ICD-O: 8380/3 - endometrioid adenocarcinoma, NOS
  • ICD-10: C54.1 - malignant neoplasm of endometrium
  • ICD-11:
    • 2C76 - malignant neoplasms of corpus uteri
    • 2C76.Y - other specified malignant neoplasms of corpus uteri
Epidemiology
Sites
Pathophysiology
  • TCGA Research Network identified 4 molecularly distinct endometrial carcinoma subgroups with different clinical outcomes (Nature 2013;497:67):
    1. POLE ultramutated (POLEmut), ultramutated
      • Pathogenic POLE exonuclease domain mutations
    2. Mismatch repair deficient (MMRd), hypermutated
      • Abnormal expression of mismatch repair proteins by IHC, highly concordant with MSI H status
    3. p53 abnormal (p53abn), copy number high; also referred to as serous-like
      • Aberrant p53 expression by IHC, frequent TP53 mutations
    4. No specific molecular profile (NSMP), copy number low
      • Pathogenic POLE mutation absent, MMR proficient and p53 nonaberrant
  • POLE gene encodes the catalytic subunit of DNA polymerase epsilon
    • DNA polymerase epsilon synthesizes DNA of the leading strand and plays a major role in proofreading (e.g., nucleotide and base excision repair) (Nat Rev Cancer 2016;16:71)
    • Defects in exonuclease domain of POLE → ultramutational state / high tumor mutational burden → high neoantigen loads (immunogenic) → associated with increased tumor infiltrating lymphocytes / peritumoral lymphocytes and elevated PD-1 / PDL1 expression (JAMA Oncol 2015;1:1319)
      • Enhanced T cell antitumor response may play a role in the observed favorable prognosis
Diagrams / tables

Contributed by Carlos Parra-Herran, M.D.
Molecular based classification of endometrial carcinoma

Molecular based classification of endometrial carcinoma



Images hosted on other servers:
<i>POLE</i>mut endometrial carcinoma cases have excellent survival outcomes

POLEmut cases have excellent survival outcomes

Diagnosis
  • Detection of pathogenic POLE exonuclease domain mutation
  • Accurate molecular classification of endometrial carcinoma requires interpretation of tests according to the recommended algorithm (see Diagram 1) (Int J Gynecol Pathol 2021;40:5)
  • Presence of pathogenic POLE exonuclease domain mutation, MMR proficient, p53 nonaberrant / wild type
  • ~3 - 5% of endometrial carcinomas harbor > 1 molecular classifying alteration and are referred to as multiple classifier (J Pathol 2020;250:312)
    • p53 alterations in the presence of POLE pathogenic mutation or MMR deficiency are likely secondary events acquired in tumor progression (passenger mutations) and are typically subclonal
    • Hierarchical clustering analyses support classification of multiple classifier endometrial carcinomas as the following single final molecular classification:
      • Pathogenic POLE exonuclease domain mutation, p53 abnormal tumors → classified as POLEmut
      • MMR deficient, p53 abnormal tumors → classified as MMRd
    • Multiple classifier tumors with POLE mutations and MMR deficiency also observed and final classification is dependent on which alteration is primary or secondary event (Mod Pathol 2022;35:688)
      • Pathogenic POLE exonuclease domain mutation, MMR deficient (subclonal) tumors → classified as POLEmut
      • Nonpathogenic POLE mutation, MMR deficient tumor → classified as MMRd
Prognostic factors
  • Based on current literature, shows excellent prognosis as compared to MMRd, p53abn and NSMP endometrial carcinomas (see Diagram 2)
    • Order of most favorable to worst prognosis: POLEmut > MMRd > NSMP > p53abn
  • Associated with early stage disease
  • Favorable prognosis is independent of tumor histotype and grade (Am J Surg Pathol 2018;42:561)
  • Unclear whether observed favorable outcome may be due to enhanced response to adjuvant therapy (chemotherapy)
Case reports
Treatment
  • Surgery with or without adjuvant therapy (vaginal brachytherapy, pelvic radiotherapy and chemotherapy, depending on stage)
    • Ongoing clinical trials (e.g., PORTEC-4a) are investigating the potential for treatment de-escalation in high - intermediate risk group patients (Int J Gynecol Cancer 2020;30:2002)
  • Immune checkpoint inhibitor therapy in advanced stage or recurrent disease (JAMA Oncol 2015;1:1319)
Microscopic (histologic) description
Microscopic (histologic) images

Contributed by Lucy Ma, M.D.
Peritumoral lymphocytes

Peritumoral lymphocytes

Tumor infiltrating lymphocytes

Tumor infiltrating lymphocytes

Bizarre atypia Bizarre atypia

Bizarre atypia


Intratumoral heterogeneity Intratumoral heterogeneity Intratumoral heterogeneity

Intratumoral heterogeneity

Endometrioid histotype

Endometrioid histotype

Positive stains
Negative stains
  • p53 nonaberrant / wild type but may be aberrant / mutant in a diffuse or subclonal pattern in multiple classifier tumors (~3 - 5%)
Molecular / cytogenetics description
  • Majority of pathogenic POLE mutations are within exons 9, 11, 13 and 14, which encode the proofreading exonculease domain
    • 5 recurrent POLE exonuclease domain mutation variants account for the majority of POLEmut cases: P286R, V411L, S297F, A456P and S459F
  • Characteristic genomic signatures:
    • High prevalence of C → A substitutions
    • Low proportion of small insertion and deletion mutations (indels)
    • High tumor mutational burden (TMB: > 100 mut/Mb)
  • Diagnostic interpretation (i.e., evaluation of pathogenicity of novel POLE mutations may be guided by POLE score) (J Pathol 2020;250:323)
Sample pathology report
  • Endometrium, curettage:
    • High grade endometrial carcinoma (see comment)
    • Comment: Sections show fragments of endometrial carcinoma with varied morphology, including areas with endometrioid and focal serous differentiation. The tumor shows complex glandular, solid and focal papillary architecture. The lining cells show predominantly high grade cytologic atypia, characterized by enlarged hyperchromatic nuclei with high nuclear to cytoplasmic ratio, loss of polarity and increased mitotic activity. Scattered markedly atypical bizarre cells are also noted. Immunohistochemical stains show that the tumor cells are positive for ER (80%, variable intensity) and PR (20%, weak). p53 shows predominantly nonaberrant (wild type) expression but a small subset of tumor cells show aberrant p53 overexpression. Mismatch repair protein (MLH1, PMS2, MSH2 and MSH6) expression is intact (retained / normal). POLE mutational analysis by Sanger sequencing has been requested and the result, along with the final molecular classification of this tumor, will be issued in an addendum report.
    • Addendum: final molecular classification: POLE ultramutated (pathogenic POLE mutation detected - see molecular report #. Mismatch repair protein expression is intact. p53 shows subclonal aberrant overexpression.)

  • Uterus, fallopian tubes and ovaries, total hysterectomy and bilateral salpingo-oophorectomy:
    • Endometrial endometrioid carcinoma, FIGO grade 2; final molecular classification: POLE ultramutated (see synoptic report and comment)
      • Tumor invades the inner half of the myometrium (< 50%)
      • Lymphovascular invasion is absent
      • Cervix, uterine serosa, bilateral fallopian tubes and ovaries are uninvolved
    • Comment: Pathogenic POLE mutation detected (see molecular report #). DNA mismatch repair protein (MLH1, PMS2, MSH2 and MSH6) expression is intact. p53 expression is nonaberrant (wild type).
Differential diagnosis
Board review style question #1

Shown above are representative images from an endometrial tumor in a 60 year woman. The tumor shows intact nuclear PMS2 and MSH6 expression, as well as p53 mutant overexpression by immunohistochemistry (seen in image 3). What additional test is required for accurate molecular classification of this endometrial cancer?

  1. ARID1A immunohistochemistry
  2. HER2 gene amplification by FISH
  3. p16 immunohistochemistry
  4. POLE exonuclease domain mutation analysis
  5. Promoter hypermethylation testing of MLH1
Board review style answer #1
D. POLE exonuclease domain mutation analysis. TCGA defined 4 molecular and prognostic subgroups of endometrial carcinoma: POLEmut, MMRd, p53abn and NSMP. Accurate molecular classification relies on correct interpretation of tests following the recommended algorithm (see Diagram 1).

Comment Here

Reference: POLE ultramutated endometrial carcinoma
Board review style question #2
Which of the following molecular subgroups of endometrial carcinoma confers the most favorable prognosis?

  1. MMR deficient
  2. No specific molecular profile
  3. p53 abnormal
  4. POLE ultramutated
Board review style answer #2

Perivascular epithelioid cell tumor (PEComa)
Definition / general
  • Rare mesenchymal neoplasm with myomelanocytic differentiation that is favored to arise from perivascular epithelioid cells
Essential features
  • Best classified as a tumor of uncertain malignant potential since recurrences may arise years after initial diagnosis
  • Characterized by sheets or nests of epithelioid cells with clear to eosinophilic and granular cytoplasm, surrounded by delicate vasculature
  • Coexpresses melanoma and smooth muscle markers, with variable staining intensity and distribution
  • 2 main molecular subgroups: TSC1 / TSC2 alterations, TFE3 fusions
ICD coding
  • ICD-10:
    • D39.0 - neoplasm of uncertain behavior of uterus
    • C54.9 - malignant neoplasm of corpus uteri, unspecified
  • ICD-11
    • 2F76 - neoplasms of uncertain behaviour of female genital organs
    • 2C76.Z - malignant neoplasms of corpus uteri, unspecified
Epidemiology
Sites
  • Uterine corpus is most common in gynecologic tract
  • Rare in cervix, vagina, ovary, broad ligament and vulva
Pathophysiology
Etiology
Clinical features
Diagnosis
  • Rarely made on endometrial sampling
  • Myomectomy or hysterectomy
Radiology description
  • No defining features to distinguish from other uterine mesenchymal neoplasms on imaging
Radiology images

Images hosted on other servers:
Sagittal T2 with heterogeneous hyperintense mass (arrows)

Sagittal T2 with heterogeneous hyperintense mass

MRI with hyperintense mass

MRI with hyperintense mass

Prognostic factors
  • 3 main algorithms have been proposed for predicting behavior
    • Nongynecologic specific criteria (Am J Surg Pathol 2005;29:1558):
      • Benign: < 5 cm, noninfiltrative, no high grade atypia, mitoses ≤ 1/50 high power fields (HPFs), no necrosis, no lymphovascular invasion (LVI)
      • Uncertain malignant potential: nuclear pleomorphism / multinucleated giant cells or > 5 cm
      • Malignant: 2+ features (> 5 cm, noninfiltrative, high grade atypia, mitoses > 1/50 HPFs, necrosis, LVI)
    • Gynecologic specific criteria (Am J Surg Pathol 2014;38:176):
      • Benign / uncertain malignant potential: < 4 features (≥ 5 cm, high grade atypia, mitoses > 1/50 HPFs, necrosis, LVI)
      • Malignant: ≥ 4 features
    • Modified gynecologic specific criteria (adopted by the WHO 2020) (Am J Surg Pathol 2018;42:1370)
      • Uncertain malignant potential: < 3 features (> 5 cm, high grade atypia, mitoses > 1/50 HPFs, necrosis, LVI)
      • Malignant: ≥ 3 features
  • Recurrences can occur years after original diagnosis
Case reports
Treatment
Gross description
  • Mean: 6.5 cm; range: 0.2 - 25 cm (Genes Chromosomes Cancer 2021;60:168)
  • Typically intramural but may be polypoid
  • Tan, white, pink or gray solid mass
  • Subset show hemorrhage or necrosis
Gross images

Images hosted on other servers:
Posterior uterine mass

Posterior uterine mass

PEComa

PEComa

Microscopic (histologic) description
  • Well circumscribed or infiltrative growth
  • Numerous growth patterns, with sheets and nests being the most common
  • Noncohesive epithelioid cells with clear to eosinophilic granular cytoplasm
    • May have a component of spindled cells (usually minor)
    • Subset have dense eosinophilic (rhabdoid) or foamy cytoplasm
  • Variable cytologic atypia and mitotic index
  • Melanoma-like nucleoli, intranuclear pseudoinclusions, multinucleated cells, Touton giant cells and melanin pigment (rare) may be present
  • Characterized by thin and delicate vessels but may also have thick walled (generally peripherally located) and staghorn vessels
  • Radial / perivascular distribution of tumor cells identified in < 25% (Am J Surg Pathol 2018;42:1370)
  • Stromal hyalinization is common
  • TFE3 translocation associated PEComas (Am J Surg Pathol 2010;34:1395, Am J Surg Pathol 2015;39:394):
    • Typically epithelioid with clear cells in a nested / alveolar growth
    • Low grade atypia and rare mitoses
  • Lymphangioleiomyomatosis (LAM)-like PEComas are rare, predominantly spindled with thick walled blood vessels, cleft / slit-like spaces, low grade atypia and infrequent mitoses (Am J Surg Pathol 2018;42:1370)
Microscopic (histologic) images

Contributed by Jennifer Bennett, M.D.
Missing Image Missing Image

Diffuse growth of epithelioid cells

Missing Image

Infiltrative growth

Missing Image

Delicate vasculature

Missing Image

Spindled cells


Missing Image

Stromal hyalinization

Missing Image

Multinucleated giant cells

Missing Image

Intranuclear pseudoinclusions

Missing Image

Melanoma-like macronucleoli

Missing Image

TFE3 associated PEComa


Missing Image

LAM-like PEComa

Missing Image

HMB45

Missing Image

MelanA / MART1

Missing Image

Desmin

Missing Image

Cathepsin K

Virtual slides

Contributed by Stephanie L. Skala, M.D.
PEComa

PEComa

Positive stains
Negative stains
Molecular / cytogenetics description
Sample pathology report
  • Uterus, hysterectomy:
    • PEComa of uncertain malignant potential (4.5 cm) (see comment)
    • Chronic cervicitis
    • Proliferative endometrium
    • Leiomyomas
    • Comment: 1 of the myometrial nodules is comprised of sheets of epithelioid cells with clear to eosinophilic and granular cytoplasm. Cytologic atypia, mitoses and necrosis are not appreciated. This nodule is positive for HMB45 (50%, strong), MelanA (focal), desmin, caldesmon and SMA. The morphologic and immunoprofile supports the diagnosis of PEComa. Based on the WHO 2020 criteria for predicting behavior in PEComas, this tumor would classify as being of uncertain malignant potential. Most tumors in this category have a benign clinical course; however, as a small subset recur, close clinicoradiological follow up is recommended.
Differential diagnosis
Board review style question #1
A 32 year old woman with a history of pulmonary lymphangioleiomyomatosis, seizures and angiofibromas presents with a myometrial mass. Which molecular / cytogenetic abnormality does this patient likely have?

  1. JAZF1-SUZ12 fusion
  2. RAD51B fusion
  3. TFE3 fusion
  4. TP53 mutation
  5. TSC2 mutation
Board review style answer #1
Board review style question #2

A 50 year old woman presents with a myometrial mass with lung metastases, as seen in the image. The myometrial tumor is characterized by noncohesive epithelioid and spindled cells surrounded by a delicate vasculature. The tumor cells are strongly and diffusely positive for HMB45, desmin, SMA and caldesmon with focal MelanA expression. What type of tumor does this likely represent?

  1. Alveolar soft part sarcoma
  2. Leiomyosarcoma
  3. Low grade endometrial stomal sarcoma
  4. Melanoma
  5. Perivascular epithelioid cell tumor (PEComa)
Board review style answer #2
E. Perivascular epithelioid cell tumor (PEComa)

Comment Here

Reference: Perivascular epithelioid cell tumor (PEComa)

Progestin therapy related changes
Definition / general
  • Hysterectomy is the standard treatment for endometrioid intraepithelial neoplasia (atypical hyperplasia) and low grade endometrial endometrioid adenocarcinoma
  • Hormonal therapy is a valid alternative for premenopausal women who desire to preserve fertility, as well as patients in which surgical treatment is not feasible (those with obesity or multiple other comorbidities)
Essential features
  • Fertility sparing treatment is increasingly considered in the management of patients with endometrioid intraepithelial neoplasia (endometrioid intraepithelial neoplasia [EIN], atypical endometrial hyperplasia) and low grade endometrial endometrioid carcinoma
  • Management recommendations include follow up samples, which determine whether the lesion is responding to progestin and to what degree
  • Pathologists need to be aware of the changes seen in the neoplasm and background endometrium, as well as the terminology currently recommended to report such changes
ICD coding
  • ICD-10:
    • N85.00 - endometrial hyperplasia, unspecified
    • N85.02 - endometrioid intraepithelial neoplasia (EIN)
    • C54.1 - malignant neoplasm of endometrium
    • Z15.04 - genetic susceptibility to malignant neoplasm of endometrium
Epidemiology
  • Proportion of premenopausal women with these diagnoses is increasing: 15 - 25% are premenopausal, 10% are < 45 years, 4% are < 40 years (Int J Womens Health 2014;6:691)
  • In these patients and those in which surgical management is not advisable, fertility sparing treatment is indicated
Diagrams / tables

Images hosted on other servers:

EIN / carcinoma remission probability

EIN / carcinoma recurrence probability

Clinical features
  • Indications for fertility sparing treatment
    • Young woman with high desire to retain reproductive capabilities
    • Diagnosis of endometrioid intraepithelial neoplasia / atypical hyperplasia or FIGO grade 1 endometrial endometrioid adenocarcinoma
    • No myometrial invasion on imaging (MRI)
    • The following patients are generally not eligible for fertility sparing treatment but can be considered on a case to case basis: patients with obesity, anovulation, grade 1 endometrioid carcinoma with superficial myoinvasion on imaging, nonmyoinvasive grade 2 endometrioid carcinoma
  • Treatment modalities
    • 2 main types of fertility sparing treatment are oral progestins and a levonorgestrel releasing intrauterine device
    • Current recommendations for oral progestins include medroxyprogesterone acetate at 400 - 600 mg/day or megestrol acetate at 160 - 320 mg/day for at least 6 months, with follow up using dilation and curettage and imaging (Ann Oncol 2016;27:16, Int J Gynecol Cancer 2015;25:1258)
    • Levonorgestrel releasing intrauterine device releases 52 mg of intrauterine progestin at a consistent rate for up to 5 years but declines thereafter (Obstet Gynecol Sci 2020;63:417)
  • Limitations
    • 10 - 30% of patients present at advanced stage (FIGO stage III - IV), usually with ovarian / adnexal metastases, which may be only detected on hysterectomy / bilateral salpingo-oopherectomy (BSO) but not with progestin treatment (Int J Womens Health 2014;6:691)
    • Moreover, 5 - 29% of premenopausal women have a synchronous ovarian carcinoma, which may be only detected on hysterectomy / BSO but not with progestin treatment (Obstet Gynecol 2005;106:693)
Diagnosis
  • Initial and follow up diagnosis in the setting of endometrioid intraepithelial neoplasia / endometrioid carcinoma treated with progestins require endometrial sampling (dilation and curettage preferred over pipelle biopsy)
  • In principle, the histopathologic diagnosis of endometrioid intraepithelial neoplasia / atypical hyperplasia and endometrial endometrioid carcinoma in the setting of exogenous progestin therapy relies on the architectural and cytologic criteria used conventionally but with the caveat that the cytologic appearance is influenced / confounded by the progestin effect
  • In this context, a lesion may appear cytologically bland but if it meets criteria for endometrioid intraepithelial neoplasia or endometrioid carcinoma, the diagnosis should be rendered as such
  • If criteria are not met but there is gland crowding suggestive of residual neoplasia, descriptive terminology should be used (see Sample pathology report)
  • Morphology of the background normal endometrium should always be reported, as the presence of progestin related changes indicates circulating progestins and adherence to the treatment; conversely, the absence of these changes (for instance, proliferative or inactive endometrium) may alert the treating clinician to the possibility of unsuccessful treatment or lack of patient adherence to the therapy
  • Reference: Int J Gynecol Pathol 2022;41:142
Prognostic factors
  • A meta analysis of 24 studies involving 370 patients undergoing fertility sparing treatment showed that the remission probability ascended to 72% after 6 months of treatment and then plateaued with 78% and 81% remission chance after 12 months and 24 months of treatment, respectively (Fertil Steril 2014;101:785)
  • Conversely, the recurrence probability was estimated at 10%, 17% and 29% at 6, 12 and 24 months, respectively, based on 100 patients in 10 studies; this data suggests that therapy beyond 6 months confers only marginal benefit compared to the steadily increasing risk of lesion recurrence or progression (Fertil Steril 2014;101:785)
  • The following factors have been associated with higher likelihood of remission under progestin therapy: young age, previous pregnancy, infertility, index diagnosis of endometrioid intraepithelial neoplasia / atypical hyperplasia (versus endometrioid carcinoma) and treatment with megestrol acetate (versus other forms of treatment) (Fertil Steril 2014;101:785)
  • In a meta analysis that included 351 patients from 22 studies, 32% of patients with fertility sparing treatment had at least one pregnancy, 54% of those using assisted reproductive technologies (Fertil Steril 2014;101:785)
  • In previous literature, pregnancy was achieved in 35% of patients
Gross description
  • Progestin induced changes in endometrioid intraepithelial neoplasia / atypical hyperplasia or endometrioid carcinoma do not have macroscopic correlates
Frozen section description
  • Intraoperative consultation with frozen section is not indicated in the setting of endometrioid intraepithelial neoplasia / atypical hyperplasia or endometrial endometrioid carcinoma treated with progestins
Microscopic (histologic) description
  • Progestin therapy related changes in the neoplastic endometrium include (Am J Surg Pathol 2007;31:988, Am J Clin Pathol 2012;138:524, Gynecol Oncol 2014;132:33)
    • Architectural changes
      • Decreased volume of disease (% and number of involved fragments)
      • Decreased glandular crowding
      • Low to absent nuclear stratification
      • Decreased cellularity (associated with complete response)
    • Cytologic changes
      • Decreased nuclear to cytoplasmic ratio
      • Decreased nuclear size
      • Cytoplasmic eosinophilia
      • Nuclear rounding
    • Metaplasia (secretory, squamous, mucinous)
  • Progestin related glandular and stromal changes in the background benign endometrium
    • Their presence is an indicator of patient compliance with the treatment
    • Conversely, their absence suggests a lack of patient adherence (in case of oral progestins) or malfunction (in case of intrauterine device)
  • Diagnosis of the degree of response to progestin therapy
    • A minimum of 6 months of therapy has been recommended based on the likelihood of persistence or progression of the neoplastic lesion while on progestin; after 6 months, therapy appears to confer only marginal benefit (Am J Surg Pathol 2007;31:988, Fertil Steril 2014;101:785)
      • Within the first 6 months of progestin therapy, persistence of architectural complexity is allowed as it does not seem to adversely impact the outcome
      • However, architectural complexity and cytologic atypia that was identified at the 6 month evaluation point or later correlated with adverse overall outcome (persistence or progression of endometrioid neoplasia)
  • Diagnostic workup of follow up endometrial samples in patients with fertility sparing treatment
    • Document the time interval between initial diagnosis and follow up
    • Compare initial and follow up samples (if possible)
    • Determine the degree of response (see chart below)
    • Determine the status of the background benign endometrium (with or without progestin therapy related changes)
  • Endometrioid intraepithelial neoplasia is now equivalent to the term atypical endometrial hyperplasia
  • Response to fertility sparing treatment can be framed in 4 categories: resolution, regression, persistence or progression
  • Definition of each term depends on the index diagnosis (pretreatment)

    Initial (pretreatment) diagnosis
    Endometrioid intraepithelial neoplasia / atypical hyperplasia FIGO grade 1 endometrioid carcinoma
    Resolution Negative for residual hyperplasia or carcinoma Negative for residual hyperplasia or carcinoma
    Regression Gland crowding or hyperplasia with progestin treatment effect Endometrial hyperplasia with or without progestin treatment effect
    Persistence Atypical endometrial hyperplasia with no progestin treatment effect FIGO 1 endometrioid carcinoma with or without progestin treatment effect
    Progression Endometrial endometrioid adenocarcinoma FIGO 2 or 3 endometrioid carcinoma

  • Following a recent survey of practicing pathologists, the following diagnostic terminology and definitions have been proposed by Ganesan et al. (table adapted from Int J Gynecol Pathol 2022;41:142)

    Category Glandular morphology Stromal morphology Treatment implications
    Negative for residual hyperplasia / carcinoma Well separated, inactive glands
    Minor glandular irregularity or dilatation
    Focal nuclear enlargement that appears reactive or degenerative
    No cytologic atypia
    Stromal decidual change Treatment has been effective and can end
    Follow up to ensure that there is no recurrence
    Residual hyperplasia with treatment effects Foci of crowded glands or papillary architecture
    Squamous metaplasia may be prominent
    No cytologic atypia
    Stromal decidual change Treatment is working but further treatment is needed (at least 6 months in total based on most authors)*
    Residual atypical hyperplasia with treatment effects Foci of crowded glands or papillary architecture
    Cytologic atypia present
    Stromal decidual change Treatment is working but further treatment is needed (at least 6 months in total based on most authors)*
    Atypical hyperplasia / endometrioid carcinoma without treatment effects Atypical hyperplasia / endometrioid carcinoma without stromal decidual change No stromal decidual change at the site of gland crowding The atypical hyperplasia / endometrioid carcinoma is not responding to progesterone treatment*
    None of the above (describe changes)
    *Treatment regimen should last at least 6 months; the likelihood of response after 6 months sharply decreases and at this point definitive treatment (e.g., hysterectomy) should be considered
Microscopic (histologic) images

Contributed by Carlos Parra-Herran, M.D.
Initial biopsy, low power Initial biopsy, high power

Initial biopsy

3 month follow up, low power 3 month follow up, high power

3 month follow up

6 month follow up, high power 6 month follow up, high power

6 month follow up

Positive stains
Negative stains
Molecular / cytogenetics description
  • Identifying genes and molecular signatures predictive of response to hormonal treatment is the subject of active research
  • A recent study reported differences in gene expression between responder and nonresponder lesions and predictive models using 9 genes (FOXO1, IRS2, PDGFC, DIO2, SOX9, BCL11A, APOE, FYN and KLF4) were shown to have > 90% predictive accuracy (Front Genet 2022;13:952083)
Videos

PathCast lecture on benign endometrial pathology (May 2020)

Sample pathology report
  • Endometrium, biopsy / curettage:
    • Endometrioid intraepithelial neoplasia / atypical hyperplasia / endometrioid carcinoma without treatment effects (see comment)
    • Endometrium with changes consistent with exogenous progestin therapy
    • Comment: Neoplastic glands do not show cytologic changes expected with exogenous progestin treatment.

  • Endometrium, biopsy / curettage:
    • Endometrioid intraepithelial neoplasia / atypical hyperplasia / endometrioid carcinoma without treatment effects (see comment)
    • Background proliferative endometrium
    • Comment: Neoplastic glands do not show cytologic changes expected with exogenous progestin treatment. Likewise, the background endometrium lacks morphology indicative of progestin effect. Correlation with clinical history and treatment adherence is recommended.

  • Endometrium, biopsy / curettage:
    • Residual endometrioid intraepithelial neoplasia / atypical hyperplasia / endometrioid carcinoma with treatment effects (see comment)
    • Endometrium with changes consistent with exogenous progestin therapy
    • Comment: Neoplastic glands and background endometrium show cytologic changes expected with exogenous progestin treatment. Changes are diagnostic of endometrioid intraepithelial neoplasia or carcinoma in context with progestin therapy related changes.

  • Endometrium, biopsy / curettage:
    • Glandular crowding consistent with residual endometrial neoplasia / hyperplasia with treatment effects (see comment)
    • Endometrium with changes consistent with exogenous progestin therapy
    • Comment: Changes are not diagnostic of endometrioid intraepithelial neoplasia / atypical hyperplasia or carcinoma but in context, likely represent residual endometrioid neoplasia with progestin therapy related changes.

  • Endometrium, biopsy / curettage:
    • Endometrium with changes consistent with exogenous progestin therapy (see comment)
    • Comment: No residual endometrioid neoplasia or carcinoma identified.
Board review style question #1

Among the following histologic features of endometrial endometrioid neoplasia / carcinoma, which one is attributable to progestin therapy?

  1. Increased cellularity
  2. Increased nuclear stratification
  3. Increased nuclear to cytoplasmic ratio
  4. Mucinous metaplasia
  5. Nonmorular solid growth
Board review style answer #1
D. Mucinous metaplasia. A range of metaplasias can be observed under progestin therapy, including secretory, squamous and mucinous metaplasia. Answers A - C are incorrect because progestin also is associated with decreases in the nuclear to cytoplasm ratio, cellularity and degree of nuclear stratification. Answer E is incorrect because nonmorular solid growth is a feature of endometrial endometrioid carcinoma used for grading purposes.

Comment Here

Reference: Progestin therapy related changes

SMARCA4 deficient uterine sarcoma
Definition / general
  • SMARCA4 deficient uterine sarcoma (SDUS) is a rare and highly aggressive uterine malignancy that occurs in young women and is defined by an inactivating mutation of the SMARCA4 gene
Essential features
  • Undifferentiated morphology (sheet-like architecture, monomorphic, discohesive and mitotically active)
  • Loss of nuclear SMARCA4 / BRG1 immunohistochemical staining (> 95% of cases)
Terminology
  • SMARCA4 deficient uterine tumor
ICD coding
  • Not assigned yet
Epidemiology
Sites
  • Uterus
Pathophysiology
  • Inactivating mutation of the SMARCA4 gene, a core subunit of the SWI / SNF complex, drives oncogenesis (Am J Surg Pathol 2020;44:263)
  • SWI / SNF (SWItch / sucrose nonfermentable) complex is an important regulator of the chromatin remodeling process
Etiology
  • Predisposition factor: rhabdoid tumor predisposition syndrome 2 (RTPS2) reported in 1 patient (Mod Pathol 2019;32:1675)
  • Cellular origin remains unknown; studies have shown DNA methylation signatures in SDUS that are distinct from other SWI / SNF deficient cancers of the gynecologic tract (J Pathol 2022;257:140)
Clinical features
Diagnosis
  • There are no established tests to screen for SDUS
  • When clinical suspicion arises, abdominal ultrasound and computed tomography scans are useful adjuncts
  • Definitive diagnosis requires biopsy
  • Referral to clinical genetics services and testing for germline SMARCA4 pathogenic variants should be considered in patients diagnosed with SDUS
Laboratory
  • Although morphologically and molecularly similar to small cell carcinoma of the ovary, hypercalcemic type, which can be associated with hypercalcemia, there are no reports of increased calcium levels in SDUS
Prognostic factors
  • Most reported tumors presented at high stage and showed a very aggressive clinical course
  • Due to the only small case series reported to date, there are no known prognostic factors
Case reports
Treatment
  • No established standard treatment
  • Patients are usually treated by a combination of surgery, chemotherapy and radiation
Gross description
  • Solid uterine mass usually with deep myometrial invasion
  • Mean size: 14 cm (range: 4 - 25 cm)
  • Tan-pink to gray-white soft cut surface
  • Hemorrhage and necrosis can be present
  • References: Mod Pathol 2019;32:1675, Mod Pathol 2018;31:1442
Microscopic (histologic) description
Microscopic (histologic) images

Contributed by Felix K.F. Kommoss, M.D.
Tumor cells in sheets, no gland or papillary formation

Tumor cells in sheets, no gland or papillary formation

Discohesive and rhabdoid morphology

Discohesive and rhabdoid morphology

Stromal hyalinization

Stromal hyalinization

Mitotic activity

Mitotic activity

Tumor necrosis

Tumor necrosis

SMARCA4 loss of expression

SMARCA4 loss of expression

Positive stains
Negative stains
Molecular / cytogenetics description
Sample pathology report
  • Uterus with ovaries and fallopian tube, total hysterectomy and bilateral salpingo-oophorectomy:
    • SMARCA4 deficient uterine sarcoma (see comment and synoptic report)
    • Comment: Immunohistochemical tests for SMARCA4 / BRG1 and SMARCA2 / BRM show loss of expression. SMARCA4 deficient uterine sarcoma (SDUS) can be associated with the rhabdoid tumor predisposition syndrome 2 (RTPS2). Referral to medical genetics is recommended.
Differential diagnosis
  • Undifferentiated and dedifferentiated endometrial carcinoma:
    • Older age (median: 55 years)
    • May be associated with a differentiated endometrial carcinoma, usually low grade endometrioid adenocarcinoma (dedifferentiated carcinoma)
    • Mismatch repair deficiency is common
    • Loss of nuclear expression of SMARCA4 / BRG1, SMARCB1 / INI1 or ARID1B
    • Associated with other genetic alterations (PTEN, ARID1A, PIK3CA, KRAS, CTNNB1, NRAS, TP53)
  • Müllerian adenosarcoma:
    • Sarcomatous cells are more spindled and display only mild atypia
    • High grade sarcomatous components (sarcomatous overgrowth) usually display marked pleomorphism and may be associated with TP53 alterations
  • Embryonal rhabdomyosarcoma:
    • Moderately cellular with both hyper and hypocellular areas
    • May show conspicuous strap cells with bright eosinophilic stroma
    • May show nodules of cartilage (may be DICER1 associated)
    • Positive for desmin, myogenin or MyoD1 (can be focal)
  • High grade endometrial stromal sarcoma:
    • Typically, diffuse positivity for cyclin D1
    • Typically harbors YWHAE::NUTM2A / B gene fusion, BCOR / BCORL1 rearrangements or BCOR internal tandem duplication (ITD)
  • Undifferentiated uterine sarcoma:
    • Older age
    • Marked pleomorphism
    • Associated with aberrant p53 staining pattern and high copy number variations
Board review style question #1

The images above show SMARCA4 / BRG1 IHC for a 28 year old woman with a 10 cm mass of the uterine corpus. What is the most likely diagnosis?

  1. Dedifferentiated endometrial carcinoma
  2. High grade endometrial stromal sarcoma
  3. Low grade endometrial stromal sarcoma
  4. Metastatic melanoma
  5. SMARCA4 deficient uterine sarcoma
Board review style answer #1
E. SMARCA4 deficient uterine sarcoma (SDUS). The H&E shows sheets of undifferentiated tumor cells with rhabdoid features. The immunohistochemical test for SMARCA4 / BRG1 shows aberrant expression (loss of nuclear staining). The morphology and immunophenotype are consistent with the diagnosis of SDUS. Answer A is incorrect because although dedifferentiated endometrial carcinoma is morphologically similar to SDUS and may show loss of SMARCA4 expression, this case lacks the presence of a differentiated component. Answers B, C and D are incorrect because these diagnoses may have overlapping morphologic features with SDUS; however, they retain SMARCA4 / BRG1 expression.

Comment Here

Reference: SMARCA4 deficient uterine sarcoma
Board review style question #2
Which of the following immunohistochemical tests would most likely show aberrant expression in SMARCA4 deficient uterine sarcoma (SDUS)?

  1. ARID1B
  2. MLH1
  3. MSH6
  4. p53
  5. SMARCA2 / BRM
Board review style answer #2
E. SMARCA2 / BRM. SMARCA4 deficient uterine sarcomas (SDUS) are molecularly defined by inactivating SMARCA4 mutations, which are associated with SMARCA4 / BRG1 and SMARCA2 / BRM nuclear loss of expression. Answers A, B, C and D are incorrect because these tests show aberrant expression in other entities such as dedifferentiated and undifferentiated endometrial carcinoma (ARID1B loss of expression and often mismatch repair deficient) and high grade endometrioid and serous carcinoma (aberrant p53 expression).

Comment Here

Reference: SMARCA4 deficient uterine sarcoma

Serous carcinoma
Definition / general
  • High grade estrogen independent carcinoma of the endometrium showing marked cytologic atypia
  • Complex papillary, solid or glandular architecture; similar to ovarian / tubal high grade serous carcinoma
Essential features
  • High grade carcinoma arising in postmenopausal women
  • Often arises within endometrial polyps; background endometrium often atrophic
  • Immunohistochemistry shows mutational pattern of p53 expression (strong and diffuse or complete absence)
  • Associated with worse prognosis than endometrioid carcinoma and can present at high stage
Terminology
  • Endometrial serous carcinoma
  • Historical: uterine papillary serous carcinoma
  • Serous endometrial intraepithelial carcinoma: replacement of the surface epithelium or preformed endometrial glands with serous carcinoma without invasion of underlying stroma (Adv Anat Pathol 2004;11:117)
ICD coding
  • ICD-10: C54.1 - malignant neoplasm of the endometrium
  • ICD-O: 8441/3 - serous carcinoma, NOS
Epidemiology
Sites
  • Uterine corpus
  • More likely to present at high stage (disease outside uterus) than endometrioid carcinomas, even when confined to a polyp or without evidence of invasive disease (Br J Cancer 2006;94:642)
Pathophysiology
Etiology
Clinical features
  • Postmenopausal bleeding is common
Diagnosis
  • Diagnosis can be made on endometrial biopsy, curettage or polypectomy upon workup for postmenopausal bleeding
  • Incidental carcinomas can be found, usually in association with endometrial polyps, within uteri removed for benign conditions
Laboratory
  • Can show elevated tumor marker CA-125
Prognostic factors
Case reports
Treatment
  • Treatment is primarily surgical + / - platinum based chemotherapy / radiotherapy
  • Recent studies suggest a role for treatment with trastuzumab in patients with tumors showing overexpression of HER2 (J Clin Oncol 2018;36:2044)
  • Pembrolizumab if tumor is microsatellite instability-high (MSI-H) or MMR-deficient (Clin Cancer Res 2019;25:3753)
Gross description
  • No unique gross findings which would separate serous from nonserous carcinoma
  • Endometrial polyp may be present
Microscopic (histologic) description
  • Architecture:
    • Papillary with or without appreciable fibrovascular cores; micropapillary pattern can be seen
    • Slit-like spaces
    • Gland-like spaces may be observed (but luminal borders are not sharp as seen in endometrioid carcinoma)
    • Solid growth
  • Psammoma bodies may be present in up to 33% of cases
  • Cytoplasm usually scant but can be abundant with eosinophilia or clearing
  • Tumor cells can colonize existing endometrial glands
  • Tumor cells can appear discohesive
  • Nuclei are typically high grade with pleomorphism, hyperchromasia, prominent nucleoli and frequent mitotic figures (including atypical mitotic figures)
  • Can be confined to an endometrial polyp as a small surface proliferation or completely effacing the polyp
  • Background endometrium, if present, is often atrophic
  • Not graded into low and high grade as done in the ovary but by default is a high grade carcinoma
  • Select references / reviews: Adv Anat Pathol 2011;18:415, Adv Anat Pathol 2004;11:117
Microscopic (histologic) images

Contributed by Ricardo Lastra, M.D.
Involving polyp (intraepithelial carcinoma)

Involving polyp (intraepithelial carcinoma)

Solid and papillary growth

Solid and papillary growth

Pleomorphic nuclei

Pleomorphic nuclei

Glandular growth

Glandular growth


p53 overexpression

p53 overexpression

Complete absence of p53 IHC

Complete absence of p53 IHC

p16

p16

Cytology description
  • Cytologic findings are similar to ovarian high grade serous carcinoma
  • Tumor cells should look frankly malignant and pleomorphic:
    • Nuclear hyperchromasia
    • Irregular nuclear contours
    • Irregular coarse chromatin
    • Prominent cherry-red nucleoli
  • Cytoplasmic vacuoles may be present
  • Reference: Diagn Cytopathol 2016;44:1039
Cytology images

Contributed by Ricardo Lastra, M.D.
Diff-Quik

Diff-Quik

Positive stains
  • p53: mutation type staining either strong and diffuse, complete absence of staining (“null type” pattern) or abnormal cytoplasmic localization
  • p16: often strong and diffuse (not related to HPV infection)
  • AE1 / AE3 and CK7: strong membranous staining
  • PAX8: strong nuclear staining
  • MLH1, MSH1, MSH2 and MSH6: typically retained but can show loss of at least one marker in 10% of cases (Int J Gynecol Pathol 2019;38 Suppl 1:S40)
Negative stains
  • CK20: no expression
  • ER / PR: decreased expression; often negative or focally positive in approximately 50% of cases
  • WT-1: maybe focally positive in 30% of cases; if strong and diffuse, extrauterine serous carcinoma enters the differential
  • See review: Int J Gynecol Pathol 2019;38 Suppl 1:S40
Molecular / cytogenetics description
  • Mutations in TP53 (80 - 90%) and PIK3CA (24 - 40%) common
  • Mutations in PTEN and ARIDA1A uncommon
  • Serous carcinomas show high somatic copy number abnormalities (Nature 2013;497:67)
Sample pathology report
  • Endometrial polyps, curettage / polypectomy:
    • Endometrial serous carcinoma involving endometrial polyp (see comment)
    • Background atrophic endometrium
    • Comment: Immunohistochemical stains show that the tumor is positive for p53 (strong and diffuse, mutated) and p16 (strong and diffuse), focally positive for ER and negative for PR. This immunoprofile supports the above diagnosis.
  • Uterus, hysterectomy:
    • Endometrial serous carcinoma (0.8 cm) with superficial myometrial invasion (see synoptic report and comment)
    • Comment: Immunohistochemical stains show that the tumor is positive for p16 (strong and diffuse), focally positive for ER and PR and has no expression for p53 (null mutation). This immunoprofile supports the above diagnosis.
Differential diagnosis
  • Endometrial endometrioid adenocarcinoma
    • Most important differential is villoglandular pattern
    • Areas with well developed, readily identifiable glands with sharp luminal borders should be present
    • Squamous or mucinous metaplasia often seen in endometrioid carcinomas (and not present in serous carcinomas)
    • Cytologically bland in comparison to serous carcinoma
    • ER / PR strongly positive
    • p53 and p16 weak and patchy
  • Endometrial clear cell adenocarcinoma
    • Typical papillary, tubulocystic and solid architecture
    • Papillae are typically small with hyalinized cores lined by a single layer of polygonal hobnailed cells
    • Uniform nuclear atypia but overt nuclear pleomorphism at a lesser degree than what can be seen in serous and high grade endometrioid carcinoma
    • Clear to eosinophilic (oxyphilic) cytoplasm; often shows prominent nucleoli
    • More likely to be positive for HNF-1β and Napsin A
  • Mixed carcinoma
    • Defined by WHO as a tumor composed of at least two components (with each component encompassing at least 10% of the tumor volume), with one of the two components being high grade
    • Endometrioid adenocarcinoma and serous carcinoma are the typical tumor types identified
  • Secondary involvement of the endometrium by tubo-ovarian carcinoma
    • Clinical findings should indicate adnexal mass
    • WT-1 often strong and diffuse in extrauterine primary serous carcinoma
  • Metaplastic changes
    • Morphologic clues can help differentiate
      • Cilia present in tubal metaplasia
      • Hobnail metaplasia shows only mild nuclear atypia and infrequent mitoses
      • Papillary syncytial metaplasia is often associated with endometrial stromal breakdown and often demonstrates neutrophilic microabscesses
    • Metaplastic processes, in general, should show low Ki67 proliferation indices and wild type (non-mutated) p53 expression
Board review style question #1
    The carcinoma shown in the image above, which involves the surface of an endometrial polyp, shows strong and diffuse positivity for p53 and p16. Which of the follow is true regarding the above carcinoma?

  1. Likely arose in a patient less than 30 years of age
  2. Considered an estrogen dependent endometrial carcinoma
  3. Even without definitive invasion, can present at high stage with tumor dissemination in the peritoneal cavity
  4. Often shows monotonous nuclei with little pleomorphism and low proliferation index
Board review style answer #1
C. This is a serous endometrial intraepithelial carcinoma in an endometrial polyp. Even without definitive invasion, can present at high stage with tumor dissemination in the peritoneal cavity.

Comment Here

Reference: Serous carcinoma
Board review style question #2
    Which of the following is true regarding serous carcinomas of the endometrium?

  1. Mutations in p53 are the most common driving molecular event
  2. WT-1 is often strong and diffusely positive
  3. Show well formed glands with sharp luminal borders
  4. p16 overexpression indicates a role for HPV infection in the etiology
Board review style answer #2
A. Mutations in p53 are the most common driving molecular event.

Comment Here

Reference: Serous carcinoma

Smooth muscle tumors of uncertain malignant potential
Definition / general
  • Smooth muscle tumor with morphologic features exceeding diagnostic criteria for leiomyoma (including subtypes) but insufficient for a diagnosis of leiomyosarcoma
Essential features
  • STUMPs are morphologically heterogeneous and diagnostically challenging, requiring generous sampling for microscopic examination
  • Variable diagnostic criteria proposed for spindle cell, myxoid and epithelioid STUMPs
  • Usually occur in women of reproductive age or postmenopausal women
  • Recurrence rates are 7 - 28%, with higher recurrence rates for epithelioid and myxoid STUMPs
  • Recurrent tumors may look histologically similar to the initial STUMP or may be consistent with leiomyosarcoma
Terminology
  • Spindle cell STUMP
  • Myxoid STUMP
  • Epithelioid STUMP
ICD coding
  • ICD-O: 8897/1 - smooth muscle tumor of uncertain malignant potential
  • ICD-11: 2F76 & XH1EN1 - neoplasms of uncertain behaviour of female genital organs & smooth muscle tumor of uncertain malignant potential
Epidemiology
Sites
  • Myometrium
  • Rarely, broad ligament, ovaries, cervix or vagina
Pathophysiology
Etiology
  • Unknown
Clinical features
Diagnosis
  • Microscopic examination
Radiology description
  • Ultrasonography:
    • Usually circumscribed, showing isoechoic or mixed echogenicity with microcystic anechoic areas, poorly or moderately vascularized with both circumferential and intralesional flows and rarely shadowing (Eur J Obstet Gynecol Reprod Biol 2020;251:167)
  • Magnetic resonance imaging based on limited data:
Prognostic factors
Case reports
Treatment
Gross description
Gross images

Images hosted on other servers:

Intramural tumor with hemorrhage

Frozen section description
  • Not usually diagnosed on frozen section
Microscopic (histologic) description
Microscopic (histologic) images

Contributed by Gulisa Turashvili, M.D., Ph.D.
Coagulative necrosis

Coagulative necrosis

Elevated mitoses

Elevated mitoses

Epithelioid morphology

Epithelioid morphology

Irregular borders

Irregular borders

Positive stains
Negative stains
  • p16: negative or patchy
  • p53: typically wild type
  • CD10: usually negative
Sample pathology report
  • Uterus with cervix, total hysterectomy:
    • Myometrium:
      • Smooth muscle tumor of uncertain malignant potential (see comment)
      • Leiomyomas and adenomyosis
    • Endometrium: proliferative
    • Cervix and uterine serosa: unremarkable
    • Comment: The largest myometrial nodule is well circumscribed and measures 7.5 cm. Microscopically, it is a smooth muscle neoplasm composed of intersecting fascicles of spindle cells lacking cytologic atypia or elevated mitotic activity (3 mitoses seen per 10 HPF, FD = 0.55). However, multiple foci of unequivocal coagulative tumor cell necrosis are present in several blocks. Despite the lack of cytologic atypia or elevated mitoses, the presence of coagulative necrosis warrants classification as a smooth muscle tumor of uncertain malignant potential.
Differential diagnosis
  • Conventional (spindle cell) leiomyoma:
    • Intersecting fascicles of spindle cells, eosinophilic fibrillary cytoplasm and cigar shaped nuclei lacking coagulative necrosis, cytologic atypia or elevated mitotic activity
  • Cellular leiomyoma:
    • More cellular compared with the surrounding myometrium
    • Thick walled vessels and cleft-like spaces
    • Usually spindled cells with scant cytoplasm
    • No coagulative necrosis, cytologic atypia or elevated mitoses
  • Epithelioid leiomyoma:
    • Rounded or polygonal cells with eosinophilic granular or clear cytoplasm with low mitotic count at < 0.8 mitoses / mm2 or < 2 mitoses / 10 HPF (FD = 0.55, 0.24 mm2 in area) and lacking cytologic atypia
  • Myxoid leiomyoma:
    • Circumscribed, hypocellular, myxoid tumor lacking cytologic atypia or mitotic activity
  • Leiomyoma with bizarre nuclei:
    • Cells with bizarre nuclei in a background of typical leiomyoma, lacking coagulative necrosis or elevated mitotic activity (usually < 2 mitoses / mm2 or < 5 mitoses / 10 HPF, FD = 0.55 mm, 0.24 mm2 in area)
  • Fumarate hydratase deficient leiomyoma:
    • Leiomyoma with staghorn vessels, alveolar type edema, scattered cells with bizarre nuclei, enlarged nuclei with prominent nucleoli and perinucleolar haloes, eosinophilic cytoplasmic inclusions
  • Mitotically active leiomyoma:
    • Leiomyoma with elevated mitotic count at 2.5 - 6 mitoses / mm2 or 6 - 14 mitoses / 10 HPF (FD = 0.55 mm, 0.24 mm2 in area) but lacking coagulative necrosis or cytologic atypia
  • Conventional (spindle cell) leiomyosarcoma:
    • 2 of 3 diagnostic features present, including coagulative necrosis, cytologic atypia, elevated mitotic count at ≥ 4 mitoses / mm2 or ≥ 10 mitoses / 10 HPF (FD = 0.55, 0.24 mm2 in area)
  • Epithelioid leiomyosarcoma:
    • Predominant epithelioid morphology (> 50% of overall tumor volume)
    • Moderate to severe cytologic atypia or coagulative necrosis or ≥ 1.6 mitoses / mm2 / ≥ 4 mitoses / 10 HPF (FD = 0.55, 0.24 mm2 in area)
  • Myxoid leiomyosarcoma:
    • Moderate to severe cytologic atypia or coagulative necrosis or > 0.4 mitoses / mm2 / > 1 mitosis / 10 HPF (FD = 0.55, 0.24 mm2 in area) or infiltrative / irregular borders
Board review style question #1
Which of the following smooth muscle tumors falls short for a diagnosis of STUMP?

  1. Smooth muscle tumor lacking coagulative necrosis and cytologic atypia, with 16 mitoses / 10 HPF
  2. Smooth muscle tumor with 3 mitoses / 10 HPF, lacking coagulative necrosis and cytologic atypia
  3. Smooth muscle tumor with coagulative necrosis and 2 mitoses / 10 HPF, lacking cytologic atypia
  4. Smooth muscle tumor with epithelioid morphology and 3 mitoses / 10 HPF, lacking coagulative necrosis and cytologic atypia
  5. Smooth muscle tumor with myxoid morphology, focally irregular borders and mild cytologic atypia, lacking coagulative necrosis or mitotic activity
Board review style answer #1
B. Smooth muscle tumor with 3 mitoses / 10 HPF, lacking coagulative necrosis and cytologic atypia

Comment Here

Reference: Smooth muscle tumors of uncertain malignant potential
Board review style question #2
Which immunohistochemical markers are useful for differentiating leiomyoma from STUMP?

  1. CD10, hormone receptors, p53 and p16
  2. Desmin, h-caldesmon, p53 and p16
  3. Desmin, h-caldesmon, SMA, CD10 and fumarate hydratase
  4. Desmin, h-caldesmon, SMA, CD10 and hormone receptors
  5. Immunohistochemistry has no value in this differential diagnosis
Board review style answer #2
E. Immunohistochemistry has no value in this differential diagnosis

Comment Here

Reference: Smooth muscle tumors of uncertain malignant potential

Staging-carcinoma and carcinosarcoma
Definition / general
  • Current staging of endometrial carcinoma is based on the International Federation of Obstetrics and Gynecology (FIGO for its name in French), most recently updated in 2021 (Int J Gynaecol Obstet 2021;155:45)
  • The American Joint Committee on Cancer (AJCC) staging system currently harmonizes its tumor (T), lymph node (N) and metastasis (M) categories with the FIGO system (see Table 1) (AJCC: Cancer Staging Manual, 8th Edition, 2017)
  • Initial stage should not be changed due to disease progression or recurrence or based on response to initial radiation therapy or chemotherapy that precedes primary tumor resection
  • NCCN guidelines for management of uterine neoplasms utilize the 2018 FIGO staging criteria (J Natl Compr Canc Netw 2023;21:181)
    • However, the 2023 update takes histologic and molecular features into account with the goal of creating substages that better align with prognosis and treatment options (see Tables 2 and 3) (Int J Gynaecol Obstet 2023;162:383)
    • Management guidelines are likely to be updated in the relatively near future
  • The International Collaboration on Cancer Reporting (ICCR) recently updated its reporting tool for endometrial carcinoma, incorporating core and optional elements of reporting and current staging elements (Int J Gynecol Pathol 2022;41:S90, ICCR: Endometrial Cancers [Accessed 9 February 2023])
  • In addition, the International Society of Gynecological Pathologists (ISGyP) endometrial carcinoma consensus papers provide guidance in the evaluation of staging and other prognostic variables (Int J Gynecol Pathol 2019;38:S93)
Essential features
  • AJCC 7th edition staging was sunset on December 31, 2017; as of January 1, 2018, use of the 8th edition is mandatory
  • Primary tumor (T) staging depends on depth of myometrial invasion (absent or inner half stage T1a, outer half stage T1b), cervical involvement (stage T2), serosal or adnexal involvement (stage T3a) and parametrial or vaginal involvement (stage T3b)
  • Regarding lymph node (N) staging, the AJCC system now includes staging categories for lymph node metastases according to metastatic tumor size: isolated tumor cells (ITCs), micrometastases and macrometastases; the FIGO stage does not recognize metastatic nodal tumor size for staging purposes
ICD coding
  • ICD-10: C54.1 - malignant neoplasm of endometrium
Diagrams / tables

Table 1: Staging of endometrial carcinoma as per FIGO (2018 update) and AJCC 8th edition
FIGO (2018) AJCC (2018) Description
I T1 Tumor confined to the uterus
1A T1a Tumor confined to the endometrium or invades < 50% of the myometrial wall
1B T1b Tumor invades > 50% of the myometrial wall
II T2 Tumor infiltrates cervical stroma
III T3, N1 Tumor extends outside the uterus
IIIA T3a Tumor involves serosa or adnexa (direct extension or metastases)
IIIB T3b Tumor involves vagina or parametria (direct extension or metastases)
IIIC N1 Tumor with pelvic lymph node metastasis
IIIC1 N1 mi Micrometastases in pelvic lymph nodes (> 0.2 mm or 200 cells but ≤ 2 mm)
N1a Macrometastases in pelvic lymph nodes (> 2 mm in size)
IIIC2 N2 mi Micrometastases in para-aortic lymph nodes (> 0.2 mm or 200 cells but ≤ 2 mm)
N2a Macrometastases in para-aortic lymph nodes (> 2 mm in size)
IV T4, M1 Tumor invades bladder or bowel mucosa or involves distant organs
IVA T4 Tumor invades bladder mucosa or bowel mucosa
IVB M1 Distant metastases (including intra-abdominal metastases and inguinal lymph nodes;
excluding metastases to vagina, pelvic serosa or adnexa)
N0i+ Isolated tumor cells in regional lymph nodes*
* Isolated tumor cells category is recognized in the AJCC (TNM) system but not in the FIGO system

Table 2: 2023 FIGO staging of cancer of the endometrium
Stage Description
I Confined to the uterine corpus and ovary
IA Disease limited to the endometrium or nonaggressive histological type (i.e., low grade endometroid, with invasion of < 50% of myometrium with no or focal lymphovascular space involvement [LVSI] or good prognosis disease)
IA1 Nonaggressive histological type limited to an endometrial polyp or confined to the endometrium
IA2 Nonaggressive histological types involving < 50% of the myometrium with no or focal LVSI
IA3 Low grade endometrioid carcinomas limited to the uterus and ovary
IB Nonaggressive histological types with invasion of ≥ 50% of the myometrium and with no or focal LVSI
IC Aggressive histological types limited to a polyp or confined to the endometrium
II Invasion of cervical stroma without extrauterine extension or with substantial LVSI or aggressive histological types with myometrial invasion
IIA Invasion of the cervical stroma of nonaggressive histological types
IIB Substantial LVSI of nonaggressive histological types
IIC Aggressive histological types with any myometrial involvement
III Local or regional spread of the tumor of any histological subtype
IIIA Invasion of uterine serosa, adnexa or both by direct extension or metastasis
IIIA1 Spread to ovary or fallopian tube (except when meeting stage IA3 criteria)
IIIA2 Involvement of uterine subserosa or spread through the uterine serosa
IIIB Metastasis or direct spread to the vagina or to the parametria or pelvic peritoneum
IIIB1 Metastasis or direct spread to the vagina or the parametria
IIIB2 Metastasis to the pelvic peritoneum
IIIC Metastasis to the pelvic or para-aortic lymph nodes or both
IIIC1 Metastasis to the pelvic lymph nodes
IIIC1i Micrometastasis
IIIC1ii Macrometastasis
IIIC2 Metastasis to para-aortic lymph nodes up to the renal vessels, with or without metastasis to the pelvic lymph nodes
IIIC2i Micrometastasis
IIIC2ii Macrometastasis
IV Spread to the bladder mucosa or intestinal mucosa or distance metastasis
IVA Invasion of the bladder mucosa or the intestinal / bowel mucosa
IVB Abdominal peritoneal metastasis beyond the pelvis
IVC Distant metastasis, including metastasis to any extra or intra-abdominal lymph nodes above the renal vessels, lungs, liver, brain or bone
  • Disease limited to low grade endometrioid carcinomas involving the endometrium and ovaries (stage IA3) must be distinguished from extensive spread of the endometrial carcinoma to the ovary (stage IIIA1), by the following criteria
    1. No more than superficial myometrial invasion is present (< 50%)
    2. Absence of extensive / substantial LVSI
    3. Absence of additional metastases
    4. The ovarian tumor is unilateral, limited to the ovary, without capsule invasion / rupture (equivalent to pT1a)
  • LVSI as defined in WHO 2021: extensive / substantial, ≥ 5 vessels involved
  • Aggressive histological types include serous carcinomas, clear cell carcinomas, mesonephric-like carcinomas, gastrointestinal type mucinous endometrial carcinoma, undifferentiated carcinomas and carcinosarcomas
    • For endometrial endometrioid carcinomas, grade is based on the proportion of solid areas: low grade = grade 1 (≤ 5%) and grade 2 (6 - 50%); and high grade = grade 3 (> 50%)
    • Nuclear atypia excessive for the grade raises the grade of a grade 1 or 2 tumor by one
  • Nonaggressive histological types are composed of low grade (grade 1 and 2) endometrial endometrioid carcinomas


Table 3: 2023 FIGO endometrial cancer stage with molecular classification (for patients with early endometrial cancer [stages I and II after surgical staging])
Stage Description
IAmPOLEmut POLEmut endometrial carcinoma, confined to the uterine corpus or with cervical extension, regardless of the degree of LVSI or histological type
IICmp53abn p53abn endometrial carcinoma confined to the uterine corpus with any myometrial invasion, with or without cervical invasion and regardless of the degree of LVSI or histological type
Primary tumor (pT) and FIGO stages ()
  • pTX: primary tumor cannot be assessed
  • pT0: no evidence of primary tumor
  • pT1 (I): tumor confined to corpus uteri
    • pT1a (IA): tumor limited to endometrium or invades < 50% of the myometrium
    • pT1b (IB): tumor invades ≥ 50% of the myometrium
  • pT2 (II): tumor invades stromal connective tissue of the cervix but does not extend beyond uterus
  • pT3 (III): tumor involving serosa, adnexa, vagina or parametrium
    • pT3a (IIIA): tumor involves serosa or adnexa (direct extension or metastasis)
    • pT3b (IIIB): vaginal involvement (direct extension or metastasis) or parametrial involvement
  • pT4 (IVA): tumor invades bladder mucosa or bowel mucosa (bullous edema is not sufficient to classify a tumor as pT4)

Notes:
  • Endocervical glandular involvement only should be considered as stage I and not stage II
  • pTis is no longer a staging category
  • Myometrial invasion:
    • According to the ISGYP recommendations, this variable should be reported as none or less than half (< 50%) or half or more (≥ 50%) of the myometrial wall thickness
    • This approach is relatively easy and obviates in most cases the need to locate the endomyometrial junction (which is difficult when the junction is completely obliterated or not sampled in the sections)
    • In addition, other variables than can be reported include myoinvasion in terms of absolute numbers (invasive carcinoma depth in mm / myometrial thickness in mm) and the percentage of myometrial wall involved by tumor (a derivative of the absolute measurements in mm)
    • Assessment should be performed at the deepest point of tumor invasion on a full thickness section of the wall spanning from mucosal surface to serosa (Figure 1)
    • Involvement of adenomyosis by carcinoma does not by itself portend a worse prognosis
      • These cases behave similarly to stage IA tumors (Gynecol Oncol 1990;37:401, Int J Gynecol Pathol 2010;29:445)
      • Carcinoma is confined to the endometrium / adenomyosis when:
        • Focus of carcinoma has a round or ovoid shape with a smooth and convex outer contour (regardless of its size) (Figure 2)
        • Outer contour smoothly continues with adjacent uninvolved endomyometrial interface
        • Residual endometrial type stroma or benign endometrial glands are present in the periphery or within the focus
        • There is no significant desmoplastic reaction
    • If myometrial invasion arises from areas of adenomyosis, the location of the deepest myoinvasive point with respect to the entire uterine wall should be recorded (inner versus outer wall) (Figure 3)
      • There is controversy in whether to assign stage IA or IB to a carcinoma with foci of invasion in the outer wall arising from adenomyosis; the ISGyP endometrial carcinoma consensus recommends:
        • If the foci are few and clearly arising from deep adenomyosis, it is advisable to categorize these lesions as stage IA; there is emerging evidence showing that these lesions behave more indolently, similar to stage IA tumors (Int J Gynecol Pathol 2019;38:S93, Mod Pathol 2019;32:1)
        • If, on the other hand, myoinvasive carcinoma from adenomyosis is rather extensive and seen throughout the wall, it is prudent to classify the tumor as stage IB (Int J Gynecol Pathol 2019;38:S93)
    • If the tumor is exophytic at the point of deepest myometrial invasion, the nearest endomyometrial junction should be identified and depth should be measured from that point
      • Same recommendation applies to tumors arising from or involving an endometrial polyp
    • If the carcinoma at the point of deepest invasion involves a leiomyoma, the myometrial thickness should include the leiomyoma (Int J Gynecol Pathol 2019;38:S93)
    • Lymphovascular space invasion (LVI) should not be included in the assessment of myometrial invasion
  • Cervical stromal involvement:
    • Presence of cervical stromal involvement defines stage II
    • Depth of invasion into the cervical stroma and margin status should be reported, as it may influence adjuvant radiation treatment (Brachytherapy 2019;18:606)
    • Assessment of cervical involvement is inconsistent between gynecologic pathologists (Am J Surg Pathol 2011;35:289)
    • Uppermost endocervical mucinous gland in the section should be considered as the upper limit of the endocervix (Adv Anat Pathol 2018;25:71)
      • If unsure whether tumor is in the cervix or lower uterine segment, identify the most proximal endocervical mucinous gland, which will define the boundary between these anatomic locations (Figures 4 and 5)
    • Stromal invasion can be seen as obviously infiltrative glands or cells with desmoplasia or as complex architecture relative to the normal endocervical glandular compartment
    • Carcinoma confined to the surface or to endocervical glands can be reported but does not modify stage or impact prognosis
    • Reporting the depth of cervical stromal invasion by endometrial carcinoma is optional
      • The National Comprehensive Cancer Network (NCCN) Clinical Practice Guidelines in Oncology Uterine Neoplasms lists deep cervical stromal invasion as an adverse risk factor in patients with stage II endometrial carcinoma (NCCN: Uterine Neoplasms [Accessed 9 February 2023])
      • Estimation of depth of cervical wall invasion may be used to determine the type of adjuvant radiation (Brachytherapy 2019;18:606)
      • Depth of invasion into the cervix can be measured in mm, providing the thickness of the cervical wall at that level in mm as well
      • Alternatively, it can be reported in terms of invasion confined to 33% of the wall, 66% of the wall or extending to the outer third of the wall
    • It is recommended to measure the distance between tumor and margin: distal cervical / vaginal mucosa and parametrial margins
  • Serosal involvement:
    • Involvement of the uterine serosa is an adverse prognostic factor and warrants a stage IIIA
    • Involvement of the serosa is diagnosed when the tumor perforates through the mesothelium lined outer surface of the uterus; this is seen as an area of disruption of the serosa with tumor involvement, sometimes with soft tissue or other structures attached to it
    • Sometimes a desmoplastic stromal reaction is present
      • Locating the serosal plane flanking the area in question and extending the plane through the area of desmoplasia can be helpful; serosal involvement is considered present if there is disruption of that plane or carcinoma extends beyond the plane (Int J Gynecol Pathol 2022;41:S90)
    • According to ISGYP guidelines, tumor that infiltrates the entire myometrial thickness and reaches submesothelial fibroconnective tissue should also be reported as serosal involvement (Int J Gynecol Pathol 2019;38:S93)
  • Adnexal involvement (versus synchronous ovarian carcinoma):
    • Distinguishing between the following is relevant for staging and treatment:
      • Synchronous ovarian and endometrial tumors
      • Primary endometrial with metastatic ovarian involvement
      • Primary ovarian with metastatic endometrial involvement
    • However, it is often difficult to separate these scenarios on pathologic examination and one may only suggest one possibility over the others
    • Synchronous ovarian carcinoma is present in up to 30% of reproductive age women with endometrial cancer (Int J Womens Health 2014;6:691, Obstet Gynecol 2005;106:693)
      • When present, synchronous ovarian and endometrial carcinomas usually have the same histologic type (vast majority are low grade endometrioid)
      • Synchronous endometrial and ovarian endometrioid carcinoma have good prognosis, similar to those with only endometrial or only ovarian stage I cancer (Int J Gynecol Cancer 2014;24:54, Clin Cancer Res 2008;14:5840)
    • Features that support 2 synchronous carcinomas include:
      • Different histologic types or tumor grades
      • Endometrial carcinoma has absent or only superficial myometrial invasion
      • Ovarian carcinoma is confined to the ovary
      • Absence of lymphovascular space invasion
      • Different patterns of mismatch repair (MMR) or hormone receptor expression by IHC
    • Conversely, secondary ovarian involvement by a primary endometrial tumor should be considered when:
      • Endometrial tumor has deep myometrial invasion, lymphovascular space invasion, serosal or parametrial involvement
      • Bilateral ovarian involvement
      • Ovarian tumor involves surface and has extensive lymphovascular space invasion
      • Both tumors show similar MMR and hormone receptor expression by IHC
Regional lymph nodes (pN)
  • pNX: regional lymph nodes cannot be assessed
  • pN0: no regional lymph node metastasis
    • pN0(i+): isolated tumor cells in regional lymph node(s) ≤ 0.2 mm
  • pN1 (IIIC1): regional lymph node metastasis to pelvic lymph nodes
    • pN1mi: regional lymph node metastasis (> 0.2 mm but ≤ 2 mm in diameter) to pelvic lymph nodes
    • pN1a: regional lymph node metastasis (> 2 mm in diameter) to pelvic lymph nodes
  • pN2 (IIIC2): regional lymph node metastasis to para-aortic lymph nodes, with or without positive pelvic lymph nodes
    • pN2mi: regional lymph node metastasis (> 0.2 mm but ≤ 2 mm in diameter) to pelvic lymph nodes
    • pN2a: regional lymph node metastasis (> 2 mm in diameter) to pelvic lymph nodes

Notes:
  • Emerging data shows that patients with low volume disease (isolated tumor cells and micrometastases) have better recurrence free survival compared to those with macrometastatic nodal disease (Ann Surg Oncol 2016;23:1653, Gynecol Oncol 2017;146:240)
  • When measuring size of tumor within lymph nodes, tumor foci should be measured separately; then the largest individual size is reported
  • It is recommended to report extranodal extension if present (although it is not a mandatory staging criterion)
Distant metastasis (pM)
  • pM0: no distant metastasis
  • pM1 (IVB): distant metastasis (includes metastasis to inguinal lymph nodes, intraperitoneal disease or lung, liver or bone; it excludes metastasis to para-aortic lymph nodes, vagina, pelvic serosa or adnexa)
Prefixes
  • c: clinical classification
  • p: pathological classification
  • y: post therapy classification
  • r: recurrence or retreatment classification
  • a: autopsy classification
AJCC prognostic stage groups
Stage 0: Tis N0 M0
Stage I: T1 N0 M0
Stage IA: T1a N0 M0
Stage IB: T1b N0 M0
Stage II: T2 N0 M0
Stage III: T3 N0 M0
Stage IIIA: T3a N0 M0
Stage IIIB: T3b N0 M0
Stage IIIC1: T1 - 3 N1 M0
Stage IIIC2: T1 - 3 N2 M0
Stage IVA: T4 any N M0
Stage IVB: any T any N M1
Histologic grade (G)
  • FIGO grading applies to endometrial carcinomas of the endometrioid type:
    • Grade 1: ≤ 5% of a nonsquamous or nonmorular solid growth pattern
    • Grade 2: 6 - 50% of a nonsquamous or nonmorular solid growth pattern
    • Grade 3: > 50% of a nonsquamous or nonmorular solid growth pattern

Notes:
Microscopic (histologic) images

Contributed by Carlos Parra-Herran, M.D.

Depth of myometrial invasion

Endometrial carcinoma involving adenomyosis

Myometrial invasion from adenomyosis

Cervical involvement by endometrial carcinoma (pT2)

Board review style question #1
What is the defining feature of stage T3a / IIIA carcinoma of the uterine corpus?

  1. Involvement of bladder or rectal mucosa
  2. Involvement of cervical stromal tissue
  3. Involvement of uterine serosa or adnexa
  4. Involvement of vagina or parametrial tissue
  5. Pelvic lymph node metastases
Board review style answer #1
C. Involvement of uterine serosa or adnexa. Serosal or adnexal involvement by endometrial carcinoma is the defining feature of stage T3a (AJCC) / IIIA (FIGO). Bladder / rectal mucosal involvement, cervical stromal involvement, vaginal / parametrial involvement and lymph node metastases are the defining features of stages T4 / IVA, T2 / II, T3b / IIIB and N1 / IIIC1, respectively.

Comment here

Reference: Uterus - Staging-carcinoma and carcinosarcoma
Board review style question #2
What is the defining feature of stage T2 / II carcinoma of the uterine corpus?

  1. Involvement of bladder or rectal mucosa
  2. Involvement of cervical epithelium
  3. Involvement of cervical stromal tissue
  4. Involvement of vagina or parametrial tissue
  5. Myometrial invasion involving 50% of the uterine wall or more
Board review style answer #2
C. Involvement of cervical stromal tissue. Cervical stromal involvement by endometrial carcinoma is the defining feature of stage T2 (AJCC) / II (FIGO). Bladder / rectal mucosal involvement, vaginal / parametrial involvement and outer half myometrial invasion are the defining features of stages T4 / IVA, T3b / IIIB and T1b / IB, respectively. Cervical involvement confined to the epithelium does not affect staging.

Comment here

Reference: Uterus - Staging-carcinoma and carcinosarcoma

Staging-sarcoma

Pathologic TNM staging of sarcoma of the corpus uteri, AJCC 8th edition and FIGO 2018 update
Definition / general
  • Both AJCC Cancer Staging Manual (8th edition) and the International Federation of Gynecology and Obstetrics (Fédération Internationale de Gynécologie et d'Obstétrique, FIGO) 2018 update have a new chapter on staging of uterine sarcomas, with staging classifications separate from uterine carcinoma
  • FIGO uses surgical / pathologic staging for uterine sarcomas; the diagnosis of uterine sarcoma is often difficult based on clinical and radiologic findings
  • Initial stage should not be changed due to disease progression or recurrence
  • Staging classification of adenosarcoma differs from that of leiomyosarcoma, endometrial stromal sarcoma and other pure sarcomas of the uterus
    • Most adenosarcomas arise in the endometrium and stage depends on the presence and depth of myometrial invasion
    • Leiomyosarcoma and endometrial stromal sarcoma, on the other hand, are universally myoinvasive and stage depends on tumor size and extrauterine spread
  • AJCC / TNM system now includes N0(i+) category for isolated tumor cells (ITCs)
Essential features
Primary tumor [pT] and FIGO () stage
Uterine leiomyosarcoma and endometrial stromal sarcoma
  • pTX: primary tumor cannot be assessed
  • pT0: no evidence of primary tumor
  • pT1 (I): tumor confined to the uterus
    • pT1a (IA): tumor ≤ 5 cm in greatest dimension
    • pT1b (IB): tumor > 5 cm
  • pT2 (II): tumor extends beyond the uterus, within the pelvis
    • pT2a (IIA): tumor involves adnexa
    • pT2b (IIB): tumor involves other pelvic tissues
  • pT3 (III): tumor infiltrates abdominal tissues (lesions must not just protrude into abdominal cavity)
    • pT3a (IIIA): tumor infiltrates abdominal tissues in 1 site
    • pT3b (IIIB): tumor infiltrates abdominal tissues in > 1 site
  • pT4 (IVA): tumor invades bladder or rectum

Uterine adenosarcoma
  • pTX: primary tumor cannot be assessed
  • pT0: no evidence of primary tumor
  • pT1 (I): tumor limited to the uterus
    • pT1a (IA): tumor limited to the endometrium / endocervix
    • pT1b (IB): tumor invades < 50% of the myometrium
    • pT1c (IC): tumor invades ≥ 50% of the myometrium
  • pT2 (II): tumor extends beyond the uterus, within the pelvis
    • pT2a (IIA): tumor involves adnexa
    • pT2b (IIB): tumor involves other pelvic tissues
  • pT3 (III): tumor infiltrates abdominal tissues (lesions must not just protrude into abdominal cavity)
    • pT3a (IIIA): tumor infiltrates abdominal tissues in 1 site
    • pT3b (IIIB): tumor infiltrates abdominal tissues in > 1 site
  • pT4 (IVA): tumor invades bladder or rectum
Regional lymph nodes [pN] and FIGO () stage
  • pNX: regional lymph nodes cannot be assessed
  • pN0: no regional lymph node metastasis
    • pN0(i+): isolated tumor cells in regional lymph node(s) ≤ 0.2 mm
  • pN1 (IIIC): regional lymph node metastasis
Distant metastasis [pM] and FIGO () stage
  • pM0: no distant metastasis
  • pM1 (IVB): distant metastasis (excludes adnexa, pelvic and abdominal tissues)
Stage grouping and FIGO stage
Uterine leiomyosarcoma and endometrial stromal sarcoma
Stage I: T1 N0 M0
Stage IA: T1a N0 M0
Stage IB: T1b N0 M0
Stage II: T2 N0 M0
Stage IIIA: T3a N0 M0
Stage IIIB: T3b N0 M0
Stage IIIC: T1 - 3 N1 M0
Stage IVA: T4 any N M0
Stage IVB: any T any N M1

Uterine adenosarcoma
Stage I: T1 N0 M0
Stage IA: T1a N0 M0
Stage IB: T1b N0 M0
Stage IC: T1c N0 M0
Stage II: T2 N0 M0
Stage IIIA: T3a N0 M0
Stage IIIB: T3b N0 M0
Stage IIIC: T1 - 3 N1 M0
Stage IVA: T4 any N M0
Stage IVB: any T any N M1
Board review style question #1
Currently, staging of uterine sarcoma:

  1. Divides stage T3 / III disease depending on the number of abdominal sites involved by tumor
  2. Follows the same staging classification of uterine carcinoma
  3. Has a unified staging classification for pure mesenchymal and mixed epithelial mesenchymal tumors
  4. Incorporates lymphovascular space invasion as a staging criterion
  5. Incorporates tumor grade as a staging criterion
Board review style answer #1
A. Divides stage T3 / III disease depending on the number of abdominal sites involved by tumor (IIIA: 1 site involved; IIIB: > 1 site involved). The 8th edition of AJCC and FIGO (2015 and 2018 updates) have a separate staging classification for uterine sarcomas, separate from uterine carcinoma staging. It contains two staging systems: one for leiomyosarcoma and endometrial stromal sarcoma (and one can guess, other pure uterine sarcomas) and a second one for adenosarcoma. Carcinosarcoma is staged as a uterine carcinoma. Tumor grade and lymphovascular space invasion are not required for staging purposes.

Comment here

Reference: Uterus - Staging - corpus uteri - sarcoma

Undifferentiated / dedifferentiated carcinoma (endometrium / ovary)
Definition / general
Essential features
  • At least focal sheet-like growth pattern with lack of glandular or papillary differentiation
  • High mitotic and proliferation indices
  • Absence or significantly diminished expression of markers of differentiation (e.g. CK7, PAX8, ER and claudin4)
  • Presence of a differentiated component (usually low grade endometrioid adenocarcinoma) is necessary for the diagnosis of dedifferentiated carcinoma
  • Loss of a core SWI / SNF protein is helpful in confirming the diagnosis but not essential to make the diagnosis
ICD coding
  • ICD-O:
    • 8020/3 - carcinoma, undifferentiated, NOS
    • 8020/3 - dedifferentiated carcinoma
  • ICD-11:
    • 2C73.Y & XH1YY4 - other specified malignant neoplasms of the ovary & carcinoma, undifferentiated, NOS
    • 2C73.Y & XH5R16 - other specified malignant neoplasms of the ovary & dedifferentiated carcinoma
    • 2C76.Y & XH1YY4 - other specified malignant neoplasms of the corpus uteri & carcinoma, undifferentiated, NOS
    • 2C76.Y & XH5R16 - other specified malignant neoplasms of the corpus uteri & dedifferentiated carcinoma
Epidemiology
Sites
  • Endometrium
  • Ovary
Pathophysiology
  • Likely evolves from a differentiated endometrial or ovarian carcinoma secondary to disrupted epigenetic modulation
  • Inactivation of the SWI / SNF chromatin remodeling complex is common
  • Often associated with mismatch repair deficiency
Etiology
  • No associated environmental risk factors
Clinical features
  • Usually presents at advanced stage
  • Abdominal pain and swelling
Diagnosis
  • There are no established tests to screen for endometrial and ovarian malignancies where the vast majority of undifferentiated and dedifferentiated carcinomas occur
  • When clinical suspicion arises, abdominal ultrasound and computed tomography scans are useful adjunct
  • Definitive diagnosis requires biopsy tissue
Radiology description
  • Large, solid adnexal mass with variable hemorrhage and necrosis
Prognostic factors
Case reports
Treatment
  • Primary ovarian tumor treated primarily with chemotherapy followed by surgery
  • Total abdominal hysterectomy and bilateral salpingo-oophorectomy (TAH BSO) with or without lymphadenectomy
  • Emerging evidence supports that SWI / SNF deficient undifferentiated and dedifferentiated carcinoma does not respond to the conventional platinum based regimens (J Pathol Clin Res 2021;7:144)
Gross description
  • Large, solid mass with extensive tumor necrosis
Gross images

Images hosted on other servers:
Surgical specimen

Surgical specimen

Microscopic (histologic) description
  • Undifferentiated component:
    • Diffuse, sheet-like growth pattern with lack of glandular or papillary architecture (occasionally glandular or papillary structures from the differentiated component may be entrapped)
    • Comprised of a proliferation of monomorphic, medium sized cells
    • Abrupt keratinization may be seen
    • Cells frequently show discohesion (at least focally)
    • Variable amount of cytoplasm, sometimes with rhabdoid features
    • Brisk mitoses
    • Necrosis frequently present
    • Some cases may show focal spindling and myxoid changes
  • Differentiated component:
    • Typically low grade endometrioid adenocarcinoma
    • Rarely high grade endometrioid adenocarcinoma, high grade serous carcinoma or clear cell carcinoma
Microscopic (histologic) images

Contributed by Basile Tessier-Cloutier, M.D.
Dedifferentiated ovarian carcinoma

Undifferentiated and differentiated components

Undifferentiated ovarian carcinoma

Solid architecture

Undifferentiated ovarian carcinoma

Discohesive rhabdoid cells

ARID1B

ARID1B

Positive stains
Negative stains
  • In the differentiated component (if present):
    • p53 wild type pattern (patchy negative to intermediate nuclear staining)
      • Uncommonly the differentiated component may be high grade, which can be associated with a mutant p53 IHC pattern
    • p16 (patchy negative to weak nuclear and cytoplasmic staining)
      • Uncommonly the differentiated component may be high grade, which can be associated with a diffuse p16 IHC pattern
    • Mismatch repair deficiency (MLH1, PMS2, MSH2, MSH6) is common (~50%)
  • In the undifferentiated component:
    • CK7, PAX8, ER, WT1, claudin4 (may have weak or focal expression)
    • SWI / SNF complex loss of protein expression (BRG1, INI1 or co-loss of ARID1A and ARID1B)
    • Usually p53 wild type (negative to patchy intermediate nuclear staining)
    • p16 (negative to patchy weak nuclear and cytoplasmic staining)
    • Mismatch repair deficiency (MLH1, PMS2, MSH2, MSH6) is common (~50%)
Molecular / cytogenetics description
  • Commonly shows an inactivating mutation in one of the core SWI / SNF complex genes, resulting in the loss of expression of BRG1 (SMARCA4 gene), INI1 (SMARCB1 gene) or co-loss of ARID1A and ARID1B
  • Most cases are associated with mismatch repair deficiency and of those with MLH1 deficiency, almost all show MLH1 promoter hypermethylation
  • Mutations in other genes associated with endometrioid carcinoma, such as PTEN, PIK3CA, KRAS and CTNNB1, are often reported
Sample pathology report
  • Uterus, fallopian tubes and ovaries, hysterectomy and bilateral salpingo-oophorectomy:
    • Endometrial dedifferentiated carcinoma (see synoptic report)
    • Associated with an endometrioid adenocarcinoma FIGO grade 1
    • Mismatch repair status: abnormal (MLH1 lost)
  • Uterus, fallopian tubes and ovaries, hysterectomy and bilateral salpingo-oophorectomy:
    • Undifferentiated carcinoma of the ovary (see synoptic report)
    • Mismatch repair status: abnormal (MLH1 lost)
Differential diagnosis
Additional references
Board review style question #1

This is a representative image from a solitary ovarian tumor in a 75 year old woman. The tumor shows absent INI1 immunohistochemistry expression. What is the most likely diagnosis?

  1. Carcinosarcoma
  2. High grade serous carcinoma of the ovary
  3. Primary diffuse large B cell lymphoma of the ovary
  4. Small cell carcinoma of the ovary hypercalcemic type
  5. Undifferentiated carcinoma of the ovary
Board review style answer #1
E. Undifferentiated carcinoma of the ovary

Comment Here

Reference: Undifferentiated / dedifferentiated carcinoma (endometrium / ovary)
Board review style question #2

This is a representative image from an endometrial tumor in a 75 year old woman. What is the most likely diagnosis?

  1. Carcinosarcoma of the ovary
  2. Collision tumor
  3. Endometrial dedifferentiated carcinoma
  4. High grade endometrioid adenocarcinoma of the ovary
  5. High grade serous carcinoma of the ovary
Board review style answer #2
C. Endometrial dedifferentiated carcinoma

Comment Here

Reference: Undifferentiated / dedifferentiated carcinoma (endometrium / ovary)

Undifferentiated uterine sarcoma
Definition / general
  • Undifferentiated uterine sarcoma is a diagnosis of exclusion applied to malignant mesenchymal tumors of the uterus with no lineage specific morphologic or immunohistochemical features and no entity defining molecular alterations
Essential features
  • Rare, aggressive malignant mesenchymal tumor, predominantly affecting postmenopausal women
  • Diagnosis of exclusion, requiring thorough morphologic, immunohistochemical and molecular evaluation of a comprehensive differential
  • Diagnosis of undifferentiated uterine sarcoma has decreased as novel molecular subtypes of uterine sarcoma are codified
Terminology
  • Undifferentiated endometrial sarcoma is discouraged
    • More general undifferentiated uterine sarcoma terminology was introduced in the 2014 WHO to acknowledge that these tumors are not necessarily of endometrial origin
ICD coding
  • ICD-10:
    • C54.1 - malignant neoplasm of endometrium
    • C54.2 - malignant neoplasm of myometrium
    • C54.3 - malignant neoplasm of fundus uteri
    • C54.8 - malignant neoplasm of overlapping sites of corpus uteri
    • C54.9 - malignant neoplasm of corpus uteri, unspecified
Epidemiology
Sites
  • Uterus
Clinical features
Diagnosis
  • Diagnosis of exclusion, rendered after comprehensive morphologic, immunohistochemical and molecular testing rule out other defined entities
  • Diagnosis best rendered on a well sampled resection specimen, as focal diagnostic findings may be missed in biopsies of uterine sarcoma (Am J Surg Pathol 2017;41:1231)
Prognostic factors
Case reports
Treatment
  • Complete surgical excision is first line treatment, if possible (Gynecol Oncol 2012;127:27, BMC Cancer 2018;18:1247)
  • Adjuvant chemotherapy (gemcitabine / docetaxel or doxorubicin based regimens) is standard
  • Additional chemotherapy regimens may be used for recurrent / progressive disease
  • Response to chemotherapy is generally short lived
Gross description
  • Generally large (median, 10 cm) (Clin Transl Oncol 2021;23:1210)
  • Fleshy, pink-tan cut surface
  • Gross necrosis and hemorrhage common
  • Deep myometrial infiltration common
  • Thorough gross sampling (at least 1 section per cm) is critical to confidently excluding differentiated tumor foci, which would preclude diagnosis of undifferentiated uterine sarcoma
Gross images

Contributed by Ayse Ayhan, M.D., Ph.D.
Missing Image

Polypoid gray-white uterine mass

Microscopic (histologic) description
  • Conceptually subdivided into tumors with uniform morphology and those with pleomorphic morphology (Am J Surg Pathol 2008;32:1228)
  • Undifferentiated uterine sarcoma with uniform morphology
    • Many tumors with uniform morphology are more appropriately classified under novel molecular diagnoses:
    • However, some uterine sarcomas with uniform spindle or epithelioid morphology may remain unclassifiable after comprehensive testing (Am J Surg Pathol 2014;38:1161)
      • In these cases, a diagnosis of undifferentiated uterine sarcoma with uniform morphology is appropriate, with a descriptive comment
      • Next generation sequencing based gene fusion testing may be considered to identify novel entities
  • Undifferentiated uterine sarcoma with pleomorphic morphology
    • Spindle, epithelioid or polygonal cells with marked nuclear atypia and pleomorphism, frequent multinucleation and macronucleoli (Am J Surg Pathol 2017;41:1231)
  • Tumors with uniform or with pleomorphic morphology characteristically show (Clin Transl Oncol 2021;23:1210):
    • Destructive myometrial invasion
    • Brisk mitotic activity (median, 25 per 10 high power fields)
    • Tumor necrosis (~90%)
    • Lymphovascular invasion (~50%)
Microscopic (histologic) images

Contributed by David B. Chapel, M.D.

Epithelioid morphology

Spindled morphology

Spindled morphology


Pleomorphic morphology

Positive stains
Negative stains
Molecular / cytogenetics description
  • TP53 mutations and complex copy number alterations are common (JCI Insight 2017;2:e94033)
  • Rare cases may harbor MDM2 amplification (Int J Gynecol Pathol 2015;34:576)
  • On RNA expression analysis, undifferentiated uterine sarcoma appears to cluster more closely with leiomyosarcoma than with endometrial stromal sarcoma (Mod Pathol 2021;34:1008)
  • 4 molecular subtypes identified in 1 study, based on differential activation of (Clin Cancer Res 2019;25:2155):
    • Genital tract development pathways
    • Extracellular matrix pathways
    • Muscle function pathways
    • Proliferation pathways
    • These subtypes may have prognostic significance but do not currently play a role in routine diagnosis
  • By definition, undifferentiated uterine sarcoma lacks disease defining molecular alterations, including:
    • Fusions in YWHAE, BCOR, PHF1, JAZF1, NTRK, COL1A1, ESR1, GREB1, TFE3
    • Internal tandem duplication of BCOR
    • SWI / SNF (SMARCA4, SMARCB1) deficiency
    • TSC1/2 alterations
    • Recognition of these alterations may permit use of targeted therapies
Sample pathology report
  • Uterus, cervix, bilateral tubes and ovaries, total abdominal hysterectomy with bilateral salpingo-oophorectomy:
    • Cervix:
      • No significant pathologic change.
    • Endomyometrium:
      • Undifferentiated uterine sarcoma (12 cm) (see comment)
      • Lymphovascular invasion is present.
    • Uterine serosa:
      • Negative for tumor.
    • Fallopian tubes and ovaries:
      • No significant pathologic change.
    • Comment: Gross examination reveals a poorly circumscribed, fleshy, 12 cm mass, diffusely infiltrating the myometrium. On microscopic examination, the tumor comprises sheets of polygonal cells with abundant eosinophilic cytoplasm and pleomorphic nuclei with prominent macronucleoli. Numerous multinucleated tumor cells are seen. Mitoses number up to 35 per 10 high power fields. There is extensive tumor necrosis (~60% of the tumor volume) and lymphovascular invasion.
    • On immunohistochemical stains, the tumor shows patchy variable CD10 positivity, strong diffuse mutant pattern p53 and strong diffuse p16. Tumor cells are negative for SMA, desmin, caldesmon, ER, PR, cyclin D1, BCOR, S100, MelanA, HMB45 and cytokeratin AE1 / AE3 / CAM5.2. SMARCA4 and SMARCB1 / INI1 are retained.
    • The morphologic and immunophenotypic findings are mostly in keeping with a diagnosis of undifferentiated uterine sarcoma. I particularly considered the possibility of a pleomorphic leiomyosarcoma or malignant PEComa. However, immunophenotype does not support these diagnoses. The tumor appears confined to the uterine corpus in this specimen (pT1b); however, correlation with surgical and radiographic findings is necessary.
Differential diagnosis
  • Spindle cell leiomyosarcoma:
    • Long intersecting fascicles of spindle cells with cigar shaped nuclei and eosinophilic cytoplasm
    • SMA, desmin or caldesmon IHC positive, at least focally
    • Mutations in TP53, ATRX, RB1 and MED12 common
  • Myxoid leiomyosarcoma:
  • High grade endometrial stromal sarcoma with BCOR rearrangement:
    • Uniform spindle cell population with mild atypia in myxoid stroma
    • Cyclin D1 and CD10 IHC consistently positive
    • BCOR IHC positive in ~50%
    • May be focally SMA positive but often desmin and caldesmon negative
    • BCOR fusion (most often with ZC3H7B)
  • High grade endometrial stromal sarcoma with BCOR internal tandem duplication:
    • Variably spindle to epithelioid cells with moderate to marked atypia but typically without frank pleomorphism, in myxoid stroma
    • Low grade fibromyxoid component often present
    • Cyclin D1, BCOR and CD10 IHC positive
    • May be focally desmin positive but caldesmon and SMA negative
    • BCOR internal tandem duplication (ITD) in exon 15
  • High grade endometrial stromal sarcoma with YWHAE-NUTM2A/B fusion:
    • Often biphasic morphology
      • High grade epithelioid component: uniform population of atypical epithelioid cells, arranged in nests surrounded by delicate capillary vasculature
      • Low grade spindle component: uniform population of bland stubby spindle cells, resembling low grade endometrial stromal sarcoma
      • Relative frequency:
        • Biphasic tumors most common
        • Occasional tumors show only the high grade epithelioid component
        • Rarely, only the low grade spindle component is present
    • Cyclin D1 and BCOR IHC consistently positive in the high grade epithelioid component and negative or focally positive in the low grade spindle component
    • CD10 negative in high grade epithelioid component but positive in low grade spindle component
    • YWHAE-NUTM2A/B fusion on molecular testing
  • NTRK fusion uterine sarcoma:
    • Uniform spindle cell population with mild to moderate atypia and haphazard or herringbone architecture
    • Most common in the cervix of young (< 40) women
    • S100 and pan-TRK IHC consistently positive but nonspecific
    • Desmin, ER and PR IHC negative
    • NTRK fusions (most often NTRK1-TPM3) define entity
  • COL1A1-PDGFB fusion uterine sarcoma:
    • Exceptionally rare
    • Uniform spindle cell population with mild to moderate atypia and storiform architecture (resembling dermatofibrosarcoma protuberans)
    • CD34 IHC consistently positive
    • S100, desmin, ER and PR IHC negative
    • COL1A1-PDGFB fusion defines entity
  • Malignant inflammatory myofibroblastic tumor:
    • Spindle cells with moderate to marked atypia and inflammatory infiltrate
    • Infiltrative borders, brisk mitoses and tumor necrosis may be present
    • ALK IHC positive in almost all cases, most often with granular cytoplasmic staining
    • ALK fusion present in almost all cases
    • Rare uterine IMT may show RET, ETV6-NTRK3 or ROS1 fusions
  • Malignant perivascular epithelioid cell tumor:
    • Variable epithelioid to spindle cells, often with frank nuclear pleomorphism, multinucleation and macronucleoli
    • Smooth muscle (SMA, desmin or caldesmon) and melanocytic (HMB45, MelanA, MITF, PNL2) markers positive
    • TSC1/2 alterations or TFE3 fusions on molecular testing
  • Epithelioid leiomyosarcoma:
    • Uniform to pleomorphic epithelioid cells with eosinophilic cytoplasm, often with an admixed spindle component
    • SMA, desmin or caldesmon positive
    • PGR fusions present in a subset
      • This subset characteristically shows uniform cytomorphology, with a rhabdoid eosinophilic cytoplasmic inclusion and eccentric nucleus
  • Uterine tumor resembling ovarian sex cord tumor with GREB1 fusion:
    • Uniform epithelioid cells with vesicular chromatin and prominent nucleoli, predominantly arranged in solid sheets, often with focal sex cord patterns (sertoliform cords, tubules, retiform structures)
      • Spindled growth may also be present
    • Polyphenotypic: smooth muscle markers, cytokeratin, sex cord markers (calretinin, inhibin, SF1, WT1, CD56, CD99), CD10 and hormone receptors typically positive, though expression is variable
    • Defined by GREB1 fusions (usually with NCOA1/2)
    • Precise clinical and biologic relationship to classic UTROSCT remains unsettled
  • Pleomorphic rhabdomyosarcoma:
    • Primary pleomorphic rhabdomyosarcoma of the uterus is exceptionally rare
    • Polygonal to spindle cells with marked nuclear pleomorphism, frequent multinucleation and abundant eosinophilic cytoplasm
    • Desmin, myogenin, MyoD1 IHC positive
    • Must be differentiated from rhabdomyosarcomatous differentiation in carcinosarcoma or adenosarcoma
  • SMARCA4 deficient uterine sarcoma:
    • Uniform epithelioid cells with moderate nuclear atypia and eosinophilic cytoplasm, arranged in solid sheets
    • Phyllodiform growth may be present
    • SMARCA4 (or more rarely, SMARCB1 / INI1) IHC negative (i.e. abnormal loss of expression)
    • SMARCA4 loss of function alterations on molecular testing
    • Subset associated with germline SMARCA4 mutations
  • Undifferentiated / dedifferentiated or poorly differentiated carcinoma:
  • Carcinosarcoma:
    • Extensive sampling will reveal a malignant epithelial component
    • In case of complete overgrowth by undifferentiated sarcoma, distinction will not be possible
  • Adenosarcoma with sarcomatous overgrowth:
    • Extensive sampling will reveal a component with conventional adenosarcomatous morphology
    • In case of complete overgrowth by undifferentiated sarcoma, distinction will not be possible
  • Melanoma:
    • Protean histomorphology; often pleomorphic polygonal cells with macronucleoli
    • Melanocytic markers (MelanA, HMB45, S100, SOX10) IHC positive
Board review style question #1

A 73 year old woman presents with postmenopausal bleeding. Ultrasound shows a 12 cm, sonographically heterogeneous uterine mass. Gross examination shows extensive necrosis and hemorrhage. A representative photomicrograph from the tumor is shown. Immunohistochemical stains for SMA, desmin, caldesmon, ALK, CD34, S100, cyclin D1, BCOR, ER, PR, SOX10, HMB45, MelanA, MyoD1, myogenin and broad spectrum cytokeratins are negative. SMARCB1 / INI1 and SMARCA4 immunostains show retained / intact expression. A CD10 immunostain shows patchy positivity and a p53 immunostain shows strong diffuse mutant pattern expression. Which of the following is the most appropriate diagnosis?

  1. Epithelioid leiomyosarcoma
  2. Malignant inflammatory myofibroblastic tumor
  3. Melanoma
  4. Pleomorphic rhabdomyosarcoma
  5. Undifferentiated uterine sarcoma
Board review style answer #1
E. Undifferentiated uterine sarcoma. The clinical history and gross findings are suggestive of a uterine malignancy but they are not specific to a particular diagnosis. Microscopic examination shows a malignant neoplasm with a loose, vaguely spindled background and numerous superimposed pleomorphic tumor giant cells. An extensive immunohistochemical workup shows no identifiable line of differentiation, ruling out epithelioid leiomyosarcoma (negative SMA, desmin and caldesmon), malignant inflammatory myofibroblastic tumor (negative SMA, desmin, caldesmon, ALK), melanoma (negative S100, SOX10, HMB45, MelanA) and pleomorphic rhabdomyosarcoma (negative desmin, myogenin, myoD1). The other stains also rule out malignant PEComa (negative smooth muscle and melanocytic markers) and high grade endometrial stromal sarcoma (negative BCOR and cyclin D1). A novel fusion related uterine sarcoma (e.g. NTRK fusion sarcoma) is unlikely, given the pleomorphic cytomorphology and mutant pattern p53 immunostaining. Having excluded all items on this lengthy differential diagnosis, a diagnosis of undifferentiated uterine sarcoma is most appropriate.

Comment Here

Reference: Undifferentiated uterine sarcoma
Board review style question #2
Which of the following statements regarding the diagnosis of undifferentiated uterine sarcoma is true?

  1. Patchy CD10 staining rules out a diagnosis of undifferentiated uterine sarcoma
  2. Undifferentiated uterine sarcoma characteristically shows a myomelanocytic immunoprofile
  3. Undifferentiated uterine sarcoma is a diagnosis of exclusion applied to malignant uterine mesenchymal tumors with no lineage specific morphologic or immunohistochemical and no entity defining molecular alterations
  4. Undifferentiated uterine sarcoma is defined by YWHAE-NUTM2A/B fusion
  5. Undifferentiated uterine sarcoma most commonly occurs in the cervix of young women (< 40 years old)
Board review style answer #2
C. Undifferentiated uterine sarcoma is a diagnosis of exclusion applied to malignant uterine mesenchymal tumors with no lineage specific morphologic or immunohistochemical and no entity defining molecular alterations. Patchy CD10 staining may be seen in undifferentiated uterine sarcoma (answer A) and is considered nonspecific. PEComa characteristically shows a myomelanocytic immunprofile (answer B), which should not be seen in undifferentiated uterine sarcoma. YWHAE-NUTM2A/B fusions define a subset of high grade endometrial stromal sarcomas (answer D); this fusion is not compatible with a diagnosis of undifferentiated uterine sarcoma. Undifferentiated uterine sarcoma principally affects postmenopausal patients (answer E); in contrast, NTRK fusion uterine sarcomas most commonly occur in the cervix of young women (< 40 years old).

Comment Here

Reference: Undifferentiated uterine sarcoma

Uterine tumors resembling ovarian sex cord tumors
Definition / general
  • Rare neoplasms that resemble ovarian sex cord tumors, without a component of recognizable endometrial stroma (IARC: WHO Classification of Tumors of the Female Reproductive Organs, 4th Edition, 2014)
  • Accounts for less than 0.5% of all uterine malignancies and 10% - 15% of mesenchymal uterine malignancies (Crum: High Yield Pathology - Gynecologic and Obstetric Pathology, 2016)
  • Etiology uncertain and most have benign behavior without infiltrative properties
  • 1945: Morehead and Bowman first described a case of a uterine neoplasm resembling a granulosa cell tumor
  • 1976: Clement and Scully clarified the concept of sex cord differentiation of uterine tumors and categorized into two groups:
    • Group I is endometrial stromal tumors with foci of sex cord differentiation (ETSCLE) < 50% (associated with recurrences and metastases)
    • Group II composed predominantly or exclusively by sex cord-like elements; this latest group continues to be classified as UTROSCTs
  • Miscellaneous category in WHO classification
Essential features
  • Rare uterine tumor of uncertain histogenesis; morphologically shows overlap with ovarian sex cord tumors, affecting perimenopausal or menopausal women (mean age 51)
  • Differential diagnosis crucial; different behavior
  • Present mostly as intramural, less frequently submucosal or subserosal or polypoid / intracavitary masses
  • Organized in sex cord structures (sheets, cord, nests, trabeculae, tubules) composed of epithelioid-looking cells with scant / abundant eosinophilic / clear cytoplasm and bland nuclei with minimal atypia and rare mitoses
  • Coexpression of epithelial (KL1, CK AE1 / AE3, CAM5.2, EMA), smooth muscle (smooth muscle actin, desmin, h-caldesmon, smooth muscle myosin heavy chain, histone deacetylase-8), sex cord markers (WT1, calretinin, inhibin, CD99, MelanA, CD56, FOXL2, SF1) and hormone receptors (ER, PR, AR)
  • Absence of JAZF1 / SUZ12 fusion (JAZF1-JJAZ1) and PFH1 gene rearrangements distinguish these neoplasms from endometrial stromal tumors
  • Treated by surgical removal, based on age / parity of the patients (total hysterectomy with bilateral adnexectomy or conservative surgery with hysterectomy / mass resection)
  • Benign behavior; however, low rates of recurrence (5%) have been reported and rarely development of metastases can occur
ICD coding
  • 8590/1
Epidemiology
  • Perimenopausal or menopausal women (22 - 84 years); mean age is 51
  • < 1% of uterine mesenchymal tumors
Sites
Etiology
  • Tumors of uncertain histogenesis (miscellaneous category in latest WHO classification)
  • Postulated theories include (Int J Gynecol Pathol 2016;35:301):
    • Derivation from ovarian sex cord cells which have been displaced during embryogenesis
    • Derivation from uncommitted mesenchymal stem cells
    • Overgrowth of sex cord elements within endometrial stromal neoplasm or adenosarcoma (however please notice that these tumors do not have the cytogenetic abnormalities found in stromal neoplasms)
Clinical features
Diagnosis
  • Based on histomorphologic features including a predominant pattern of the cords, nests and trabeculae resembling sex cord tumors of the ovary and immunophenotype, characterized by coexpression of epithelial, smooth muscle, sex cord markers and steroid receptors
  • Imaging studies are not diagnostic; histopathology is the gold standard (Arch Pathol Lab Med 2013;137:1832)
Radiology description
  • They can be diagnosed with US, CT and MRI
  • No specific image findings: at transvaginal pelvic ultrasound they may be seen in an normal or enlarged uterus and may appear as myometrial masses, with myomatous features or as masses protruding in the endometrial cavity suggesting a polypoid lesion
Prognostic factors
Case reports
Treatment
Gross description
  • Intramural / submucosal / subserosal nodules or polypoid tumors growing in the endometrial cavity
  • Solid, round, well circumscribed masses
  • Average 6 cm; ranging from 2 to 24 cm
  • Yellow, tan, grayish white surface; firm to soft to rubbery consistency
  • Cut surface grayish yellow to white
  • Rarely predominantly cystic
  • Hemorrhage can be seen; necrosis unusual
Gross images

Contributed by Ayse Ayhan, M.D., Ph.D.

Intramural lesion

Microscopic (histologic) description
  • Usually well circumscribed but unencapsulated; may have a pseudoinfiltrative appearance due to incorporated smooth muscle bundles; true myometrial invasion rare
  • Organized in sheets, cords, nests, trabeculae, hollow or solid tubules with repetitive pattern of cordlike / tubular growth; more rarely has retiform or glomeruloid appearance or papillae and solid pattern predominance
  • Neoplastic cells are small, round to ovoid with monotonous nuclei, inconspicuous nucleoli, mild nuclear hyperchromasia, rare nuclear grooves
  • Call-Exner-like bodies may be rarely present
  • Scant intervening stroma (hyalinised, fibroblastic or edematous)
    • In some tumors endometrial stromal-type cells or benign appearing smooth muscle may be present; rare findings are a sparse lymphocytic infiltrate accompanied by foamy histiocytes, a few multinucleated giant cells, hemosiderin deposition or cholesterol crystals
  • Occasionally vascular invasion, heterologous elements and necrosis
Microscopic (histologic) images

Contributed by Ayse Ayhan, M.D., Ph.D.
Varied architecture, bland cytology Varied architecture, bland cytology Varied architecture, bland cytology

Varied architecture, bland cytology

Varied architecture, bland cytology Varied architecture, bland cytology

Positive sex cord and epithelial markers



Images hosted on other servers:

Various images


Tumor cells resemble granulosa cells

Various images

Cytology description
  • Cells resembling epithelial cells, with scant cytoplasm or abundant eosinophilic / clear (including vacuolated) / foamy cytoplasm, reminiscent of Sertoli cell or granulosa cell tumors
  • Can show rhabdoid features (J Clin Pathol 2007;60:1148) with abundant eosinophilic cytoplasm and eccentric nuclei
  • Minimal atypia; rare mitoses (ranging from < 1 to 5 / 10 high power fields)
  • Ovoid and small nuclei with irregular contours (sometimes grooved)
  • Finely distributed chromatin with small to indistinct nucleoli
  • Leydig-like cells may be present
Positive stains
Electron microscopy description
  • Cell junctions, desmosome-like junctions, tonofilaments, lumina formation, microvilli (indicative of epithelial differentiation)
  • Sex cord like features (nuclear indentation, abundant intracellular filaments, endoplasmic reticulum [granulosa cells], intracytoplasmic lipid droplets
  • No dense bodies, subplasmalemmal densities or pinocytotic vesicles (indicating no smooth muscle differentiation)
  • Reference: Ultrastruct Pathol 2010;34:16
Molecular / cytogenetics description
Differential diagnosis

Uterine tumors resembling ovarian sex cord tumors
Definition / general
  • Rare neoplasms that resemble ovarian sex cord tumors, without a component of recognizable endometrial stroma
Essential features
  • Rare uterine tumor of uncertain histogenesis; morphologically shows overlap with ovarian sex cord tumors, affecting perimenopausal or menopausal women (mean age 52)
  • Absence of JAZF1::SUZ12 fusion (JAZF1::JJAZ1) and PFH1 gene rearrangements distinguish these neoplasms from endometrial stromal tumors
  • Benign behavior; however, low rates of recurrence (5%) have been reported and rarely development of metastases can occur
  • Currently within the mesenchymal category in the WHO classification of tumors of the uterine corpus
Terminology
  • Uterine tumor resembling ovarian sex cord tumor (UTROSCT), endometrial stromal tumor resembling ovarian sex cord tumor
ICD coding
  • ICD-O: 8590/1 - sex cord gonadal stromal tumor, NOS
Epidemiology
Sites
Pathophysiology
  • Postulated theories include (Int J Gynecol Pathol 2016;35:301)
    • Derivation from ovarian sex cord cells which have been displaced during embryogenesis
    • Derivation from uncommitted mesenchymal stem cells
    • Derivation from pluripotent endometrial stromal cells
    • Overgrowth of sex cord elements within endometrial stromal neoplasm or adenosarcoma (however please notice that these tumors do not have the cytogenetic abnormalities found in stromal neoplasms)
Etiology
  • Tumors of unknown histogenesis
Diagrams / tables
N/A
Clinical features
Diagnosis
  • Based on histomorphologic features including a predominant pattern of the cords, nests and trabeculae resembling sex cord tumors of the ovary and immunophenotype, characterized by coexpression of epithelial, smooth muscle, sex cord markers and steroid receptors
  • Imaging studies are not diagnostic; histopathology is the gold standard (Arch Pathol Lab Med 2013;137:1832)
Laboratory
N/A
Radiology description
  • They can be diagnosed with ultrasound, CT and MRI
  • No specific image findings (Oncol Lett 2016;11:1496)
    • At transvaginal pelvic ultrasound, they may be seen in a normal or enlarged uterus and may appear as myometrial masses, with myomatous features or as masses protruding in the endometrial cavity suggesting a polypoid lesion
    • UTROSCT reported to exhibit high signal intensity on MRI diffusion weighted images in contrast to generally low signal associated with leiomyoma (Magn Reson Med Sci 2019;18:113)
Radiology images
N/A
Prognostic factors
Case reports
Treatment
Clinical images
N/A
Gross description
  • Intramural / submucosal / subserosal nodules or polypoid tumors growing in the endometrial cavity
  • Mainly solid, round, well circumscribed masses
  • Average of 6 cm; ranging from 2 to 24 cm
  • Yellow, tan, grayish white surface; firm to soft to rubbery consistency
  • Cut surface grayish yellow to white
  • Rarely predominantly cystic
  • Hemorrhage can be seen; necrosis unusual
  • Reference: Crum: High Yield Pathology - Gynecologic and Obstetric Pathology, 1st Edition, 2016
Gross images

Contributed by Ayse Ayhan, M.D., Ph.D.

Intramural lesion

Frozen section description
Frozen section images
N/A
Microscopic (histologic) description
  • Usually well circumscribed but unencapsulated; may have a pseudoinfiltrative appearance due to incorporated smooth muscle bundles; true myometrial invasion is rare
  • Organized in sheets, cords, nests, trabeculae, hollow or solid tubules with repetitive pattern of cord-like / tubular growth; more rarely has retiform or glomeruloid appearance or papillae and solid pattern predominance
  • Neoplastic cells are small, round to ovoid with monotonous nuclei, inconspicuous nucleoli, mild nuclear hyperchromasia, rare nuclear grooves, with usually minimal cytologic atypia and low mitotic activity
  • Call-Exner-like bodies may be rarely present
  • Scant intervening stroma (hyalinized, fibroblastic or edematous)
    • In some tumors, endometrial stromal type cells or benign appearing smooth muscle may be present; rare findings are a sparse lymphocytic infiltrate accompanied by foamy histiocytes, a few multinucleated giant cells, hemosiderin deposition or cholesterol crystals
  • Occasionally vascular invasion, heterologous elements and necrosis
  • Reference: Crum: High Yield Pathology - Gynecologic and Obstetric Pathology, 1st Edition, 2016
Microscopic (histologic) images

Contributed by Ayse Ayhan, M.D., Ph.D. and Jian-Jun Wei, M.D.
Varied architecture, bland cytology Varied architecture, bland cytology Varied architecture, bland cytology

Varied architecture, bland cytology

Varied architecture, bland cytology Varied architecture, bland cytology

Positive sex cord and epithelial markers

Vimentin

Vimentin

Virtual slides
I did not see any virtual slides of UTROSCT on the University of Toronto website
Cytology description
  • Cells resembling epithelial cells, with scant cytoplasm or abundant eosinophilic / clear (including vacuolated) / foamy cytoplasm, reminiscent of Sertoli cell or granulosa cell tumors (Diagn Cytopathol 2019;47:603)
  • Can show rhabdoid features with abundant eosinophilic cytoplasm and eccentric nuclei (J Clin Pathol 2007;60:1148)
  • Minimal atypia; rare mitoses (ranging from < 1 - 5/10 high power fields)
  • Ovoid and small nuclei with irregular contours (sometimes grooved)
  • Finely distributed chromatin with small to indistinct nucleoli
  • Leydig-like cells may be present
Cytology images
N/A
Immunofluorescence description
N/A
Immunofluorescence images
N/A
Positive stains
Negative stains
Electron microscopy description
  • Cell junctions, desmosome-like junctions, tonofilaments, lumina formation, microvilli (indicative of epithelial differentiation)
  • Sex cord like features (nuclear indentation, abundant intracellular filaments, endoplasmic reticulum [granulosa cells], intracytoplasmic lipid droplets)
  • No dense bodies, subplasmalemmal densities or pinocytotic vesicles (indicating no smooth muscle differentiation)
  • Reference: Ultrastruct Pathol 2010;34:16
Electron microscopy images
N/A
Molecular / cytogenetics description
Molecular / cytogenetics images
N/A
Videos
N/A
Sample pathology report
  • Uterus, hysterectomy:
    • Uterine neoplasm most consistent with uterine tumor resembling ovarian sex cord tumor (see comment)
    • Comment: Within the uterine corpus is an intramural mass that histologically demonstrates sheets of bland cells with scant to abundant cytoplasm and ovoid nuclei. Minimal cytologic atypia is appreciated, very low mitotic activity is present (1 per 10 high power fields) and no necrosis is seen. The lesion is positive for calretinin, WT1 and inhibin, and negative for HMB45 and CD34. The morphology and immunoprofile are supportive of a diagnosis of uterine tumor resembling ovarian sex cord tumor, a rare uterine neoplasm of unknown etiology.
Differential diagnosis
Board review style question #1
The typical morphology of uterine tumors resembling ovarian sex cord tumors (UTROSCT) includes which of the following features?

  1. Cords
  2. High mitotic activity
  3. Necrosis
  4. Severe cytologic atypia
Board review style answer #1
A. Cords. UTROSCT contains morphology organized in sex cord structures, including sheets, cords, nests, trabeculae and tubules. Answers B - D are incorrect because the other choices are not typically seen in UTROSCT cases.

Comment Here

Reference: Uterine tumors resembling ovarian sex cord tumors
Board review style question #2
BRQ image BRQ image BRQ image


What would be the most expected immunoprofile for this tumor?

  1. Calretinin+, CD10+, HMB45+
  2. Calretinin+, WT1+, HMB45-
  3. Calretinin-, WT1+, HMB45-
  4. HMB45+, chromogranin+, CD34+
Board review style answer #2
B. Calretinin+, WT1+, HMB45-. The typical immunoprofile for UTROSCT is positivity for sex cord markers and negativity for melanoma markers like HMB45. Answers A and D are incorrect because chromogranin, CD34 and CD10 would all be negative in this entity and are not helpful for the diagnosis of UTROSCT. Answer C is incorrect because calretinin would be positive in this entity.

Comment Here

Reference: Uterine tumors resembling ovarian sex cord tumors

WHO classification (pending)
[Pending]
Back to top
Recent Uterus Pathology books

Clement: 2019

Crum: 2015

Damjanov: 2017

Fadare: 2015

Heller: 2015

IARC: 2014

IARC: 2020

Malpica: 2015

Murdock: 2018

Nucci: 2020

Nucci: 2023

Oliva: 2019

Vang: 2017



Find related Pathology books: breast, gynecologic
Image 01 Image 02